英语五年真题分类1阅读理解题及答案

申明敬告: 本站不保证该用户上传的文档完整性,不预览、不比对内容而直接下载产生的反悔问题本站不予受理。

文档介绍

英语五年真题分类1阅读理解题及答案

1 阅读理解 专题一 广告信息类 2018 年 Passage 1 2018 全国卷Ⅰ,6 分 话题:骑自行车在华盛顿特区观光 词数:274 Washington,D.C. Bicycle Tours Cherry Blossom Bike Tour in Washington, D.C. Duration:3 hours This small group bike tour is a fantastic way to see the world-famous cherry trees with beautiful flowers of Washington, D.C. Your guide will provide a history lesson about the trees and the famous monuments where they blossom. Reserve your spot before availability — and the cherry blossoms — disappear! Washington Capital Monuments Bicycle Tour Duration:3 hours(4 miles) Join a guided bike tour and view some of the most popular monuments in Washington, D.C. Explore the monuments and memorials on the National Mall as your guide shares unique facts and history at each stop. Guided tour includes bike, helmet, cookies and bottled water. Capital City Bike Tour in Washington, D.C. Duration:3 hours Morning or Afternoon, this bike tour is the perfect tour for D.C. newcomers and locals looking to experience Washington, D.C. in a healthy way with minimum effort. Knowledgeable guides will entertain you with the most interesting stories about Presidents, Congress, memorials, and parks. Comfortable bikes and a smooth tour route(路线) make cycling between the sites fun and relaxing. Washington Capital Sites at Night Bicycle Tour Duration:3 hours(7 miles) Join a small group bike tour for an evening of exploration in the heart of Washington, D.C. Get up close to the monuments and memorials as you bike the sites of Capitol Hill and the National Mall. Frequent stops are made for photo taking as your guide offers unique facts and history. Tour includes bike, helmet, and bottled water. All riders are equipped with reflective vests and safety lights. 1. Which tour do you need to book in advance? A. Cherry Blossom Bike Tour in Washington, D.C. B. Washington Capital Monuments Bicycle Tour. C. Capital City Bike Tour in Washington, D.C. D. Washington Capital Sites at Night Bicycle Tour. 2. What will you do on the Capital City Bike Tour? A. Meet famous people. B. Go to a national park. C. Visit well-known museums. D. Enjoy interesting stories. 3. Which of the following does the bicycle tour at night provide? A. City maps. B. Cameras. C. Meals. 2 D. Safety lights. Passage 2 2018 全国卷 II,6 分 话题:暑期活动 词数:290 Summer Activities Students should read the list with their parents/carers, and select two activities they would like to do. Forms will be available in school and online for them to indicate their choices and return to school. Before choices are finalised, parents/carers will be asked to sign to confirm their child’s choices. 3 1.Which activity will you choose if you want to go camping? A. OUT. B. WBP. C. CRF. D. POT. 2.What will the students do on Tuesday with Mrs. Wilson? A. Travel to London. B. See a parade and fireworks. C. Tour central Paris. D. Visit the WWⅠ battlefields. 3.How long does Potty about Potter last? A. Two days. B. Four days. C. Five days. D. One week. Passage 3 2018 全国卷 III,6 分 话题:英国霍克庄园和花园 词数:210 Welcome to Holker Hall & Gardens Visitor Information How to Get to Holker By Car: Follow brown signs on A590 from J36, M6. Approximate travel times:Windermere — 20 minutes, Kendal — 25 minutes, Lancaster — 45 minutes, Manchester — 1 hour 30 minutes. By Rail: The nearest station is Cark-in-Cartmel with trains to Carnforth, Lancaster and Preston for connections to major cities & airports. Opening Times Sunday-Friday (closed on Saturday) 11:00am-4:00pm, 30th March-2nd November. Admission Charges Hall & Gardens Gardens Adults:£12.00 £8.00 Groups:£9.00 £5.50 Special Events Producers’ Market 13th April Join us to taste a variety of fresh local food and drinks. Meet the producers and get some excellent recipe ideas. Holker Garden Festival 30th May The event celebrates its 22nd anniversary with a great show of the very best of gardening, making it one of the most popular events in gardening. National Garden Day 28th August Holker once again opens its gardens in aid of the disadvantaged. For just a small donation 4 you can take a tour with our garden guide. Winter Market 8th November This is an event for all the family. Wander among a variety of shops selling gifts while enjoying a live music show and nice street entertainment. 1.How long does it probably take a tourist to drive to Holker from Manchester? A. 20 minutes. B. 25 minutes. C. 45 minutes. D. 90 minutes. 2.How much should a member of a tour group pay to visit Hall & Gardens? A. £12.00. B. £9.00. C. £8.00. D. £5.50. 3.Which event will you go to if you want to see a live music show? A. Producers’ Market. B. Holker Garden Festival. C. National Garden Day. D. Winter Market. Passage 4 2018 江苏,4 分 话题:艺术博物馆的参观须知 词数:193 5 1.How much may they pay if an 11-year-old girl and her working parents visit the museum? A. $12. B. $37. C. $50. D. $62. 2.The attraction of the Cloisters museum and gardens lies in the fact that . A. it opens all the year round B. its collections date from the Middle Ages C. it has a modern European-style garden D. it sells excellent European glass collections Passage 5 2018 天津,10 分 话题:阿德莱德大学的消防举措 词数:328 Fire Prevention Information The University of Adelaide employs a full-time staff of fire prevention professionals. They inspect all campus buildings and test and maintain all sprinkler (喷水灭火装置) systems,fire alarms, and fire extinguishers (灭火器). They also provide educational programs on fire safety in the residence halls. Whenever you move to a new area, you should locate the fire alarm pull stations and the two exits nearest your room. Fire Alarms The floors of all campus buildings are equipped with manual (手动的) fire alarm systems which include fire alarm pull stations and pipes. Most are also equipped with automatic fire alarm systems consisting of heat detectors, smoke detectors and sprinklers. For your safety, never tamper with (胡乱摆弄) these systems. False fire alarms are illegal and may lead to imprisonment. Fire Drills A fire drill will be conducted in your residence hall every semester. During a fire drill, please do the following: ● Take your room key and ID, close and lock the door to your room. ● Exit immediately from the nearest emergency exit; do not use a lift. ● Meet outside of your residence hall and wait for further instructions. Fire Extinguishers Fire extinguishers are located on each floor and in each apartment. Use a fire extinguisher only if you have been trained to do so. Irresponsible use of a fire extinguisher can create a dangerous situation for other residents and could result in damage to personal property. Misuse of a fire extinguisher will result in fines. Smoke Detectors A smoke detector is on the ceiling in your room. Some buildings also have heat detectors on the ceilings. Do the following to ensure the safe operation of your smoke detector: ● If your smoke detector is working properly, the red light should be on. If the red light is not blinking(闪动), contact residence hall staff immediately. ● Do not cover or block your smoke detector in any way. ● If a smoke detector sets off an alarm and there is no fire or smoke, inform your hall staff. 1.What is the main duty of the fire prevention professionals? 6 A. To provide parttime jobs for students. B. To lead the students to the nearest exits. C. To check and maintain fire prevention equipment. D. To train teachers to be fire prevention professionals. 2.What do the automatic fire alarm systems include? A. Pipes and smoke detectors. B. Smoke detectors and sprinklers. C. Fire alarm pull stations and pipes. D. Sprinklers and fire alarm pull stations. 3.In a fire drill, the students should . A. rush quickly to a lift B. gather at the nearest exit C. shut the door and leave at once D. wait for instructions in the hall 4.What do we know about the use of fire extinguishers? A. Using them wrongly results in punishment. B. Irresponsible use of them can damage them. C. Improper use of them can destroy the apartment. D. Using them without a trainer present is forbidden. 5.To ensure the safe operation of the smoke detector, one should . A. contact the hall staff regularly B. cover the things that burn easily C. start the smoke detector in a fire D. make certain the red light is working 2017 年 Passage 1 2017 全国卷Ⅰ,6 分 话题:太平洋科学中心 词数:222 Pacific Science Center Guide ◆Visit Pacific Science Center’s Store Don’t forget to stop by Pacific Science Center’s Store while you are here to pick up a wonderful science activity or souvenir to remember your visit. The store is located(位于) upstairs in Building 3 right next to the Laser Dome. ◆Hungry? Our exhibits will feed your mind, but what about your body? Our café offers a complete menu of lunch and snack options, in addition to seasonal specials. The caf é is located upstairs in Building 1 and is open daily until one hour before Pacific Science Center closes. ◆Rental Information Lockers are available to store any belongings during your visit. The lockers are located in Building 1 near the Information Desk and in Building 3. Pushchairs and wheelchairs are available to rent at the Information Desk and Denny Way entrance. ID required. ◆Support Pacific Science Center Since 1962, Pacific Science Center has been inspiring a passion(热情)for discovery and lifelong learning in science, math and technology. Today, Pacific Science Center serves more than 1.3 million people a year and brings inquiry-based science education to classrooms and community 7 events all over Washington State. It ’s an amazing accomplishment and one we cannot achieve without generous support from individuals, corporations, and other social organizations. Visit pacificsciencecenter.org to find various ways you can support Pacific Science Center. 1.Where can you buy a souvenir at Pacific Science Center? A. In Building 1. B. In Building 3. C. At the Laser Dome. D. At the Denny Way entrance. 2.What does Pacific Science Center do for schools? A. Train science teachers. B. Distribute science books. C. Inspire scientific research. D. Take science to the classroom. 3.What is the purpose of the last part of the text? A. To encourage donations. B. To advertise coming events. C. To introduce special exhibits. D. To tell about the Center’s history. Passage 2 2017 全国卷Ⅱ,6 分 话题:全球的艺术家表演戏剧 词数:290 In the coming months, we are bringing together artists from all over the globe, to enjoy speaking Shakespeare’s plays in their own language, in our Globe, within the architecture Shakespeare wrote for. Please come and join us. National Theatre of China Beijing | Chinese This great occasion( 盛 会 )will be the National Theatre of China ’ s first visit to the UK.The company’s productions show the new face of 21st century Chinese theatre. This production of Shakespeare’s Richard Ⅲ will be directed by the National’s Associate Director, Wang Xiaoying. Date & Time: Saturday 28 April, 2.30pm & Sunday 29 April, 1.30pm & 6.30pm Marjanishvili Theatre Tbilisi | Georgian One of the most famous theatres in Georgia,the Marjanishvili,founded in 1928, appears regularly at theatre festivals all over the world. This new production of As You Like It is helmed(指导) by the company’s Artistic Director Levan Tsuladze. Date & Time: Friday 18 May, 2.30pm & Saturday 19 May, 7.30pm Deafinitely Theatre London | British Sign Language(BSL) By translating the rich and humorous text of Love’s Labour’s Lost into the physical language of BSL, Deafinitely Theatre creates a new interpretation of Shakespeare’s comedy and aims to build a bridge between deaf and hearing worlds by perfor ming to both groups as one audience. Date & Time: Tuesday 22 May, 2.30pm & Wednesday 23 May,7.30pm Habima National Theatre Tel Aviv | Hebrew The Habima is the centre of Hebrew-language theatre worldwide. Founded in Moscow after the 1905 revolution, the company eventually settled in Tel Aviv in the late 1920s. Since 1958, they have been recognised as the national theatre of Israel. This production of Shakespeare ’ s The Merchant of Venice marks their first visit to the UK. Date & Time: Monday 28 May, 7.30pm & Tuesday 29 May, 7.30pm 8 1.Which play will be performed by the National Theatre of China? A. Richard Ⅲ. B. Love’s Labour’s Lost. C. As You Like It. D. The Merchant of Venice. 2.What is special about Deafinitely Theatre? A. It has two groups of actors. B. It is the leading theatre in London. C. It performs plays in BSL. D. It is good at producing comedies. 3.When can you see a play in Hebrew? A. On Saturday 28 April. B. On Sunday 29 April. C. On Tuesday 22 May. D. On Tuesday 29 May. Passage 3 2017 全国卷Ⅲ,6 分 话题:四种消防车之旅 词数:243 San Francisco Fire Engine Tours San Francisco Winery Tour Running: February 1st through April 30th This delicious tour goes through the city on its way to Treasure Island where we will stop at the famous Winery SF. Here you can enjoy 4 pours of some of the best wine San Francisco has to offer.(Included in ticket price) Departing from the Cannery: Tour times upon request Duration(时长): 2 hours Price:$90 Back to the Fifties Tour Running: August 16th through August 31st This tour transports you back in time to one of San Francisco’s most fantastic periods, the 1950s! Enjoy fun history as we take you through San Francisco for a free taste of ice cream. Departing from the Cannery: 5:00 pm and 7:30 pm Duration: 2 hours Price:$90 Spooky Halloween Tour Running: October 10th through October 31st  Join us for a ride through the historical Presidio district. Authentic fire gear(服装) is provided for your warmth as our entertainers take you to some of the most thrilling parts of San Francisco. Departing from the Cannery: 6:30 pm and 8:30 pm Duration: 1 hour and 30 minutes Price: Available upon request  Holiday Lights Tour Running: December 6th through December 23rd This attractive tour takes you to some of San Francisco’s most cheerful holiday scenes. Authentic fire gear is provided for your warmth as you get into the holiday spirit. Departing from the Cannery: 7:00 pm and 9:00 pm 9 Duration: 1 hour and 30 minutes Advance reservations required. 1.Which of the tours is available in March? A. San Francisco Winery Tour. B. Back to the Fifties Tour. C. Spooky Halloween Tour. D. Holiday Lights Tour.  2. What can tourists do on Back to the Fifties Tour? A. Go to Treasure Island. B. Enjoy the holiday scenes. C. Have free ice cream. D. Visit the Presidio district. 3.What are tourists required to do to go on Holiday Lights Tour? A. Take some drinks. B. Set off early in the morning. C. Wear warm clothes. D. Make reservations in advance. Passage 4 2017 北京,8 分 话题:介绍 TOKNOW 杂志 词数:239 Inspiring young minds! TOKNOW Magazine is a big hit in the world of children ’ s publishing, bringing a unique combination of challenging ideas and good fun to young fans every month. What is so special about TOKNOW Magazine? Well, it has no ads or promotions inside — instead it is jam-packed with serious ideas. TOKNOW makes complex ideas attractive and accessible to children, who can become involved in advanced concepts and even philosophy(哲学)— and they will soon discover that TOKNOW feels more like a club than just a magazine. What’s inside? Every month the magazine introduces a fresh new topic with articles, experiments and creative things to make — the magazine also explores philosophy and wellbeing to make sure young readers have a balanced take on life. Sounds too good to be true? Take a look online — evidence shows that thousands of teachers and parents know a good thing when they see it and recommend TOKNOW to their friends. Happy Birthday All Year! What could be more fun than a gift that keeps coming through the letterbox every month? The first magazine with your gift message will arrive in time for the special day. SUBSCRIBE NOW □Annual Subscription Europe £55 Rest of World £65 □Annual Subscription with Gift Pack Includes a Mammoth Map, a Passport Puzzle Booklet, and Subscription Europe £60 Rest of World £70 Refund Policy — the subscription can be cancelled within 28 days and you can get your money back. 10 1.Why is TOKNOW a special magazine? A. It entertains young parents. B. It provides serious advertisements. C. It publishes popular science fictions. D. It combines fun with complex concepts. 2.What does TOKNOW offer its readers? A. Online courses. B. Articles on new topics. C. Lectures on a balanced life. D. Reports on scientific discoveries. 3.How much should you pay if you make a 12-month subscription to TOKNOW with gift pack from China? A. £55. B. £60. C. £65. D. £70. 4.Subscribers of TOKNOW would get . A. free birthday presents B. full refund within 28 days C. membership of the TOKNOW club D. chances to meet the experts in person Passage 5 2017 天津,12.5 分 话题:发送了错误的电子邮件如何补救 词数:304 Suppose you’re in a rush, feeling tired, not paying attention to your screen, and you send an email that could get you in trouble. Realisation will probably set in seconds after you’ve clicked "send". You freeze in horror and burn with shame.  What to do? Here are four common email accidents, and how to recover. Clicking "send" too soon Don’t waste your time trying to find out if the receiver has read it yet. Write another email as swiftly as you can and send it with a brief title explaining that this is the correct version and the previous version should be ignored. Writing the wrong name The sooner you notice, the better. Respond quickly and briefly, apologising for your mistake. Keep the tone measured: don’t handle it too lightly, as people can be offended, especially if your error suggests a misunderstanding of their culture(i.e. incorrect ordering of Chinese names). Clicking "reply all" unintentionally You accidentally reveal(透露)to the entire company what menu choices you would prefer at the staff Christmas dinner, or what holiday you’d like to take. In this instance, the best solution is to send a quick, light-hearted apology to explain your awkwardness. But it can quickly rise to something worse, when everyone starts hitting "reply all" to join in a long and unpleasant conversation. In this instance, step away from your keyboard to allow everyone to calm down. Sending an offensive message to its subject The most awkward email mistake is usually committed in anger. You write an unkind message about someone, intending to send it to a friend, but accidentally send it to the person you’re 11 discussing. In that case, ask to speak in person as soon as possible and say sorry. Explain your frustrations calmly and sensibly — see it as an opportunity to clear up any difficulties you may have with this person. 1.After realising an email accident, you are likely to feel   . A. curious B. tired C. awful D. funny 2.If you have written the wrong name in an email, it is best to . A. apologise in a serious manner B. tell the receiver to ignore the error C. learn to write the name correctly D. send a short notice to everyone 3.What should you do when an unpleasant conversation is started by your "reply all"email? A. Try offering other choices. B. Avoid further involvement. C. Meet other staff members. D. Make a light-hearted apology. 4.How should you deal with the problem caused by an offensive email? A. By promising not to offend the receiver again. B. By seeking support from the receiver’s friends. C. By asking the receiver to control his anger. D. By talking to the receiver face to face. 5.What is the passage mainly about? A. Defining email errors. B. Reducing email mistakes. C. Handling email accidents. D. Improving email writing. Passage 6 2017 江苏,4 分 话题:历史方面的书 词数:190 CHRONOLOGICA —The Unbelievable Years that Defined History DID YOU KNOW... *In 105 AD paper was invented in China? *When Columbus discovered the New World? *The British Museum opened in 1759? CHRONOLOGICA is a fascinating journey through time, from the foundation of Rome to the creation of the internet. Along the way are tales of kings and queens, hot air balloons...and monkeys in space. 12 Travel through 100 of the most unbelievable years in world history and learn why being a Roman Emperor wasn’t always as good as it sounds, how the Hundred Years’ War didn’t actually last for 100 years and why Spencer Perceval holds a rather unfortunate record. CHRONOLOGICA is an informative and entertaining tour into history, beautifully illustrated and full of unbelievable facts. While CHRONOLOGICA tells the stories of famous people in history such as Thomas Edison and Alexander the Great, this book also gives an account of the lives of lesser-known individuals including the explorer Mungo Park and sculptor Gutzon Borglum. This complete but brief historical collection is certain to entertain readers young and old, and guaranteed to present even the biggest history lover with something new! 1.What is CHRONOLOGICA according to the text? A. A biography. B. A travel guide. C. A history book. D. A science fiction. 2.How does the writer recommend CHRONOLOGICA to readers? A. By giving details of its collection. B. By introducing some of its contents. C. By telling stories at the beginning. D. By comparing it with other books. 2012—2016 年 Passage 1 [2016·全国卷Ⅰ, A] You probably know who Marie Curie was, but you may not have heard of Rachel Carson. Of the outstanding ladies listed below, who do you think was the most important woman of the past 100 years? Jane Addams(1860—1935) Anyone who has ever been helped by a social worker has Jane Addams to thank. Addams helped 13 the poor and worked for peace. She encouraged a sense of community(社区) by creating shelters and promoting education and services for people in need. In 1931, Addams became the first American woman to win the Nobel Peace Prize. Rachel Carson(1907—1964) If it weren't for Rachel Carson, the environmental movement might not exist today. Her popular 1962 book Silent Spring raised awareness of the dangers of pollution and the harmful effects of chemicals on humans and on the world's lakes and oceans. Sandra Day O'Connor(1930—present) When Sandra Day O'Connor finished third in her class at Stanford Law School, in 1952, she could not find work at a law firm because she was a woman. She became an Arizona state senator(参议 员) and, in 1981, the first woman to join the U.S. Supreme Court. O'Connor gave the deciding vote in many important cases during her 24 years on the top court. Rosa Parks(1913—2005) On December 1, 1955, in Montgomery, Alabama, Rosa Parks would not give up her seat on a bus to a white passenger. Her simple act landed Parks in prison. But it also set off the Montgomery bus boycott. It lasted for more than a year, and kicked off the civilrights movement. "The only tired I was, was tired of giving in," said Parks. 21.What is Jane Addams noted for in history? A. Her social work. B. Her teaching skills. C. Her efforts to win a prize. D. Her community background. 22.What was the reason for O'Connor's being rejected by the law firm? A. Her lack of proper training in law. B. Her little work experience in court. C. The discrimination against women. D. The poor financial conditions. 23.Who made a great contribution to the civil-rights movement in the U.S.? A. Jane Addams. B. Rachel Carson. C. Sandra Day O'Connor. D. Rosa Parks. 24.What can we infer about the women mentioned in the text? A. They are highly educated. B. They are truly creative. C. They are pioneers. D. They are peace-lovers. Passage 2 [2016·全国卷Ⅱ,A] What's On? Electric Underground 7.30pm—1.00amFree at the Cyclops Theatre Do you know who's playing in your area? We're bringing you an exciting evening of live rock and pop music from the best local bands. Are you interested in becoming a musician and getting a recording contract(合同)? If so, come early to the talk at 7.30pm by Jules Skye, a successful record producer. He's going to talk about how you can find the right person to produce your music. Gee Whizz 8.30pm—10.30pmComedy at Kaleidoscope Come and see Gee Whizz perform. He's the funniest stand-up comedian on the comedy scene. This joyful show will please everyone, from the youngest to the oldest. Gee Whizz really knows how to make you laugh! Our bar is open from 7.00pm for drinks and snacks(快餐). Simon's Workshop 14 5.00pm—7.30pmWednesdays at Victoria Stage This is a good chance for anyone who wants to learn how to do comedy. The workshop looks at every kind of comedy, and practices many different ways of making people laugh. Simon is a comedian and actor who has 10 years' experience of teaching comedy. His workshops are exciting and fun. An evening with Simon will give you the confidence to be funny. Charlotte Stone 8.00pm—11.00pmPizza World Fine food with beautiful jazz music; this is a great evening out. Charlotte Stone will perform songs from her new best-selling CD, with James Pickering on the piano. The menu is Italian, with excellent meat and fresh fish, pizzas and pasta(面食). Book early to get a table. Our bar is open all day, and serves cocktails, coffee, beer, and white wine. 21.Who can help you if you want to have your music produced? A. Jules Skye. B. Gee Whizz. C. Charlotte Stone. D. James Pickering. 22.At which place can people of different ages enjoy a good laugh? A. The Cyclops Theatre. B. Kaleidoscope. C. Victoria Stage. D. Pizza World. 23.What do we know about Simon's Workshop? A. It requires membership status. B. It lasts three hours each time. C. It is run by a comedy club. D. It is held every Wednesday. 24.When will Charlotte Stone perform her songs? A. 5.00pm—7.30pm. B. 7.30pm—1.00am. C. 8.00pm—11.00pm. D. 8.30pm—10.30pm. Passage 3 [2016·全国卷Ⅲ,A] Music Opera at Music Hall: 1243 Elm Street. The season runs June through August, with additional performances in March and September. The Opera honors Enjoy the Arts membership discounts. Phone: 241-2742. http://www.cityopera.com. Chamber Orchestra: The Orchestra plays at Memorial Hall at 1406 Elm Street, which offers several concerts from March through June. Call 723-1182 for more information. http://www.chamberorch.com. Symphony Orchestra: At Music Hall and Riverbend. For ticket sales, call 381-3300. Regular season runs September through May at Music Hall and in summer at Riverbend. http://www.symphony.org/home.asp. College Conservatory of Music (CCM): Performances are on the main campus( 校 园 ) of the university, usually at Patricia Cobbett Theater. CCM organizes a variety of events, including performances by the well-known LaSalle Quartet, CCM's Philharmonic Orchestra, and various groups of musicians presenting Baroque through modern music. Students with I.D. cards can attend the events for free. A free schedule of events for each term is available by calling the box office at 556-4183. http://www.ccm.uc.edu/events/calendar. Riverbend Music Theater: 6295 Kellogg Ave. Large outdoor theater with the closest seats under cover (price difference). Big name shows all summer long! Phone: 232-6220. http://www.riverbendmusic.com. 1.Which number should you call if you want to see an opera? 15 A. 241-2742. B. 723-1182. C. 381-3300. D. 232-6220. 2.When can you go to a concert by Chamber Orchestra? A. February. B. May. C. August. D. November. 3.Where can students go for free performances with their I.D. cards? A. Music Hall. B. Memorial Hall. C. Patricia Cobbett Theater. D. Riverbend Music Theater. 4.How is Riverbend Music Theater different from the other places? A. It has seats in the open air. B. It gives shows all year round. C. It offers membership discounts. D. It presents famous musical works. Passage 4 [2016·全国卷Ⅲ,C] If you are a fruit grower — or would like to become one — take advantage of Apple Day to see what's around. It's called Apple Day but in practice it's more like Apple Month. The day itself is on October 21, but since it has caught on, events now spread out over most of October around Britain. Visiting an apple event is a good chance to see, and often taste, a wide variety of apples. To people who are used to the limited choice of apples such as Golden Delicious and Royal Gala in supermarkets, it can be quite an eye opener to see the range of classical apples still in existence, such as Decio which was grown by the Romans. Although it doesn't taste of anything special, it's still worth a try, as is the knobbly( 多 疙 瘩 的 ) Cat's Head which is more of a curiosity than anything else. There are also varieties developed to suit specific local conditions. One of the very best varieties for eating quality is Orleans Reinette, but you'll need a warm, sheltered place with perfect soil to grow it, so it's a pipe dream for most apple lovers who fall for it. At the events, you can meet expert growers and discuss which ones will best suit your conditions, and because these are family affairs, children are well catered for with apple-themed fun and games. Apple Days are being held at all sorts of places with an interest in fruit, including stately gardens and commercial orchards( 果 园 ). If you want to have a real orchard experience, try visiting the National Fruit Collection at Brogdale, near Faversham in Kent. 8.What can people do at the apple events? A. Attend experts' lectures. B. Visit fruit-loving families. C. Plant fruit trees in an orchard. D. Taste many kinds of apples. 9.What can we learn about Decio? A. It is a new variety. B. It has a strange look. C. It is rarely seen now. D. It has a special taste. 10.What does the underlined phrase "a pipe dream" in Paragraph 3 mean? A. A practical idea. B. A vain hope. C. A brilliant plan. D. A selfish desire. 11.What is the author's purpose in writing the text? A. To show how to grow apples. B. To introduce an apple festival. C. To help people select apples. D. To promote apple research. Passage 5 [2016·天津,A] A A Language Programme for Teenagers 16 Welcome to Teenagers Abroad! We invite you to join us on an amazing journey of language learning. Our Courses Regardless of your choice of course, you’ll develop your language ability both quickly and effectively. Our Standard Course guarantees a significant increase in your confidence in a foreign language, with focused teaching in all 4 skill areas----speaking, listening, reading and writing. Our Intensive Course builds on our Standard Course, with 10 additional lessons per week, guaranteeing the fastest possible language learning (see table below). Course Type Days Number of Lesson Course Timetable Standard Course Mon-Fri 20 lessons 9:00-12:30 20 lessons 9:00-12:30Intensive Course Mon-Fri 10 lessons 13:00-14:30 Evaluation Students are placed into classes according to their current language skills. The majority of them take on online language test before starting their programme. However, if this is not available, students sit the exam on the first Monday of their course. Learning materials are provided to students throughout their course, and there will never be more than 15 participant s in each class. Arrivals and Transfer Our programme offers the full package—students are take good care of from the start through to the very end. They are collected from the airport upon arrival and brought to their accommodation in comfort. We require the student’s full details at least 4 weeks in advance. Meals/Allergies(过敏)/Special Dietary Requirements Students are provided with breakfast, dinner and either a cooked or packed lunch(which consists of a sandwich, a drink and a dessert). Snacks outside of mealtimes may be purchased by the student individually. We ask that you let us know of any allergies or dietary requirements as well as information about any medicines you take. Depending on the type of allergies and/or dietary requirements, an extra 17 charge may be made for providing special food. 36. How does Intensive Course differ from Standard Course? A. It is less effective. B. It focuses on speaking. C. It includes extra lessons. D. It give you confidence 37. When can a student attend Standard Course? A. 13:00-14:30 Monday. B. 9:00-12:30 Tuesday C. 13:00-14:30 Friday. D. 9:00-12:30 Saturday. 38. Before starting their programme, students are expected to _____. A. take a language test B. have an online interview C. prepare learning materials D. report their language levels 39. With the full package, the programme organizer is supposed to_____. A. inform students of their full flight details B. look after students throughout the programme C. offer students free sightseeing trips D. collect students’ luggage in advance 40. Which of the following may require an extra payment? A. Cooked dinner. B. Mealtime dessert. C. Packed lunch. D. Special diet. Passage 6 [2016·浙江,B] B Below are search record from a university library’s database. 18 Quick Search | Category | Full Text | Advanced Search full text of books for children Displaying 1 to 100 of 639 titles for children where Category is Education Refine Your Search Remove all filters Year Published 2016( 9 ) 2015( 90 ) 2014( 290 ) 2013( 118 ) 2012(62 ) Show more... Language English(637 ) German(2) Category Education(639) Social Science(27) Medicine (15) Psychology(11) Language/Linguistics(10) Show more... 19 Build It ,Make It ,Play It ! Guides for Children and Teens Bomhold Catharine ;Elder Terri,2004 |ABC-CLIO Series: Children’s and Young Adult Literature Reference ●Available For busy librarians and educators ,finding instructions for projects, activities, sports, and games that children and teens will find interesting is a constant challenge, This guide is a time-saving,one-stop… Read this book | View details |Add to Collection Circle Time for Young Children Mosley Jenny,2014 |Taylor and Francis Series: Essential Guides for Early Years Practitioners ●Available Jenny Mosley’s quality circle time model involves setting up an on-going, timetabled process of circle-meeting for adults and children. As a basis for teaching relationship skills, building up self-esteem... Read this book |View details | Add to Collection Connecting Animals and Children in Early Childhood Selly Patty Born,2014 | Redleaf Press ●Available Understand the value of connecting animals and children. From familys pets and wild animals to toys, stuffed animals, and media images, animals are a central part of every child’s world. This book examines… Read this book |View details |Add to Collection 20 Education and Disadvantaged Children and Young People Matsumoto Mitsuko; Brock Colin,2013| Bloomsbury Publishiing Series: Education as a Humanitarian Response ● Available Do street children go to school, and if not, why not? What kind of education can be ‘meaningful’ to young people affected by conflict? The contributors explore groups of children and young people who have… Read this book |View details | Add to Collection Children with School Problems:A Physician’s Manual The Canadian Paediatric Society; Andrews Debra; Mahoney William J, 2012|Wiley ●Available The physician’s guide to diagnosing and treating learning disabilities in children. 1 to 10 Canadians have a learning disability, and doctors must be able to identify, diagnose, treat, and manage children… Read this book | View details | Add to Collection Songs in Their Heads:Music and Its Meaning in Children’s Lives Campbell Patricia Shehan,1998 |Oxford University Press ●Available This book explores the musical interest and needs of children in their daily lives. Based upon their expressed thoughts and actual “musicking” behaviors, this text examines the songs they sing, the ryhthms… Read this book| View details | Add to Collection 21 46. Supp ose you are doing research on children’s relationship skills, you may want to read . A.Circle Time for Young Children B. Children with School Problems: A Physician’s Manual C. Education and Disadvantaged Children and Young People D. Build It, Make It, Do It, Play It! Guides for Children and Teens 47.Which book would you recommend to someone interested in children’s mental images? A.Connecting Animals and Children in Early Childhood. B.Songs in Their Heads: Music and Its Meaning in Children’s Lives. C.Big Ideas for Little Kids: Teaching Philosophy Through Children’s Literature. D.Young Children as Artists: Art and Design in the Early Years and Key Stage 1. 48.How many books published in 2015 are found in this search? A. 9. B.90. C.118. D.290. 49. Children with School Problems: A Physician’s Manual is most likely intended for . Young Children as Artists: Art and Design in the Earty Years and Key Stage 1 Tutchell Suzy, 2014| Taylor Francis ●Available From the moment a child is born,they interact with the sensory world, looking at colours, feeling textures; constructing mental and physical images of what they see and experience.Within all early years… Read this book | view details| Add to Collection Big Ideas for Little Kids: Teaching Philosophy Through Children’s Literature Wartenberg Thomas E, 2014 |Rowman&Littlefield Publishers ●Available Big Ideas for Little Kids includes everything a teacher, a parent, or a college student needs to teach philosophy to elementary school children from picture books.Written in a clear and accessible style, … Read this book |View details| Add to Collection 22 A. educators B. librarians C. doctors D.artists Passage 7 [2016·四川,A] A Basketball Statistician Help Wanted The Athletic Department is looking for students to help assist staff during the Fall 2016, Winter 2016-17 and Spring 2017 semesters. Students in this position will be keeping live statistics during basketball games. Students must meet all of the following requirements: ●Good computer skills ●Available evenings and weekends ●Knowing basketball rules and statistics Students interested in working for the Athletic Department should contact the Athletic Coordinator at their respective(各自的) campuses. ●TP/SS Athletic Coordinator, Michael Simone,240-567-1308 ●Rockville Athletic Coordinator, Jorge Zuniga,240-567-7589 ●Springfield Athletic coordinator, Gary Miller,240-567-2273 ●Germantown Athletic Coordinator, GavriChavan, 240-567-6915 21.When will the job start? A .In May 2016 B. In May 2017 C.In September 2016 D. In September2017 22.Who is more likely to get job? A.Sam,English major ,member of the college basketball team B.Judy,IT staff with night classes,children’s basketball team coach C.Ted,computer major, basketball fan,free on evenings and weekends D.Molly,part-time programmer,high school basketball player ,new mother 23.Whom should you contact if you want to apply for the job in Rockville? A.Michael B.Jorge C.Gauri D.Gary Passage 8 [2015·新课标全国 II,D] Choose Your One-Day Tours! 23 Tour A — Bath & Stonehenge including entrance fees to the ancient Roman bathrooms and Stonehenge — £37 until 26 March and £39 thereafter. Visit the city with over 2,000 years of history and Bath Abbey, the Royal Crescent and the Costume Museum. Stonehenge is one of the world's most famous prehistoric monuments dating back over 5,000 years. Tour B — Oxford & Stratford including entrance fees to the University St Mary's Church Tower and Anne Hathaway's house — £32 until 12 March and £36 thereafter. Oxford: Includes a guided tour of England's oldest university city and colleges. Look over the "city of dreaming spires (尖顶)" from St Mary's Church Tower. Stratford: Includes a guided tour exploring much of the Shakespeare wonder. Tour C — Windsor Castle & Hampton Court including entrance fees to Hampton Court Palace — £34 until 11 March and £37 thereafter. Includes a guided tour of Windsor and Hampton Court, Henry VIII's favourite palace. Free time to visit Windsor Castle(entrance fees not included). With 500 years of history, Hampton Court was once the home of four Kings and one Queen. Now this former royal palace is open to the public as a major tourist attraction. Visit the palace and its various historic gardens, which include the famous maze(迷宫) where it is easy to get lost! Tour D — Cambridge including entrance fees to the Tower of Saint Mary the Great — £33 until 18 March and £37 thereafter. Includes a guided tour of Cambridge, the famous university town, and the gardens of the 18th century. 33.Which tour will you choose if you want to see England's oldest university city? 24 A. Tour A. B. Tour B. C. Tour C. D. Tour D. 34.Which of the following tours charges the lowest fee on 17 March? A. Windsor Castle & Hampton Court. B. Oxford & Stratford. C. Bath & Stonehenge. D. Cambridge. 35.Why is Hampton Court a major tourist attraction? A. It used to be the home of royal families. B. It used to be a well-known maze. C. It is the oldest palace in Britain. D. It is a world-famous castle. Passage 9 [2015·北京,B] B Revolutionary TV Ears TV Ears has helped thousands of people with various degrees of hearing loss hear the television clearly without turning up the volume( 音量) and now it’s better and more affordable than ever! With TV Ears wireless technology, you set your own headset volume, while other TV listeners hear the television at a volume level that’s comfortable for them. You can even listen through the headset only and put the TV on mute(静音) if the situation calls for a quiet environment —maybe the baby is sleeping. Or perhaps you are the only one who is interested in listening to the ballgame. TV Ears patented technology includes a revolutionary noise reduction car tip, not used in any other commercially available headset. This tip reduces outside noise so that television dialogue is clear and understandable. Get the technology that has proven to help the most demanding customers. That’s why TV Ears has earned the trust and confidence of audiologists( 听觉学家) nationwide as well as world-famous doctors. 25 Doctor Recommended TV Ears! “My wife and I have used TV Ears almost daily for the past two years and find them a great help in our enjoyment of television . As a retired ear doctor, I heartily recommend TV Ears to people with normal hearing as well as those with hearing loss.” — Robert Forbes, M. D, CA Customer Recommended TV Ears! “ Now my husband can have the volume as loud as he needs and I can have the TV at my hearing level. TV Ears is so comfortable that Jack forgets he has them on ! He can once again hear and understand the dialogue.” — Darlene & Jack B, CA Risk Free Trial! TV Ears comes with a 30-day risk free trial. Special Offer — Now $59.95. If you’re not satisfied, return it. Money-back guarantee! Call now ! 800-123-7832 60. TV Ears helps you ______. A. improve your sleeping quality B. listen to TV without disturbing others C. change TV channels without difficulty D. become interested in ballgame programs 61. What makes TV Ears different from other headsets? A. It can easily set TV on mute B. Its headset volume is adjustable C. It has a new noise reduction ear tip D. It applies special wireless technology 62.This advertisement is made more believable by ______. A. using recommendations B. offering reasons for this invention C. providing statistics D. showing the results of experiments Passage 10 [2015·天津,A] University Room Regulations Approved and Prohibited Items 26 The following items are approved for use in residential(住宿的)rooms: electric blankets, hair dryers, personal computers, radios, televisions and DVD players. Items that are not allowed in student rooms include: candles, ceiling fans, fireworks, waterbeds, sun lamps and wireless routers. Please note that any prohibited items will be taken away by the Office of Residence Life. Access to Residential Rooms Students are provided with a combination( 组合密码)for their room door locks upon check-in. Do not share your room door lock combination with anyone. The Office of Residence Life may change the door lock combination at any time at the expense of the resident if it is found that the student has shared the combination with others. The fee is $25 to change a room combination. Cooking Policy Students living in buildings that have kitchens are only permitted to cook in the kitchen. Students must clean up after cooking. This is not the responsibility of housekeeping staff. Kitchens that are not kept clean may be closed for use. With the exception of using a small microwave oven(微波炉) to heat food, students are not permitted to cook in their rooms. Pet Policy No pets except fish are permitted in student rooms. Students who are found with pets, whether visiting or owned by the student, are subject to an initial fine of $100 and a continuing fine of $50 a day per pet. Students receive written notice when the fine goes into effect. If, one week from the date of written notice, the pet is not removed, the student is referred to the Student Court. Quiet Hours Residential buildings must maintain an atmosphere that supports the academic mission of the University. Minimum quiet hours in all campus residences are 11:00 pm to 8:00 am Sunday through Thursday. Quiet hours on Friday and Saturday nights are 1:00 am to 8:00 am. Students who violate quiet hours are subject to a fine of $25. 36.Which of the following items are allowed in student rooms? A. Ceiling fans and waterbeds. B. Wireless routers and radios. C. Hair dryers and candles. D. TVs and electric blankets. 37.What if a student is found to have told his combination to others? A. The combination should be changed. B. The Office should be charged. 27 C. He should replace the door lock. D. He should check out of the room. 38.What do we know about the cooking policy? A. A microwave oven can be used. B. Cooking in student rooms is permitted. C. A housekeeper is to clean up the kitchen. D. Students are to close kitchen doors after cooking. 39.If a student has kept a cat in his room for a week since the warning, he will face . A. parent visits B. a fine of $100 C. the Student Court D. a written notice 40.When can students enjoy a party in residences? A. 7:00 am, Sunday. B. 7:30 am, Thursday. C. 11:30 pm, Monday. D. 00:30 am, Saturday. Passage 11 [2014·新课标全国 II,D] Metro Pocket Guide Metrorail (地铁) Each passenger needs a farecard to enter and go out. Up to two children under age five may travel free with a paying customer. Farecard machines are in every station. Bring small bills because there are no change machines in the stations and farecard machines only provide up to $5 in change. Get one day of unlimited Metrorail rides with a One Day Pass. Buy it from a farecard machine in Metro stations. Use it after 9:30 a.m. until closing on weekdays, and all day on weekends and holidays. Hours of service Open: 5 a.m. Mon.—Fri. 7 a.m. Sat.—Sun. Close: midnight Sun.—Thurs. 3 a.m. Fri.—Sat. nights Last train times vary. To avoid missing the last train, please check the last train times posted in stations. Metrobus 28 When paying with exact change, the fare is $1.35. When paying with a SmarTrip card, the fare is $1.25. Fares for senior/disabled customers Senior citizens 65 and older and disabled customers may ride for half the regular fare. On Metrorail and Metrobus, use a senior/disabled farecard or SmarTrip card. For more information about buying senior/disabled farecards, SmarTrip cards and passes, please visit MetroOpensDoors.com or call 202-637-7000 and 202-637-8000. Senior citizens and disabled customers can get free guide on how to use proper Metrobus and Metrorail services by calling 202-962-1100. Travel tips (提示)  ●Avoid riding during weekday rush periods — before 9:30 a.m. and between 4 and 6 p.m. ●If you lose something on a bus or train or in a station, please call Lost & Found at 202-962-1195. 32.What should you know about farecard machines? A. They start selling tickets at 9:30 a.m. B. They are connected to change machines. C. They offer special service to the elderly. D. They make change for no more than $5. 33.At what time does Metrorail stop service on Saturday? A. At midnight. B. At 3 a.m. C. At 5 a.m. D. At 7 p.m. 34.What is good about a SmarTrip card? A. It is convenient for old people. B. It saves money for its users. C. It can be bought at any time. D. It is sold on the Internet. 35.Which number should you call if you lose something on the Metro? A. 202-962-1195 B. 202-962-1100 C. 202-637-7000 D. 202-637-8000 Passage 12 [2014·安徽,A] A 29 Personal Robot Make your parents and teachers happy ! Are you having problems finishing your homework on time? Do you avoid tidying your room until your mom shouts at you? You don’t need to worry if you buy a Mr. Helping Hand personal robot.Mr. H can be programmed to organize your homework.Your own personal robot will follow you around, putting away books and objects that you have left on the floor or bed. Mr. H also has these features (特点) ·weighs only 500 grams ·includes long-lasting batteries ·comes with a 5-year guarantee ·remembers simple instructions Originally (最初) sold for $499 NOW ONLY $299 BUY NOW 56. With help from a Mr.H, you can . A. stop using batteries. B. finish your homework on time. C. remember your teacher’s instructions. D. get your room tidied on your way home. 57. A PENGO WATCH CONTROL can help you to . A. repair your TV B. organize your homework C. be a James Bond D. know what the weather is like 58. You can get your Mr. H for . A. $499 B. $299 C. $199 D. $99 59. Where would you be most likely to find the two texts? A. On a notice board B. In a company brochure. C. On a teenage website D. In a college newspaper. Passage 13 [2013·北京,A] EP Portable Heater We all know that the cost of heating our homes will continue to be a significant burden on the family budget. Now millions of people are saving on their heating bills with the EP portable heater. With over one million satisfied customers around the world, the new EP heats better and faster, WATCH CONTROL This is a watch that James Bond would be proud to wear! This is NOT a watch for ordinary people! Your electronic PENGO WATCH CONTROL  acts as a remote control for TVs and videos.  gives you a daily weather forecast.  reminds you when to hand in your homework.  sets off a silent warning alarm when parents or teachers are near. Besides, your PENGO WATCH CONTROL will always tell you the time accurately! Originally sold for $199 NOW ONLY $99 For further information, click here. 30 saves more on heating bills, and runs almost silent. The EP has no exposed heating parts that can cause a fire. The outside of the EP only gets warm to the touch so that it will not burn children or pets. The EP will not reduce oxygen in the room. With other heaters, you'll notice that you get sleepy when the heat comes on because they are burning up oxygen. The advanced EP also heats the room evenly, wall to wall and floor to ceiling. It comfortably covers an area up to 350 square feet. Other heaters heat rooms unevenly with most of the heat concentrated to the center of the room. And they only heat an area a few feet around the heater. With the EP, the temperature will not vary in any part of the room. The EP comes with a 3-year warranty (保修) and a 60-day, no questions asked, satisfaction guarantee. If you are not totally satisfied, return it at our expense and your money will be given back to you. Now we have a special offer for 10 days, during which you can enjoy a half price discount and a free delivery.If you order after that, we reserve the right to either accept or reject order requests at the discounted price.Take action right now! 56.What is mainly discussed in Paragraph 2? A. The heat of the EP. B. The safety of the EP. C. The appearance of the EP. D. The material of the EP. 57.From the passage,we can learn that the EP____. A. doesn't burn up oxygen B. runs without any noise C. makes people get sleepy D. is unsuitable for children and pets 58.The underlined word "evenly" in Paragraph 4 probably means ____. A. continuously B. separately C. quickly D. equally 59.The main purpose of the passage is to ____. A. persuade people to buy the product B. advise people to save on heating bills C. report the new development of portable heaters D. compare the functions of different heater brands Passage 14 [2012·新课标全国,A] 31 Are you looking for some new and exciting places to take your kids to? Try some of these places: Visit art museums. They offer a variety of activities to excite your kids' interest. Many offer workshops for making hand-made pieces, traveling exhibits, book signings( 签名) by children's favorite writers, and even musical performances and other arts.  Head to a natural history museum. This is where kids can discover the past from dinosaur models to rock collections and pictures of stars in the sky. Also, ask what kind of workshops and educational programs are prepared for kids and any special events that are coming up. Go to a Youtheater. Look for one in your area offering plays for child and family visitors. Pre-show play shops are conducted by area artists and educators where kids can discover the secret about performing arts. Puppet ( 木偶) making and stage make-up are just a couple of the special offerings you might find. Try hands-on science. Visit one of the many hands-on science museums around the country. These science play-lands are great fun for kids and grown-ups alike. They'll keep your child mentally and physically active the whole day through while pushing buttons, experimenting, and building. When everyone is tired, enjoy a fun family science show, commonly found in these museums. 56.If a child is interested in the universe, he probably will visit ______. A. a Youtheater B. an art museum C. a natural history museum D. a hands-on science museum 57.What can kids do at a Youtheater? A. Look at rock collections. B. See dinosaur models. C. Watch puppet making. D. Give performances. 58.What does "hands-on science" mean in the last paragraph? A. Science games designed by kids. B. Learning science by doing things. C. A show of kids' science work. D. Reading science books. 59.Where does this text probably come from? A. A science textbook. B. A tourist map. C. A museum guide. D. A news report. 32 专题二 科普研究类 2018 年 Passage 1 2018 北京,8 分 话题:无人驾驶汽车 词数:476 Preparing Cities for Robot Cars The possibility of self-driving robot cars has often seemed like a futurist’s dream, years away from materializing in the real world. Well, the future is apparently now. The California Department of Motor Vehicles began giving permits in April for companies to test truly self-driving cars on public roads. The state also cleared the way for companies to sell or rent out self-driving cars, and for companies to operate driverless taxi services. California, it should be noted, isn’t leading the way here. Companies have been testing their vehicles in cities across the country. It’s hard to predict when driverless cars will be everywhere on our roads. But however long it takes, the technology has the potential to change our transportation systems and our cities, for better or for worse, depending on how the transformation is regulated. While much of the debate so far has been focused on the safety of driverless cars ( and rightfully so ) , policymakers also should be talking about how self-driving vehicles can help reduce traffic jams, cut emissions ( 排 放 ) and offer more convenient, affordable mobility options. The arrival of driverless vehicles is a chance to make sure that those vehicles are environmentally friendly and more shared. Do we want to copy — or even worsen — the traffic of today with driverless cars? Imagine a future where most adults own individual self-driving vehicles. They tolerate long, slow journeys to and from work on packed highways because they can work, entertain themselves or sleep on the ride, which encourages urban spread. They take their driverless car to an appointment and set the empty vehicle to circle the building to avoid paying for parking. Instead of walking a few blocks to pick up a child or the dry cleaning, they send the self-driving minibus. The convenience even leads fewer people to take public transport — an unwelcome side effect researchers have already found in ride-hailing(叫车) services. A study from the University of California at Davis suggested that replacing petrol-powered private cars worldwide with electric, self-driving and shared systems could reduce carbon emissions from transportation 80% and cut the cost of transportation infrastructure(基础设施) and operations 40% by 2050. Fewer emissions and cheaper travel sound pretty appealing. The first commercially available driverless cars will almost certainly be fielded by ride-hailing services, considering the cost of self-driving technology as well as liability and maintenance issues(责任与 维 护 问 题 ). But driverless car ownership could increase as the prices drop and more people become comfortable with the technology. Policymakers should start thinking now about how to make sure the appearance of driverless vehicles doesn’t extend the worst aspects of the car-controlled transportation system we have today. The coming technological advancement presents a chance for cities and states to develop transportation systems designed to move more people , and more affordably. The car of the future is coming. We just have to plan for it. 1. According to the author, attention should be paid to how driverless cars can. A. help deal with transportation-related problems B. provide better services to customers C. cause damage to our environment D. make some people lose jobs 33 2. As for driverless cars, what is the author’s major concern? A. Safety. B. Side effects. C. Affordability. D. Management. 3. What does the underlined word "fielded" in Paragraph 4 probably mean? A. Employed. B. Replaced. C. Shared. D. Reduced. 4.What is the author’s attitude to the future of self-driving cars? A. Doubtful. B. Positive. C. Disapproving. D. Sympathetic. Passage 2 2018 天津,10 分 话题:3D 打印机打印书屋 词数:360 There’s a new frontier in 3D printing that’s beginning to come into focus: food.Recent development has made possible machines that print, cook, and serve foods on a mass scale. And the industry isn’t stopping there. Food production With a 3D printer, a cook can print complicated chocolate sculptures and beautiful pieces for decoration on a wedding cake. Not everybody can do that — it takes years of experience, but a printer makes it easy. A restaurant in Spain uses a Foodini to "re-create forms and pieces" of food that are "exactly the same," freeing cooks to complete other tasks. In another restaurant, all of the dishes and desserts it serves are 3D-printed, rather than farm to table. Sustainability (可持续性) The global population is expected to grow to 9.6 billion by 2050, and some analysts estimate that food production will need to be raised by 50 percent to maintain current levels. Sustainability is becoming a necessity. 3D food printing could probably contribute to the solution. Some experts believe printers could use hydrocolloids (水解胶体) from plentiful renewables like algae (藻类) and grass to replace the familiar ingredients ( 烹 饪 原 料 ).3D printing can reduce fuel use and emissions. Grocery stores of the future might stock "food" that lasts years on end, freeing up shelf space and reducing transportation and storage requirements. Nutrition Future 3D food printers could make processed food healthier. Hod Lipson,a professor at Columbia University, said, "Food printing could allow consumers to print food with customized nutritional content,like vitamins. So instead of eating a piece of yesterday’s bread from the supermarket, you’d eat something baked just for you on demand." Challenges Despite recent advancements in 3D food printing, the industry has many challenges to overcome. Currently, most ingredients must be changed to a paste(糊状物)before a printer can use them, and the printing process is quite time-consuming, because ingredients interact with each other in very complex ways. On top of that, most of the 3D food printers now are restricted to dry ingredients, because meat and milk products may easily go bad.Some experts are skeptical about 3D food printers, believing they are better suited for fast food restaurants than homes and high-end restaurants. 1. What benefit does 3D printing bring to food production? A. It helps cooks to create new dishes. B. It saves time and effort in cooking. 34 C. It improves the cooking conditions. D. It contributes to restaurant decorations. 2.What can we learn about 3D food printing from Paragraph 3? A. It solves food shortages easily. B. It quickens the transportation of food. C. It needs no space for the storage of food. D. It uses renewable materials as sources of food. 3.According to Paragraph 4, 3Dprinted food . A. is more available to consumers B. can meet individual nutritional needs C. is more tasty than food in supermarkets D. can keep all the nutrition in raw materials 4.What is the main factor that prevents 3D food printing from spreading widely? A. The printing process is complicated. B. 3D food printers are too expensive. C. Food materials have to be dry. D. Some experts doubt 3D food printing. 5.What could be the best title of the passage? A. 3D Food Printing: Delicious New Technology B. A New Way to Improve 3D Food Printing C. The Challenges for 3D Food Production D. 3D Food Printing: From Farm to Table 2017 年 Passage 1 2017 全国卷Ⅰ,8 分 话题:太阳能蒸馏器 词数:300 A build-it-yourself solar still(蒸馏器) is one of the best ways to obtain drinking water in areas where the liquid is not readily available. Developed by two doctors in the U.S.Department of Agriculture, it ’ s an excellent water collector. Unfortunately, you must carry the necessary equipment with you, since it’s all but impossible to find natural substitutes. The only components required, though, are a 5 ′×5 ′sheet of clear or slightly milky plastic, six feet of plastic tube, and a container —perhaps just a drinking cup — to catch the water. These pieces can be folded into a neat little pack and fastened on your belt. To construct a working still, use a sharp stick or rock to dig a hole four feet across and three feet deep. Try to make the hole in a damp area to increase the water catcher’s productivity. Place your cup in the deepest part of the hole. Then lay the tube in place so that one end rests all the way in the cup and the rest of the line runs up — and out — the side of the hole. Next, cover the hole with the plastic sheet, securing the edges of the plastic with dirt and weighting the sheet’s center down with a rock. The plastic should now form a cone(圆锥体) with 45-degree-angled sides. The low point of the sheet must be centered directly over, and no more than three inches above, the cup. The solar still works by creating a greenhouse under the plastic. Ground water evaporates(蒸发) and collects on the sheet until small drops of water form, run down the material, and fall off into the cup. When the container is full, you can suck the refreshment out through the tube, and won’t have to break down the still every time you need a drink. 35 1.What do we know about the solar still equipment from the first paragraph? A. It’s delicate. B. It’s expensive. C. It’s complex. D. It’s portable. 2.What does the underlined phrase "the water catcher" in paragraph 2 refer to? A. The tube. B. The still. C. The hole. D. The cup. 3.What is the last step of constructing a working solar still? A. Dig a hole of a certain size. B. Put the cup in place. C. Weight the sheet’s center down. D. Cover the hole with the plastic sheet. 4.When a solar still works, drops of water come into the cup from . A. the plastic tube B. outside the hole C. the open air D. beneath the sheet Passage 2 2017 全国卷Ⅱ,8 分 话题:植物如何进行自我保护 词数:289 When a leafy plant is under attack, it doesn ’ t sit quietly. Back in 1983, two scientists, Jack Schultz and Ian Baldwin, reported that young maple trees getting bitten by insects send out a particular smell that neighboring plants can get. These chemicals come from the injured parts of the plant and seem to be an alarm. What the plants pump through the air is a mixture of chemicals known as volatile organic compounds, VOCs for short. Scientists have found that all kinds of plants give out VOCs when being attacked. It’s a plant’s way of crying out. But is anyone listening? Apparently. Because we can watch the neighbors react. Some plants pump out smelly chemicals to keep insects away. But others do double duty. They pump out perfumes designed to attract different insects who are natural enemies to the attackers. Once they arrive, the tables are turned. The attacker who was lunching now becomes lunch. In study after study, it appears that these chemical conversations help the neighbors. The damage is usually more serious on the first plant, but the neighbors, relatively speaking, stay safer because they heard the alarm and knew what to do. Does this mean that plants talk to each other? Scientists don ’t know. Maybe the first plant just made a cry of pain or was sending a message to its own branches, and so, in effect, was talking to itself. Perhaps the neighbors just happened to  "overhear  " the cry. So information was exchanged, but it wasn’t a true, intentional back and forth. Charles Darwin, over 150 years ago, imagined a world far busier, noisier and more intimate(亲密 的) than the world we can see and hear. Our senses are weak. There’s a whole lot going on. 1.What does a plant do when it is under attack? A. It makes noises. B. It gets help from other plants. C. It stands quietly. D. It sends out certain chemicals. 2.What does the author mean by "the tables are turned" in paragraph 3? A. The attackers get attacked. B. The insects gather under the table. C. The plants get ready to fight back. 36 D. The perfumes attract natural enemies. 3.Scientists find from their studies that plants can . A. predict natural disasters B. protect themselves against insects C. talk to one another intentionally D. help their neighbors when necessary 4.What can we infer from the last paragraph? A. The world is changing faster than ever. B. People have stronger senses than before. C. The world is more complex than it seems. D. People in Darwin’s time were more imaginative. Passage 3 2017 全国卷Ⅲ,8 分 话题:老年人安全驾车研究 词数:314 The Intelligent Transport team at Newcastle University have turned an electric car into a mobile laboratory named "DriveLAB"in order to understand the challenges faced by older drivers and to discover where the key stress points are. Research shows that giving up driving is one of the key reasons for a fall in health and well-being among older people, leading to them becoming more isolated(隔绝) and inactive. Led by Professor Phil Blythe, the Newcastle team are developing in-vehicle technologies for older drivers which they hope could help them to continue driving into later life. These include custom-made navigation( 导航) tools, night vision systems and intelligent speed adaptations. Phil Blythe explains: "For many older people, particularly those living alone or in the country, driving is important for preserving their independence, giving them the freedom to get out and about without having to rely on others. "But we all have to accept that as we get older our reactions slow down and this often results in people avoiding any potentially challenging driving conditions and losing confidence in their driving skills. The result is that people stop driving before they really need to." Dr Amy Guo, the leading researcher on the older driver study, explains: "The DriveLAB is helping us to understand what the key stress points and difficulties are for older drivers and how we might use technology to address these problems." "For example, most of us would expect older drivers always go slower than everyone else but surprisingly, we found that in 30mph zones they struggled to keep at a constant speed and so were more likely to break the speed limit and be at risk of getting fined. We’re looking at the benefits of systems which control their speed as a way of preventing that. "We hope that our work will help with technological solutions( 解决方案) to ensure that older drivers stay safer behind the wheel." 1.What is the purpose of the DriveLAB? A. To explore new means of transport. B. To design new types of cars. C. To find out older drivers’ problems. D. To teach people traffic rules. 2.Why is driving important for older people according to Phil Blythe? A. It keeps them independent. B. It helps them save time. C. It builds up their strength. 37 D. It cures their mental illnesses 3.What do researchers hope to do for older drivers? A. Improve their driving skills. B. Develop driver-assist technologies. C. Provide tips on repairing their cars. D. Organize regular physical checkups. 4.What is the best title for the text? A. A New Model Electric Car  B. A Solution to Traffic Problems  C. Driving Services for Elders D. Keeping Older Drivers on the Road Passage 4 2017 天津,12.5 分 话题:自主驾驶车辆的交通法规 词数:334 This month, Germany’s transport minister, Alexander Dobrindt, proposed the first set of rules for autonomous vehicles ( 自主驾驶车辆). They would define the driver ’s role in such cars and govern how such cars perform in crashes where lives might be lost. The proposal attempts to deal with what some call the "death valley" of autonomous vehicles: the grey area between semi-autonomous and fully driverless cars that could delay the driverless future. Dobrindt wants three things: that a car always chooses property ( 财产) damage over personal injury; that it never distinguishes between humans based on age or race; and that if a human removes his or her hands from the driving wheel — to check email, say — the car ’s maker is responsible if there is a crash. "The change to the road traffic law will permit fully automatic driving," says Dobrindt. It will put fully driverless cars on an equal legal footing to human drivers, he says. Who is responsible for the operation of such vehicles is not clear among car makers, consumers and lawyers. "The liability (法律责任) issue is the biggest one of them all," says Natasha Merat at the University of Leeds, UK. An assumption behind UK insurance for driverless cars, introduced earlier this year, insists that a human "be watchful and monitoring the road" at every moment. But that is not what many people have in mind when thinking of driverless cars. "When you say ‘driverless cars’, people expect driverless cars," Merat says. "You know — no driver." Because of the confusion, Merat thinks some car makers will wait until vehicles can be fully automated without human operation. Driverless cars may end up being a form of public transport rather than vehicles you own, says Ryan Calo at Stanford University, California. That is happening in the UK and Singapore, where government-provided driverless vehicles are being launched. That would go down poorly in the US, however."The idea that the government would take over driverless cars and treat them as a public good would get absolutely nowhere here," says Calo. 1. What does the phrase "death valley" in Paragraph 2 refer to? A. A place where cars often break down. B. A case where passing a law is impossible. C. An area where no driving is permitted. D. A situation where drivers’ role is not clear. 2.The proposal put forward by Dobrindt aims to . 38 A. stop people from breaking traffic rules B. help promote fully automatic driving C. protect drivers of all ages and races D. prevent serious property damage 3.What do consumers think of the operation of driverless cars? A. It should get the attention of insurance companies. B. It should be the main concern of law makers. C. It should not cause deadly traffic accidents. D. It should involve no human responsibility. 4.Driverless vehicles in public transport see no bright future in . A. Singapore B. the UK C. the US D. Germany 5.What could be the best title for the passage? A. Autonomous Driving: Whose Liability? B. Fully Automatic Cars: A New Breakthrough C. Autonomous Vehicles: Driver Removed! D. Driverless Cars: Root of Road Accidents Passage 5 2017 北京,8 分 话题:人工智能 词数:446 Hollywood’s theory that machines with evil(邪恶的)minds will drive armies of killer robots is just silly. The real problem relates to the possibility that artificial intelligence (AI) may become extremely good at achieving something other than what we really want. In 1960 a well-known mathematician Norbert Wiener, who founded the field of cybernetics (控制论), put it this way: "If we use, to achieve our purposes, a mechanical agency with whose operation we cannot effectively interfere (干预), we had better be quite sure that the purpose put into the machine is the purpose which we really desire." A machine with a specific purpose has another quality, one that we usually associate with living things: a wish to preserve its own existence. For the machine, this quality is not in-born, nor is it something introduced by humans; it is a logical consequence of the simple fact that the machine cannot achieve its original purpose if it is dead. So if we send out a robot with the single instruction of fetching coffee, it will have a strong desire to secure success by disabling its own off switch or even killing anyone who might interfere with its task. If we are not careful, then, we could face a kind of global chess match against very determined, super intelligent machines whose objectives conflict with our own, with the real world as the chessboard. The possibility of entering into and losing such a match should concentrate the minds of computer scientists. Some researchers argue that we can seal the machines inside a kind of firewall, using them to answer difficult questions but never allowing them to affect the real world. Unfortunately, that plan seems unlikely to work: we have yet to invent a firewall that is secure against ordinary humans, let alone super intelligent machines. Solving the safety problem well enough to move forward in AI seems to be possible but not easy. There are probably decades in which to plan for the arrival of super intelligent machines. But the problem should not be dismissed out of hand, as it has been by some AI researchers. Some argue that humans and machines can coexist as long as they work in  teams — yet that is not 39 possible unless machines share the goals of humans. Others say we can just "switch them off" as if super intelligent machines are too stupid to think of that possibility. Still others think that super intelligent AI will never happen. On September 11, 〖 JP2 〗 1933, famous physicist Ernest Rutherford stated, with confidence, "Anyone who expects a source of power in the transformation of these atoms is talking moonshine." However, on September 12, 1933, physicist Leo Szilard invented the neutron-induced (中子诱导)nuclear chain reaction. 1.Paragraph 1 mainly tells us that artificial intelligence may . A. run out of human control B. satisfy human’s real desires C. command armies of killer robots D. work faster than a mathematician 2.Machines with specific purposes are associated with living things partly because they might be able to . A. prevent themselves from being destroyed  B. achieve their original goals independently C. do anything successfully with given orders D. beat humans in international chess matches 3.According to some researchers, we can use firewalls to . A. help super intelligent machines work better B. be secure against evil human beings C. keep machines from being harmed D. avoid robots’affecting the world 4.What does the author think of the safety problem of super intelligent machines? A. It will disappear with the development of AI. B. It will get worse with human interference. C. It will be solved but with difficulty. D. It will stay for a decade. 2012—2016 年 Passage 1 2016 全国卷Ⅲ,8 分词数:349 Bad news sells. If it bleeds, it leads. No news is good news, and good news is no news. Those are the classic rules for the evening broadcasts and the morning papers. But now that information is being spread and monitored(监控) in different ways, researchers are discovering new rules. By tracking people's e-mails and online posts, scientists have found that good news can spread faster and farther than disasters and sob stories. "The ‘if it bleeds’ rule works for mass media," says Jonah Berger, a scholar at the University of Pennsylvania. "They want your eyeballs and don't care how you're feeling. But when you share a story with your friends, you care a lot more how they react. You don't want them to think of you as a Debbie Downer." Researchers analyzing word-of-mouth communication — e-mails, Web posts and reviews, face-to-face conversations — found that it tended to be more positive than negative( 消极的) , but that didn't necessarily mean people preferred positive news. Was positive news shared more often simply because people experienced more good things than bad things? To test for that 40 possibility, Dr. Berger looked at how people spread a particular set of news stories: thousands of articles on The New York Times'website.He and a Penn colleague analyzed the "most e-mailed" list for six months. One of his first findings was that articles in the science section were much more likely to make the list than non-science articles. He found that science amazed Times'readers and made them want to share this positive feeling with others. Readers also tended to share articles that were exciting or funny, or that inspired negative feelings like anger or anxiety, but not articles that left them merely sad. They needed to be aroused(激发) one way or the other, and they preferred good news to bad. The more positive an article, the more likely it was to be shared, as Dr. Berger explains in his new book, "Contagious:Why Things Catch On." 1.What do the classic rules mentioned in the text apply to? A. News reports. B. Research papers. C. Private e-mails. D. Daily conversations. 2.What can we infer about people like Debbie Downer? A. They're socially inactive. B. They're good at telling stories. C. They're inconsiderate of others. D. They're careful with their words. 3.Which tended to be the most e-mailed according to Dr. Berger's research? A. Sports news. B. Science articles. C. Personal accounts. D. Financial reviews. 4.What can be a suitable title for the text? A. Sad Stories Travel Far and Wide B. Online News Attracts More People C. Reading Habits Change with the Times D. Good News Beats Bad on Social Networks Passage 2 2016 天津,12.5 分词数:372 When John was growing up, other kids felt sorry for him. His parents always had him weeding the garden, carrying out the garbage and delivering newspapers. But when John reached adulthood, he was better off than his childhood playmates. He had more job satisfaction, a better marriage and was healthier. Most of all, he was happier. Far happier. These are the findings of a 40-year study that followed the lives of 456 teenage boys from Boston. The study showed that those who had worked as boys enjoyed happier and more productive lives than those who had not. "Boys who worked in the home or community gained competence( 能 力 ) and came to feel they were worthwhile members of society," said George Vaillant, the psychologist(心理学家) who made the discovery. "And because they felt good about themselves, others felt good about them." Vaillant's study followed these males in great detail. Interviews were repeated at ages 25,31 and 47. Under Vaillant, the researchers compared the men's mental-health scores with their boyhood-activity scores. Points were awarded for part-time jobs, housework, effort in school, and ability to deal with problems. The link between what the men had done as boys and how they turned out as adults was surprisingly sharp. Those who had done the most boyhood activities were twice as likely to have warm relations with a wide variety of people, five times as likely to be well paid and 16 times less likely to have been unemployed. The researchers also found that IQ and family social and economic class made no real difference in how the boys turned out. Working — at any age — is important. Childhood activities help a child develop responsibility, independence, confidence and competence — the underpinnings( 基 础 ) of 41 emotional health. They also help him understand that people must cooperate and work toward common goals. The most competent adults are those who know how to do this. Yet work isn't everything. As Tolstoy once said, "One can live magnificently in this world if one knows how to work and how to love, to work for the person one loves and to love one's work." 1.What do we know about John? A. He enjoyed his career and marriage. B. He had few childhood playmates. C. He received little love from his family. D. He was envied by others in his childhood. 2.Vaillant's words in Paragraph 2 serve as____. A. a description of personal values and social values B. an analysis of how work was related to competence C. an example for parents'expectations of their children D. an explanation why some boys grew into happy men 3.Vaillant's team obtained their findings by____. A. recording the boys'effort in school B. evaluating the men's mental health C. comparing different sets of scores D. measuring the men's problem solving ability 4.What does the underlined word "sharp" probably mean in Paragraph 4? A. Quick to react. B. Having a thin edge. C. Clear and definite. D. Sudden and rapid. 5.What can be inferred from the last paragraph? A. Competent adults know more about love than work. B. Emotional health is essential to a wonderful adult life. C. Love brings more joy to people than work does. D. Independence is the key to one's success. Passage 3 2016 浙江,10 分词数:449 A scientist working at her lab bench and a six-month-old baby playing with his food might seem to have little in common.After all, the scientist is engaged in serious research to uncover the very nature of the physical world,and the baby is, well, just playing...right? Perhaps, but some developmental psychologists ( 心 理 学 家 )have argued that this "play" is more like a scientific investigation than one might think. Take a closer look at the baby playing at the table.Each time the bowl of rice is pushed over the table edge,it falls to the ground — and, in the process, it brings out important evidence about how physical objects interact(相互作用):bowls of rice do not float in mid-air, but require support to remain stable. It is likely that babies are not born knowing this basic fact of the universe; nor are they ever clearly taught it. Instead, babies may form an understanding of object support through repeated experiments and then build on this knowledge to learn even more about how objects interact. Though their ranges and tools differ, the baby's investigation and the scientist's experiment appear to share the same aim (to learn about the natural world), overall approach (gathering direct evidence from the world), and logic (are my observations what I expected?). Some psychologists suggest that young children learn about more than just the physical world in this way — that they investigate human psychology and the rules of language using similar 42 means. For example, it may only be through repeated experiments, evidence gathering,and finally overturning a theory, that a baby will come to accept the idea that other people can have different views and desires from what he or she has, for example, unlike the child, Mommy actually doesn't like Dove chocolate. Viewing childhood development as a scientific investigation throws light on how children learn, but it also offers an inspiring look at science and scientists. Why do young children and scientists seem to be so much alike? Psychologists have suggested that science as an effort — the desire to explore, explain, and understand our world — is simply something that comes from our babyhood. Perhaps evolution( 进化) provided human babies with curiosity and a natural drive to explain their worlds, and adult scientists simply make use of the same drive that served them as children. The same cognitive(认知的) systems that make young children feel good about figuring something out may have been adopted by adult scientists. As some psychologists put it, "It is not that children are little scientists but that scientists are big children." 1.According to some developmental psychologists,____. A. a baby's play is nothing more than a game B. scientific research into babies'games is possible C. the nature of babies'play has been thoroughly investigated D. a baby's play is somehow similar to a scientist's experiment 2.We learn from Paragraph 2 that____. A. scientists and babies seem to observe the world differently B. scientists and babies often interact with each other C. babies are born with the knowledge of object support D. babies seem to collect evidence just as scientists do 3.Children may learn the rules of language by____. A. exploring the physical world B. investigating human psychology C. repeating their own experiments D. observing their parents'behaviors 4.What is the main idea of the last paragraph? A. The world may be more clearly explained through children's play. B. Studying babies'play may lead to a better understanding of science. C. Children may have greater ability to figure out things than scientists. D. One's drive for scientific research may become stronger as he grows. 5.What is the author's tone when he discusses the connection between scientists'research and babies'play? A. Convincing. B. Confused. C. Confident. D. Cautious. Passage 4 2016 四川,8 分词数:259 A warm drink of milk before bed has long been the best choice for those wanting a good night's sleep. But now a study has found it really does help people nod off — if it is milked from a cow at night. Researchers have discovered that "night milk" contains more melatonin (褪黑激素),which has been proven to help people feel sleepy and reduce anxiety. The study, by researchers from Seoul, South Korea, involved mice being fed with dried milk 43 powder made from cows milked both during the day and at night. Those given night milk, which contained 10 times the amount of melatonin, were less active and less anxious than those fed with the milk collected during daytime, according to the study published in The Journal of Medicinal Food. Night milk quickened the start of sleep and caused the mice to sleep longer. While the effect of cows milk harvested at different time has not been tested on humans up to now, taking melatonin drugs has been suggested to those who are struggling to fall asleep at night. Previous studies have also indicated that milk can be excellent for helping sleep because of the calcium content, which helps people to relax. Milk is also sugar-free and additive-free with nutritionists recommending skimmed milk as the best choice before bed as it is the least fattening. The more fat you take in before bedtime, the greater burden you will put on your body at night. 1.According to the text, the mice fed with daytime milk ____. A. started sleep more easily B. were more anxious C. were less active D. woke up later 2.Which of the following is true of melatonin according to the text? A. It's been tested on mice for ten times. B. It can make people more energetic. C. It exists in milk in great amount. D. It's used in sleeping drugs. 3.What can be a suitable title for the text? A. Night Milk and Sleep B. Fat, Sugar and Health C. An Experiment on Mice D. Milk Drinking and Health 4.How does the author support the theme of the text? A. By giving examples. B. By stating arguments. C. By explaining statistical data. D. By providing research results. Passage 5 2016 江苏,6 分词数:396 Chimps (黑猩猩) will cooperate in certain ways, like gathering in war parties to protect their territory. But beyond the minimum requirements as social beings, they have little instinct (本能) to help one another. Chimps in the wild seek food for themselves. Even chimp mothers regularly decline to share food with their children, who are able from a young age to gather their own food. In the laboratory, chimps don't naturally share food either. If a chimp is put in a cage where he can pull in one plate of food for himself or, with no greater effort, a plate that also provides food for a neighbor in the next cage, he will pull at random — he just doesn't care whether his neighbor gets fed or not. Chimps are truly selfish. Human children, on the other hand, are naturally cooperative. From the earliest ages, they desire to help others, to share information and to participate in achieving common goals. The psychologist Michael Tomasello has studied this cooperativeness in a series of experiments with 44 very young children. He finds that if babies aged 18 months see an unrelated adult with hands full trying to open a door, almost all will immediately try to help. There are several reasons to believe that the urges to help, inform and share are not taught, but naturally possessed in young children. One is that these instincts appear at a very young age before most parents have started to train their children to behave socially. Another is that the helping behaviors are not improved if the children are rewarded. A third reason is that social intelligence develops in children before their general cognitive ( 认 知 的 ) skills, at least when compared with chimps. In tests conducted by Tomasello, the human children did no better than the chimps on the physical world tests but were considerably better at understanding the social world. The core of what children's minds have and chimps'don't is what Tomasello calls shared intentionality. Part of this ability is that they can infer what others know or are thinking. But beyond that, even very young children want to be part of a shared purpose. They actively seek to be part of a "we", a group that intends to work toward a shared goal. 1.What can we learn from the experiment with chimps? A. Chimps seldom care about others'interests. B. Chimps tend to provide food for their children. C. Chimps like to take in their neighbors'food. D. Chimps naturally share food with each other. 2.Michael Tomasello's tests on young children indicate that they ____. A. have the instinct to help others B. know how to offer help to adults C. know the world better than chimps D. trust adults with their hands full 3.The passage is mainly about ____. A. the helping behaviors of young children B. ways to train children's shared intentionality C. cooperation as a distinctive human nature D. the development of intelligence in children Passage 6 2015 新课标全国Ⅰ,8 分词数:361 Conflict is on the menu tonight at the cafe La Chope. This evening, as on every Thursday night, psychologist Maud Lehanne is leading two of France's favorite pastimes, coffee drinking and the "talking cure". Here they are learning to get in touch with their true feelings. It isn't always easy. The customers — some thirty Parisians who pay just under $2 (plus drinks)per session — are quick to intellectualize (高谈阔论), slow to open up and connect. "You are forbidden to say ‘one feels,’ or ‘people think’," Lehanne told them. "Say ‘I think,’ ‘Think me’." A cafe society where no intellectualizing is allowed? It couldn't seem more un-French. But Lehanne's psychology cafe is about more than knowing oneself: It's trying to help the city's troubled neighborhood cafes. Over the years, Parisian cafes have fallen victim to changes in the French lifestyle — longer working hours, a fast-food boom and a younger generation's desire to spend more time at home. Dozens of new theme cafes appear to change the situation. Cafes focused around psychology, history, and engineering are catching on, filling tables well into the evening. The city's "psychology cafes", which offer great comfort, are among the most popular places. 45 Middle-aged homemakers, retirees, and the unemployed come to such cafes to talk about love, anger, and dreams with a psychologist. And they come to Lehanne's group just to learn to say what they feel. "There's a strong need in Paris for communication," says Maurice Frisch, a cafe La Chope regular who works as a religious instructor in a nearby church. "People have few real friends. And they need to open up." Lehanne says she'd like to see psychology cafes all over France. "If people had normal lives, these cafes wouldn't exist," she says. "If life weren't a battle, people wouldn't need a special place just to speak." But then, it wouldn't be France. 1.What are people encouraged to do at the cafe La Chope? A. Learn a new subject. B. Keep in touch with friends. C. Show off their knowledge. D. Express their true feelings. 2.How are cafes affected by French lifestyle changes? A. They are less frequently visited. B. They stay open for longer hours. C. They have bigger night crowds. D. They start to serve fast food. 3.What are theme cafes expected to do? A. Create more jobs. B. Supply better drinks. C. Save the cafe business. D. Serve the neighborhood. 4.Why are psychology cafes becoming popular in Paris? A. They bring people true friendship. B. They give people spiritual support. C. They help people realize their dreams. D. They offer a platform for business links. Passage 7 2015 新课标全国Ⅱ,8 分词数:331 Your house may have an effect on your figure. Experts say the way you design your home could play a role in whether you pack on the pounds or keep them off. You can make your environment work for you instead of against you. Here are some ways to turn your home into part of your diet plan. Open the curtains and turn up the lights. Dark environments are more likely to encourage overeating, for people are often less self-conscious (难为情) when they're in poorly lit places — and so more likely to eat lots of food. If your home doesn't have enough window light, get more lamps and flood the place with brightness. Mind the colors. Research suggests warm colors fuel our appetites. In one study, people who ate meals in a blue room consumed 33 percent less than those in a yellow or red room. Warm colors like yellow make food appear more appetizing, while cold colors make us feel less hungry. So when it's time to repaint, go blue. Don't forget the clock — or the radio. People who eat slowly tend to consume about 70 fewer calories ( 卡路里) per meal than those who rush through their meals. Begin keeping track of the time, and try to make dinner last at least 30 minutes. And while you're at it, actually sit down to eat. If you need some help slowing down, turn on relaxing music. It makes you less likely to rush through a meal. Downsize the dishes. Big serving bowls and plates can easily make us fat. We eat about 22 percent more when using a 12-inch plate instead of a 10-inch plate. When we choose a large spoon over a smaller one, total intake (摄入) jumps by 14 percent. And we'll pour about 30 percent more liquid into a short, wide glass than a tall, skinny glass. 1.The text is especially helpful for those who care about ____. A. their home comforts B. their body shape C. house buying D. healthy diets 46 2.A home environment in blue can help people ____. A. digest food better B. reduce food intake C. burn more calories D. regain their appetites 3.What are people advised to do at mealtimes? A. Eat quickly. B. Play fast music. C. Use smaller spoons. D. Turn down the lights. 4.What can be a suitable title for the text? A. Is Your House Making You Fat? B. Ways of Serving Dinner C. Effects of Self-Consciousness D. Is Your Home Environment Relaxing? Passage 8 2015 重庆,8 分词数:288 In ancient Egypt, a shopkeeper discovered that he could attract customers to his shop simply by making changes to its environment. Modern businesses have been following his lead, with more tactics (策略). One tactic involves where to display the goods. For example, stores place fruits and vegetables in the first section. They know that customers who buy the healthy food first will feel happy so that they will buy more junk food (垃圾食品) later in their trip. In department stores, the women's shoe section is generally next to the women's cosmetics (化妆品) section: while the shop assistant is going back to find the right size shoe, bored customers are likely to wander over and find some cosmetics they might want to try later. Besides, businesses seek to appeal to customers'senses. Stores notice that the smell of baked goods encourages shopping, so they make their own bread each morning and then fan the bread smell into the store throughout the day. Music sells goods, too. Researchers in Britain found that when French music was played, sales of French wines went up. When it comes to the selling of houses, businesses also use highly rewarding tactics. They find that customers make decisions in the first few seconds upon walking in the door, and turn it into a business opportunity. A California builder designed the structure of its houses smartly. When entering the house, the customer would see the Pacific Ocean through the windows, and then the pool through an open stairway leading to the lower level. The instant view of water on both levels helped sell these $10 million houses. 1.Why do stores usually display fruits and vegetables in the first section? A. To save customers time. B. To show they are high quality foods. C. To help sell junk food. D. To sell them at discount prices. 2.According to Paragraph 3, which of the following encourages customers to buy? A. Opening the store early in the morning. B. Displaying British wines next to French ones. C. Inviting customers to play music. D. Filling the store with the smell of fresh bread. 3.What is the California builder's story intended to prove? A. The house structure is a key factor customers consider. B. The more costly the house is, the better it sells. C. An ocean view is much to the customers'taste. D. A good first impression increases sales. 4.What is the main purpose of the passage? A. To explain how businesses turn people into their customers. 47 B. To introduce how businesses have grown from the past. C. To report researches on customer behavior. D. To show dishonest business practices. Passage 9 2015 安徽,8 分词数:293 There are an extremely large number of ants worldwide. Each individual (个体的) ant hardly weighs anything, but put together they weigh roughly the same as all of mankind. They also live nearly everywhere, except on frozen mountain tops and around the poles. For animals their size, ants have been astonishingly successful, largely due to their wonderful social behavior. In colonies ( 群体) that range in size from a few hundred to tens of millions, they organize their lives with a clear division of labor. Even more amazing is how they achieve this level of organization. Where we use sound and sight to communicate, ants depend primarily on pheromones ( 外 激 素 ) , chemicals sent out by individuals and smelled or tasted by fellow members of their colony. When an ant finds food, it produces a pheromone that will lead others straight to where the food is. When an individual ant comes under attack or is dying, it sends out an alarm pheromone to warn the colony to prepare for a conflict as a defense unit. In fact, when it comes to the art of war, ants have no equal. They are completely fearless and will readily take on a creature much larger than themselves, attacking in large groups and overcoming their target. Such is their devotion to the common good of the colony that not only soldier ants but also worker ants will sacrifice their lives to help defeat an enemy. Behaving in this selfless and devoted manner, these little creatures have survived on Earth for more than 140 million years, far longer than dinosaurs. Because they think as one, they have a collective (集体的) intelligence greater than you would expect from its individual parts. 1.We can learn from the passage that ants are ____ . A. not willing to share food B. not found around the poles C. more successful than all other animals D. too many to achieve any level of organization 2.Ants can use pheromones for ____. A. escape B. communication C. warning enemies D. arranging labor 3.What does the underlined expression "take on" in Paragraph 3 mean? A. Accept. B. Employ. C. Play with. D. Fight against. 4.Which of the following contributes most to the survival of ants? A. Their behavior. B. Their size. C. Their number. D. Their weight. Passage 10 2014 江苏,8 分词数:367 Most damagingly, anger weakens a person's ability to think clearly and keep control over his behaviour. The angry person loses objectivity in evaluating the emotional significance of the person or situation that arouses his anger. Not everyone experiences anger in the same way; what angers one person may amuse another. The specific expression of anger also differs from person to person based on biological and cultural forces. In contemporary culture, physical expressions of anger are generally considered too socially harmful to be tolerated. We no longer regard duels ( 决 斗 ) as an appropriate expression of anger resulting from one person's awareness of insulting behaviour on the part of another. Anger can be identified in the brain, where the electrical activity changes. Under most conditions EEG (脑电图) measures of electrical activity show balanced activity between the right 48 and left prefrontal ( 额叶前部) areas. Behaviourally this corresponds to the general even-handed disposition (意向) that most of us possess most of the time. But when we are angry the EEG of the right and left prefrontal areas aren't balanced and, as a result of this, we're likely to react. And our behavioural response to anger is different from our response to other emotions, whether positive or negative. Most positive emotions are associated with approach behaviour: we move closer to people we like. Most negative emotions, in contrast, are associated with avoidance behaviour: we move away from people and things that we dislike or that make us anxious. But anger is an exception to this pattern. The angrier we are, the more likely we are to move towards the object of our anger. This corresponds to what psychologists refer to as offensive anger: the angry person moves closer in order to influence and control the person or situation causing his anger. This approach-and-confront behaviour is accompanied by a leftward prefrontal asymmetry (不对称) of EEG activity. Interestingly, this asymmetry lessens if the angry person can experience empathy ( 同感) towards the individual who is bringing forth the angry response. In defensive anger, in contrast, the EEG asymmetry is directed to the right and the angry person feels helpless in the face of the anger-inspiring situation. 1.The "duels" example in Paragraph 2 proves that the expression of anger . A. usually has a biological basis B. varies among people C. is socially and culturally shaped D. influences one's thinking and evaluation 2.What changes can be found in an angry brain? A. Balanced electrical activity can be spotted. B. Unbalanced patterns are found in prefrontal areas. C. Electrical activity corresponds to one's behaviour. D. Electrical activity agrees with one's disposition. 3.Which of the following is typical of offensive anger? A. Approaching the source of anger. B. Trying to control what is disliked. C. Moving away from what is disliked. D. Feeling helpless in the face of anger. 4.What is the key message of the last paragraph? A. How anger differs from other emotions. B. How anger relates to other emotions. C. Behavioural responses to anger. D. Behavioural patterns of anger. Passage 11 2014 陕西,8 分词数:270 Why do Americans struggle with watching their weight,while the French,who consume rich food, continue to stay thin? Now a research by Cornell University suggests how lifestyle and decisions about eating may affect weight.Researchers conclude that the French tend to stop eating when they feel full.However,Americans tend to stop when their plate is empty or their favorite TV show is over. According to Dr.Joseph Mercola,a health expert,the French see eating as an important part of their lifestyle.They enjoy food and therefore spend a fairly long time at the table,while Americans see eating as something to be squeezed between the other daily activities.Mercola believes Americans lose the ability to sense when they are actually full. So they keep eating long after the French would have stopped.In addition,he points out that Americans drive to huge supermarkets to buy canned and frozen foods for the week. The French,instead,tend to shop daily,walking to small shops and farmers'markets where they have a choice of fresh fruits,vegetables,and eggs as well as high-quality meats for each meal. 49 After a visit to the United States, Mireille Guiliano,author of French Women Don't Get Fat,decided to write about the importance of knowing when to stop rather than suggesting how to avoid food.Today she continues to stay slim and rarely goes to the gym. In spite of all these differences, evidence shows that recent lifestyle changes may be affecting French eating habits.Today the rate of obesity — or extreme overweight — among adults is only 6 % .However, as American fast food gains acceptance and the young reject older traditions, the obesity rate among French children has reached 17% — and is growing. 1.In what way are the French different from Americans according to Dr.Joseph Mercola? A. They go shopping at supermarkets more frequently. B. They squeeze eating between the other daily activities. C. They regard eating as a key part of their lifestyle. D. They usually eat too much canned and frozen food. 2.This text is mainly about the relationship between____. A. Americans and the French B. lifestyle and obesity C. children and adults D. fast food and overweight 3.This text is mainly developed____. A. by contrast B. by space C. by process D. by classification 4.Where does this text probably come from? A. A TV interview. B. A food advertisement. C. A health report. D. A book review. Passage 12 2013 湖北,8 分词数:373 A German study suggests that people who were too optimistic about their future actually faced greater risk of disability or death within 10 years than those pessimists who expected their future to be worse.   The paper, published this March in Psychology and Aging, examined health and welfare surveys from roughly 40,000 Germans between ages 18 and 96. The surveys were conducted every year from 1993 to 2003.   Survey respondents (受访者) were asked to estimate their present and future life satisfaction on a scale of 0 to 10, among other questions.   The researchers found that young adults (age 18 to 39) routinely overestimated their future life satisfaction, while middle-aged adults (age 40 to 64) more accurately predicted how they would feel in the future. Adults of 65 and older, however, were far more likely to underestimate their future life satisfaction. Not only did they feel more satisfied than they thought they would, the older pessimists seemed to suffer a lower ratio ( 比率) of disability and death for the study period.   "We observed that being too optimistic in predicting a better future than actually observed was associated with a greater risk of disability and a greater risk of death within the following decade," wrote Frieder R. Lang, a professor at the University of Erlangen-Nuremberg.   Lang and his colleagues believed that people who were pessimistic about their future may be more careful about their actions than people who expected a rosy future.   "Seeing a dark future may encourage positive evaluations of the actual self and may contribute to taking improved precautions (预防措施)," the authors wrote.   Surprisingly, compared with those in poor health or who had low incomes, respondents who 50 enjoyed good health or income were associated with expecting a greater decline. Also, the researchers said that higher income was related to a greater risk of disability.   The authors of the study noted that there were limitations to their conclusions. Illness, medical treatment and personal loss could also have driven health outcomes.   However, the researchers said a pattern was clear. "We found that from early to late adulthood, individuals adapt their expectations of future life satisfaction from optimistic to accurate, to pessimistic," the authors concluded. 1. According to the study, who made the most accurate prediction of their future life satisfaction?  A. Optimistic adults.   B.Middle-aged adults.  C. Adults in poor health.   D.Adults of lower income. 2. Pessimism may be positive in some way because it causes people ____.  A.to fully enjoy their present life  B.to estimate their contribution accurately  C. to take measures against potential risks  D.to value health more highly than wealth 3. How do people of higher income see their future?  A. They will earn less money.   B.They will become pessimistic.  C. They will suffer mental illness.  D.They will have less time to enjoy life. 4. What is the clear conclusion of the study?  A. Pessimism guarantees chances of survival. B.Good financial condition leads to good health.  C. Medical treatment determines health outcomes. D.Expectations of future life satisfaction decline with age. Passage 13 2012 全国Ⅱ,8 分词数:240 Cold weather can be hard on pets, just like it can be hard on people. Sometimes owners forget that their pets are just as used to the warm shelter ( 住所) as they are. Some owners will leave their animals outside for a long period of time, thinking that all animals are used to living outdoors. This can put their pets in danger of serious illness. There are things you can do to keep your animal warm and safe. Keep your pets inside as much as you can when the weather is bad. If you have to take them out, stay outside with them. When you're cold enough to go inside, they probably are too. If you must leave them outside for a long time, make sure they have a warm, solid shelter against the wind, thick bedding, and plenty of non-frozen water. If left alone outside, dogs and cats can be very smart in their search for warm shelter. They can dig into snow banks or hide somewhere. Watch them closely when they are left outdoors, and provide them with shelter of good quality. Keep an eye on your pet's water. Sometimes owners don't realize that a water bowl has frozen and their pet can't get anything to drink. Animals that don't have clean and unfrozen water may drink dirty water outside, which may contain something unhealthy for them. 1.What do we learn about pets from Paragraph 1? A. They are often forgotten by their owners. B. They are used to living outdoors. C. They build their own shelters. D. They like to stay in warm places. 2.Why are pet owners asked to stay with their pets when they are out in cold weather? A. To know when to bring them inside. B. To keep them from eating bad food. 51 C. To help them find shelters. D. To keep them company. 3.If pets are left on their own outdoors in cold weather, they may____ . A. run short of clean water B. dig deep holes for fun C. dirty the snow nearby D. get lost in the wild 4.What is the purpose of this text? A. To solve a problem. B. To give practical advice. C. To tell an interesting story. D. To present a research result. 专题三 生态环保类 2018 年 Passage 1 2018 全国 I,8 分 话题:过时电子设备耗能高 词数:349 We may think we ’ re a culture that gets rid of our worn technology at the first sight of something shiny and new, but a new study shows that we keep using our old devices(装置) well after they go out of style. That ’s bad news for the environment — and our wallets — as these outdated devices consume much more energy than the newer ones that do the same things. To figure out how much power these devices are using, Callie Babbitt and her colleagues at the Rochester Institute of Technology in New York tracked the environmental costs for each product throughout its life — from when its minerals are mined to when we stop using the device. This method provided a readout for how home energy use has evolved since the early 1990s. Devices were grouped by generation. Desktop computers, basic mobile phones, and box-set TVs defined 1992. Digital cameras arrived on the scene in 1997. And MP3 players, smart phones, and LCD TVs entered homes in 2002, before tablets and e-readers showed up in 2007. As we accumulated more devices, however, we didn ’ t throw out our old ones. "The living-room television is replaced and gets planted in the kids’ room, and suddenly one day, you have a TV in every room of the house," said one researcher. The average number of electronic devices rose from four per household in 1992 to 13 in 2007. We ’re not just keeping these old devices — we continue to use them. According to the analysis of Babbitt ’s team, old desktop monitors and box TVs with cathode ray tubes are the worst devices with their energy consumption and contribution to greenhouse gas emissions(排放) more than doubling during the 1992 to 2007 window. So what ’ s the solution( 解 决 方 案 )? The team ’ s data only went up to 2007, but the researchers also explored what would happen if consumers replaced old products with new electronics that serve more than one function, such as a tablet for word processing and TV viewing. They found that more on-demand entertainment viewing on tablets instead of TVs and desktop computers could cut energy consumption by 44%. 1.What does the author think of new devices? A. They are environment-friendly. B. They are no better than the old. C. They cost more to use at home. D. They go out of style quickly. 2.Why did Babbitt’s team conduct the research? A. To reduce the cost of minerals. B. To test the life cycle of a product. 52 C. To update consumers on new technology. D. To find out electricity consumption of the devices. 3.Which of the following uses the least energy? A. The box-set TV. B. The tablet. C. The LCD TV. D. The desktop computer. 4.What does the text suggest people do about old electronic devices? A. Stop using them. B. Take them apart. C. Upgrade them. D. Recycle them. Passage 2 2018 北京,8 分 话题:大蜡螟幼虫分解塑料垃圾 词数:340 Plastic-Eating Worms Humans produce more than 300 million tons of plastic every year. Almost half of that winds up in landfills ( 垃圾填埋场) , and up to 12 million tons pollute the oceans. So far there is no effective way to get rid of it, but a new study suggests an answer may lie in the stomachs of some hungry worms. Researchers in Spain and England recently found that the worms of the greater wax moth can break down polyethylene, which accounts for 40% of plastics. The team left 100 wax worms on a commercial polyethylene shopping bag for 12 hours, and the worms consumed and broke down about 92 milligrams, or almost 3% of it. To confirm that the worms’ chewing alone was not responsible for the polyethylene breakdown, the researchers made some worms into paste (糊状 物)and applied it to plastic films.14 hours later the films had lost 13%of their mass — apparently broken down by enzymes ( 酶 ) from the worms’ stomachs.Their findings were published in Current Biology in 2017. Federica Bertocchini, coauthor of the study, says the worms’ ability to break down their everyday food — beeswax — also allows them to break down plastic."Wax is a complex mixture, but the basic bond in polyethylene, the carboncarbon bond, is there as well,"she explains. "The wax worm evolved a method or system to break this bond." Jennifer DeBruyn, a microbiologist at the University of Tennessee, who was not involved in the study , says it is not surprising that such worms can break down polyethylene. But compared with previous studies, she finds the speed of breaking down in this one exciting. The next step , DeBruyn says , will be to identify the cause of the breakdown. Is it an enzyme produced by the worm itself or by its gut microbes(肠道微生物)? Bertocchini agrees and hopes her team’s findings might one day help employ the enzyme to break down plastics in landfills. But she expects using the chemical in some kind of industrial process — not simply "millions of worms thrown on top of the plastic." 1.What can we learn about the worms in the study? A. They take plastics as their everyday food. B. They are newly evolved creatures. C. They can consume plastics. D. They wind up in landfills. 2.According to Jennifer DeBruyn, the next step of the study is to . A. identify other means of the breakdown B. find out the source of the enzyme C. confirm the research findings 53 D. increase the breakdown speed 3.It can be inferred from the last paragraph that the chemical might . A. help to raise worms B. help make plastic bags C. be used to clean the oceans D. be produced in factories in future 4.What is the main purpose of the passage? A. To explain a study method on worms. B. To introduce the diet of a special worm. C. To present a way to break down plastics. D. To propose new means to keep eco-balance. 2017 年 Passage 1 2017 全国卷Ⅰ,8 分 话题:野生动物保护 词数:289 I work with Volunteers for Wildlife, a rescue and education organization at Bailey Arboretum in Locust Valley. Trying to help injured, displaced or sick creatures can be heartbreaking; survival is never certain. However, when it works, it is simply beautiful. I got a rescue call from a woman in Muttontown. She had found a young owl(猫头鹰) on the ground. When I arrived, I saw a 2- to 3-week-old owl. It had already been placed in a carrier for safety. I examined the chick(雏鸟)and it seemed fine. If I could locate the nest, I might have been able to put it back, but no luck. My next work was to construct a nest and anchor it in a tree. The homeowner was very helpful. A wire basket was found. I put some pine branches into the basket to make this nest safe and comfortable. I placed the chick in the nest, and it quickly calmed down. Now all that was needed were the parents, but they were absent. I gave the homeowner a recording of the hunger screams of owl chicks. These advertise the presence of chicks to adults; they might also encourage our chick to start calling as well. I gave the owner as much information as possible and headed home to see what news the night might bring. A nervous night to be sure, but sometimes the spirits of nature smile on us all! The homeowner called to say that the parents had responded to the recordings. I drove over and saw the chick in the nest looking healthy and active. And it was accompanied in the nest by the greatest sight of all 54 — LUNCH! The parents had done their duty and would probably continue to do so. 1.What is unavoidable in the author’s rescue work according to paragraph 1? A. Efforts made in vain. B. Getting injured in his work. C. Feeling uncertain about his future. D. Creatures forced out of their homes. 2. Why was the author called to Muttontown? A. To rescue a woman.B. To take care of a woman. C. To look at a baby owl.D. To cure a young owl. 3. What made the chick calm down? A. A new nest.B. Some food. C. A recording. D. Its parents. 4. How would the author feel about the outcome of the event? A. It’s unexpected.B. It’s beautiful. C. It’s humorous.D. It’s discouraging. Passage 2 2017 全国卷Ⅲ,8 分 话题:生态平衡 词数:288 After years of heated debate, gray wolves were reintroduced to Yellowstone National Park. Fourteen wolves were caught in Canada and transported to the park. By last year, the Yellowstone wolf population had grown to more than 170 wolves. Gray wolves once were seen here and there in the Yellowstone area and much of the continental United States, but they were gradually displaced by human development. By the 1920s, wolves had practically disappeared from the Yellowstone area. They went farther north into the deep forests of Canada, where there were fewer humans around. The disappearance of the wolves had many unexpected results. Deer and elk populations — major food sources ( 来 源 ) for the wolf — grew rapidly. These animals consumed large amounts of vegetation (植被), which reduced plant diversity in the park. In the absence of wolves, coyote populations also grew quickly. The coyotes killed a large percentage of the park’s red foxes, and completely drove away the park’s beavers. As early as 1966, biologists asked the government to consider reintroducing wolves to Yellowstone Park. They hoped that wolves would be able to control the elk and coyote problems. 55 Many farmers opposed the plan because they feared that wolves would kill their farm animals or pets. The government spent nearly 30 years coming up with a plan to reintroduce the wolves. The U.S. Fish and Wildlife Service carefully monitors and manages the wolf packs in Yellowstone. Today, the debate continues over how well the gray wolf is fitting in at Yellowstone. Elk, deer, and coyote populations are down, while beavers and red foxes have made a comeback. The Yellowstone wolf project has been a valuable experiment to help biologists decide whether to reintroduce wolves to other parts of the country as well. 1.What is the text mainly about? A. Wildlife research in the United States. B. Plant diversity in the Yellowstone area. C. The conflict between farmers and gray wolves. D. The reintroduction of wolves to Yellowstone Park. 2.What does the underlined word "displaced" in paragraph 2 mean? A. Tested.B. Separated. C. Forced out.D. Tracked down. 3.What did the disappearance of gray wolves bring about? A. Damage to local ecology. B. A decline in the park’s income. C. Preservation of vegetation. D. An increase in the variety of animals. 4.What is the author’s attitude towards the Yellowstone wolf project? A. Doubtful.B. Positive. C. Disapproving.D. Uncaring. 2012—2016 年 Passage 1 2016 北京,8 分词数:357 California Condor's Shocking Recovery California condors are North America's largest birds, with wing-length of up to 3 meters. In the 1980s, electrical lines and lead poisoning( 铅 中 毒 ) nearly drove them to dying out. Now, 56 electric shock training and medical treatment are helping to rescue these big birds. In the late 1980s, the last few condors were taken from the wild to be bred(繁殖). Since 1992, there have been multiple reintroductions to the wild, and there are now more than 150 flying over California and nearby Arizona, Utah and Baja in Mexico. Electrical lines have been killing them off. "As they go in to rest for the night, they just don't see the power lines," says Bruce Rideout of San Diego Zoo. Their wings can bridge the gap between lines, resulting in electrocution(电死) if they touch two lines at once. So scientists have come up with a shocking idea. Tall poles, placed in large training areas, teach the birds to stay clear of electrical lines by giving them a painful but undeadly electric shock. Before the training was introduced, 66% of set-free birds died of electrocution. This has now dropped to 18%. Lead poisoning has proved more difficult to deal with. When condors eat dead bodies of other animals containing lead, they absorb large quantities of lead. This affects their nervous systems and ability to produce baby birds, and can lead to kidney( 肾 ) failure and death. So condors with high levels of lead are sent to Los Angeles Zoo, where they are treated with calcium EDTA, a chemical that removes lead from the blood over several days. This work is starting to pay off. The annual death rate for adult condors has dropped from 38% in 2000 to 5.4% in 2011. Rideout's team thinks that the California condors' average survival time in the wild is now just under eight years. "Although these measures are not effective forever, they are vital for now," he says. "They are truly good birds that are worth every effort we put into recovering them." 1.California condors attract researchers' interest because they ____. A. are active at night B. had to be bred in the wild C. are found only in California D. almost died out in the 1980s 2.Researchers have found electrical lines are ____. A. blocking condors' journey home B. big killers of California condors C. rest places for condors at night D. used to keep condors away 57 3.According to Paragraph 5, lead poisoning ____. A. makes condors too nervous to fly B. has little effect on condors' kidneys C. can hardly be gotten rid of from condors' blood D. makes it difficult for condors to produce baby birds 4.This passage shows that ____. A. the average survival time of condors is satisfactory B. Rideout's research interest lies in electric engineering C. the efforts to protect condors have brought good results D. researchers have found the final answers to the problem Passage 2 2016 江苏,8 分词数:437 El Niño, a Spanish term for "the Christ child," was named by South American fishermen who noticed that the global weather pattern, which happens every two to seven years, reduced the amount of fishes caught around Christmas. El Niño sees warm water, collected over several years in the western Pacific, flow back eastwards when winds that normally blow westwards weaken, or sometimes the other way round. The weather effects, both good and bad, are felt in many places. Rich countries gain more from powerful Niños, on balance, than they lose. A study found that a strong Niño in 1997-98 helped America's economy grow by $15 billion, partly because of better agricultural harvests: farmers in the Midwest gained from extra rain. The total rise in agricultural incomes in rich countries is greater than the fall in poor ones. But in Indonesia extremely dry forests are in flames. A multi-year drought ( 干 旱 ) in south-east Brazil is becoming worse. Though heavy rains brought about by El Niño may relieve the drought in California, they are likely to cause surface flooding and other disasters. The most recent powerful Niño, in 1997-98, killed around 21,000 people and caused damage worth $36 billion around the globe. But such Niños come with months of warning, and so much is known about how they happen that governments can prepare. According to the Overseas Development Institute (ODI), however, just 12% of disaster-relief funding in the past two decades has gone on reducing risks in advance, rather than recovery and rebuilding afterwards. This is despite evidence that a dollar spent on risk-reduction saves at least two on reconstruction. 58 Simple improvements to infrastructure ( 基础设施) can reduce the spread of disease. Better sewers ( 下水道) make it less likely that heavy rain is followed by an outbreak of the disease of bad stomach. Stronger bridges mean villages are less likely to be left without food and medicine after floods. According to a paper in 2011 by Mr Hsiang and co-authors, civil conflict is related to El Niño's harmful effects — and the poorer the country, the stronger the link. Though the relationship may not be causal, helping divided communities to prepare for disasters would at least reduce the risk that those disasters are followed by killing and wounding people. Since the poorest are least likely to make up for their losses from disasters linked to El Niño, reducing their losses needs to be the priority. 1.What can we learn about El Niño in Paragraph 1? A. It is named after a South American fisherman. B. It takes place almost every year all over the world. C. It forces fishermen to stop catching fish around Christmas. D. It sees the changes of water flow direction in the ocean. 2.What may El Niños bring about to the countries affected? A. Agricultural harvests in rich countries fall. B. Droughts become more harmful than floods. C. Rich countries' gains are greater than their losses. D. Poor countries suffer less from droughts economically. 3.The data provided by ODI in Paragraph 4 suggest that ____. A. more investment should go to risk reduction B. governments of poor countries need more aid C. victims of El Niño deserve more compensation D. recovery and reconstruction should come first 4.What is the author's purpose in writing the passage? A. To introduce El Niño and its origin. B. To explain the consequences of El Niño. C. To show ways of fighting against El Niño. D. To urge people to prepare for El Niño. Passage 3 2015 浙江,10 分词数:492 If humans were truly at home under the light of the moon and stars, we would go in darkness happily, the midnight world as visible to us as it is to the vast number of nocturnal (夜间活动的) 59 species on this planet. Instead, we are diurnal creatures, with eyes adapted to living in the sun's light. This is a basic evolutionary fact, even though most of us don't think of ourselves as diurnal beings. Yet it's the only way to explain what we've done to the night: We've engineered it to receive us by filling it with light. The benefits of this kind of engineering come with consequences — called light pollution — whose effects scientists are only now beginning to study. Light pollution is largely the result of bad lighting design, which allows artificial light to shine outward and upward into the sky. Ill-designed lighting washes out the darkness of night and completely changes the light levels — and light rhythms — to which many forms of life, including ourselves, have adapted. Wherever human light spills into the natural world, some aspect of life is affected. In most cities the sky looks as though it has been emptied of stars, leaving behind a vacant haze ( 霾 ) that mirrors our fear of the dark. We've grown so used to this orange haze that the original glory of an unlit night — dark enough for the planet Venus to throw shadows on Earth — is wholly beyond our experience, beyond memory almost. We've lit up the night as if it were an unoccupied country, when nothing could be further from the truth. Among mammals alone, the number of nocturnal species is astonishing. Light is a powerful biological force, and on many species it acts as a magnet ( 磁 铁 ). The effect is so powerful that scientists speak of songbirds and seabirds being "captured" by searchlights on land or by the light from gas flares on marine oil platforms. Migrating at night, birds tend to collide with brightly lit tall buildings. Frogs living near brightly lit highways suffer nocturnal light levels that are as much as a million times brighter than normal, throwing nearly every aspect of their behavior out of joint, including their nighttime breeding choruses. Humans are no less trapped by light pollution than the frogs. Like most other creatures, we do need darkness. Darkness is as essential to our biological welfare, to our internal clockwork, as light itself. Living in a glare of our own making, we have cut ourselves off from our evolutionary and cultural heritage — the light of the stars and the rhythms of day and night. In a very real sense, light pollution causes us to lose sight of our true place in the universe, to forget the scale of our being, which is best measured against the dimensions of a deep night with the Milky Way — the edge of our galaxy — arching overhead. 60 1.According to the passage, human beings ____. A. prefer to live in the darkness B. are used to living in the day light C. were curious about the midnight world D. had to stay at home with the light of the moon 2.What does "it" (Paragraph 1) most probably refer to? A. The night. B. The moon. C. The sky. D. The planet. 3.The writer mentions birds and frogs to ____. A. provide examples of animal protection B. show how light pollution affects animals C. compare the living habits of both species D. explain why the number of certain species has declined 4.It is implied in the last paragraph that ____. A. light pollution does harm to the eyesight of animals B. light pollution has destroyed some of the world heritages C. human beings cannot go to the outer space D. human beings should reflect on their position in the universe 5.What might be the best title for the passage? A. The Magic Light. B. The Orange Haze. C. The Disappearing Night. D. The Rhythms of Nature. Passage 4 2015 湖北,8 分词数:380 "I see you've got a bit of water on your coat,"said the man at the petrol station. "Is it raining out there?" "No,it's pretty nice," I replied, checking my sleeve."Oh, right. A pony ( 马驹) bit me earlier." As it happened, the bite was virtually painless: more the kind of small bite you might get from a naughty child. The pony responsible was queuing up for some ice cream in the car park near Haytor,and perhaps thought I'd jumped in ahead of him. The reason why the ponies here are naughty is that Haytor is a tourist-heavy area and tourists are constantly feeding the ponies foods,despite signs asking them not to.By feeding the ponies,tourists increase the risk of them getting hit by a car,and make them harder to gather during the area's annual pony drift (迁移). The purpose of a pony drift is to gather them up so their health can be checked,the baby ones 61 can be stopped from feeding on their mothers' milk,and those who've gone beyond their limited area can be returned to their correct area.Some of them are also later sold,in order to limit the number of ponies according to the rules set by Natural England. Three weeks ago, I witnessed a small near-disaster a few miles west of here. While walking, I noticed a pony roll over on his back."Hello!" I said to him, assuming he was just rolling for fun, but he was very still and, as I got closer, I saw him kicking his legs in the air and breathing heavily. I began to properly worry about him. Fortunately, I managed to get in touch with a Dartmoor's Livestock Protection officer and send her a photo. The officer immediately sent a local farmer out to check on the pony. The pony had actually been trapped between two rocks. The farmer freed him, and he began to run happily around again. Dartmoor has 1,000 or so ponies, who play a critical role in creating the diversity of species in this area. Many people are working hard to preserve these ponies, and trying to come up with plans to find a sustainable ( 可持续的) future for one of Dartmoor's most financially-troubled elements. 1.Why are tourists asked not to feed the ponies? A. To protect the tourists from being bitten. B. To keep the ponies off the petrol station. C. To avoid putting the ponies in danger. D. To prevent the ponies from fighting. 2.One of the purposes of the annual pony drift is ____. A. to feed baby ponies on milk B. to control the number of ponies C. to expand the habitat for ponies D. to sell the ponies at a good price 3.What was the author's first reaction when he saw a pony roll on its back? A. He freed it from the trap. B. He called a protection officer. C. He worried about it very much. D. He thought of it as being naughty. 4.What does the author imply about the preservation of Dartmoor's ponies? A. It lacks people's involvement. B. It costs a large amount of money. C. It will affect tourism in Dartmoor. D. It has caused an imbalance of species. Passage 5 2014 新课标全国Ⅱ,8 分词数:254 Since the first Earth Day in 1970, Americans have gotten a lot "greener" toward the environment. "We didn't know at that time that there even was an environment, let alone that there was a problem with it," says Bruce Anderson, president of Earth Day USA. 62 But what began as nothing important in public affairs has grown into a social movement. Business people, political leaders, university professors, and especially millions of grass-roots Americans are taking part in the movement. "The understanding has increased many, many times," says Gaylord Nelson, the former governor from Wisconsin, who thought up the first Earth Day. According to US government reports, emissions ( 排放) from cars and trucks have dropped from 10.3 million tons a year to 5.5 million tons. The number of cities producing CO beyond the standard has been reduced from 40 to 9. Although serious problems still remain and need to be dealt with, the world is a safer and healthier place. A kind of "green thinking" has become part of practices. Great improvement has been achieved. In 1988 there were only 600 recycling programs; today in 1995 there are about 6,600. Advanced lights, motors, and building designs have helped save a lot of energy and therefore prevented pollution. Twenty-five years ago, there were hardly any education programs for environment. Today, it's hard to find a public school, university, or law school that does not have such a kind of program. "Until we do that, nothing else will change!" says Bruce Anderson. 1.According to Anderson, before 1970, Americans had little idea about ____. A. the social movement B. recycling techniques C. environmental problems D. the importance of Earth Day 2.Where does the support for environmental protection mainly come from? A. The grass-roots level. B. The business circle. C. Government officials. D. University professors. 3.What have Americans achieved in environmental protection? A. They have cut car emissions to the lowest. B. They have settled their environmental problems. C. They have lowered their CO levels in forty cities. D. They have reduced pollution through effective measures. 4.What is especially important for environmental protection according to the last paragraph? A. Education. B. Planning. C. Green living. D. CO reduction. Passage 6 2014 湖南,10 分词数:319 63 The behaviour of a building's users may be at least as important as its design when it comes to energy use, according to new research from the UK Energy Research Centre(UKERC). The UK promises to reduce its carbon emissions ( 排放) by 80 percent by 2050, part of which will be achieved by all new homes being zero-carbon by 2016. But this report shows that sustainable building design on its own — though extremely important — is not enough to achieve such reductions: the behaviour of the people using the building has to change too. The study suggests that the ways that people use and live in their homes have been largely ignored by existing efforts to improve energy efficiency ( 效 率 ), which instead focus on architectural and technological developments. ‘Technology is going to assist but it is not going to do everything,’explains Katy Janda, a UKERC senior researcher, ‘consumption patterns of building users can defeat the most careful design.’ In other words, old habits die hard, even in the best-designed eco-home. Another part of the problem is information. Households and bill-payers don't have the knowledge they need to change their energy-use habits. Without specific information, it's hard to estimate the costs and benefits of making different choices. Feedback(反馈) facilities, like smart meters and energy monitors, could help bridge this information gap by helping people see how changing their behaviour directly affects their energy use; some studies have shown that households can achieve up to 15 percent energy savings using smart meters. Social science research has added a further dimension ( 方面), suggesting that individuals' behaviour in the home can be personal and cannot be predicted — whether people throw open their windows rather than turn down the thermostat (恒温器), for example. Janda argues that education is the key. She calls for a focused programme to teach people about buildings and their own behaviour in them. 1.As to energy use, the new research from UKERC stresses the importance of ____. A. zero-carbon homes B. the behaviour of building users C. sustainable building design D. the reduction of carbon emissions 2.The underlined word "which" in Paragraph 2 refers to "____". A. the ways B. their homes C. developments D. existing efforts 3.What are Katy Janda's words mainly about? A. The importance of changing building users' habits. 64 B. The necessity of making a careful building design. C. The variety of consumption patterns of building users. D. The role of technology in improving energy efficiency. 4.The information gap in energy use ____. A. can be bridged by feedback facilities B. affects the study on energy monitors C. brings about problems for smart meters D. will be caused by building users' old habits 5.What does the dimension added by social science research suggest? A. The social science research is to be furthered. B. The education programme is under discussion. C. The behaviour of building users is unpredictable. D. The behaviour preference of building users is similar. Passage 7 2014 浙江,10 分词数:422 Last summer, two nineteenth-century cottages were rescued from remote farm fields in Montana, to be moved to an Art Deco building in San Francisco. The houses were made of wood. These cottages once housed early settlers as they worked the dry Montana soil; now they hold Twitter engineers. The cottages could be an example of the industry's odd love affair with "low technology" , a concept associated with the natural world, and with old-school craftsmanship ( 手艺) that exists long before the Internet era. Low technology is not virtual (虚拟的) — so, to take advantage of it, Internet companies have had to get creative. The rescued wood cottages, fitted by hand in the late eighteen-hundreds, are an obvious example, but Twitter's designs lie on the extreme end. Other companies are using a broader interpretation (阐释) of low technology that focuses on nature. Amazon is building three glass spheres filled with trees, so that employees can "work and socialize in a more natural, park-like setting" . At Google's office, an entire floor is carpeted in grass. Facebook's second Menlo Park campus will have a rooftop park with a walking trail. Olle Lundberg, the founder of Lundberg Design, has worked with many tech companies over the years. "We have lost the connection to the maker in our lives, and our tech engineers are the ones who feel most impoverished (贫乏的), because they're surrounded by the digital world," he says. "They're looking for a way to regain their individual identity, and we've found that introducing real crafts is one way to do that." 65 This craft-based theory is rooted in history. William Morris, the English artist and writer, turned back to pre- industrial arts in the eighteen-sixties, just after the Industrial Revolution. The Arts and Crafts movement defined itself against machines. "Without creative human occupation, people became disconnected from life," Morris said. Research has shown that natural environments can restore ( 恢复) our mental capacities. In Japan, patients are encouraged to "forest-bathe", taking walks through woods to lower their blood pressure. These health benefits apply to the workplace as well. Rachel Kaplan, a professor of environmental psychology, has spent years researching the restorative effects of natural environments. Her research found that workers with access to nature at the office — even simple views of trees and flowers — felt their jobs were less stressful and more satisfying. If lowtech offices can potentially nourish the brains and improve the mental health of employees then, fine, bring on the cottages. 1.The writer mentions the two nineteenth century cottages to show that____. A. Twitter is having a hard time B. old cottages are in need of protection C. early settlers once suffered from a dry climate in Montana D. Internet companies have rediscovered the benefits of low technology 2.Low technology is regarded as something that____. A. is related to nature B. is out of date today C. consumes too much energy D. exists in the virtual world 3.The main idea of Paragraph 5 is that human beings _____. A. have destroyed many pre-industrial arts B. have a tradition of valuing arts and crafts C. can become intelligent by learning history D. can regain their individual identity by using machines 4.The writer's attitude to "low technology" can best be described as____. A. positive B. defensive C. cautious D. doubtful 5.What might be the best title for the passage? A. Past Glories, Future Dreams. B. The Virtual World, the Real Challenge. 66 C. High-tech Companies, Low-tech Offices. D. The More Craftsmanship, the Less Creativity. Passage 8 2013 江西,8 分词数:293 Going green seems to be a fad (时尚) for a lot of people these days. Whether that is good or bad, we can't really say, but for the two of us, going green is not a fad but a lifestyle. On April 22, 2011, we decided to go green every single day for an entire year. This meant doing 365 different green things, and it also meant challenging ourselves to go green beyond the easy things. Rather than recycle and reduce our energy, we had to think of 365 different green things to do and this was no easy task. With the idea of going green every single day for a year, Our Green Year started. My wife and I decided to educate people about how they could go green in their lives and hoped we could show people all the green things that could be done to help the environment. We wanted to push the message that every little bit helps. Over the course of Our Green Year, we completely changed our lifestyles. We now shop at organic(有机的) stores. We consume less meat, choosing green food. We have greatly reduced our buying we don't need. We have given away half of what we owned through websites. Our home is kept clean by vinegar and lemon juice, with no chemical cleaners. We make our own butter, enjoying the smell of home-made fresh bread. In our home office anyone caught doing something ungreen might be punished. Our minds have been changed by Our Green Year. We are grateful for the chance to have been able to go green and educate others. We believe that we do have the power to change things and help our planet. 1.What might be the best title for the passage? A. Going Green B. Protecting the Planet C. Keeping Open Minded D. Celebrating Our Green Year 2.It was difficult for the couple to live a green life for the whole year because____. A. they were expected to follow the green fad B. they didn't know how to educate other people C. they were unwilling to reduce their energy D. they needed to perform unusual green tasks 3.What did the couple do over the course of Our Green Year? 67 A. They tried to get out of their ungreen habits. B. They ignored others' ungreen behavior. C. They chose better chemical cleaners. D. They sold their homemade food. 4.What can we infer from the last paragraph? A.The government will give support to the green project. B.The couple may continue their project in the future. C.Some people disagree with the couple's green ideas. D.Our Green Year is becoming a national campaign. Passage 9 2013 安徽,8 分词数:256 Using too much water or throwing rubbish into our rivers are clear ways that humans can put our water supply in danger,but we also affect our water supply in less obvious ways.You may wonder how paving (铺砌) a road can lead to less useable fresh water.A major part of the water we use every day is groundwater.Groundwater does not come from lakes or rivers.It comes from underground. The more roads and parking lots we pave,the less water can flow into the ground to become groundwater. Human activity is not responsible for all water shortages (短缺) . Drier climates are of course more likely to have droughts ( 干 旱 ) than areas with more rainfall, but in any case, good management can help to make sure there is enough water to meet our basic needs. Thinking about the way we use water every day can make a big difference, too. In the United States, a family of four can use 1.5 tons of water a day! This shows how much we depend on water to live, but there's a lot we can do to lower the number. You can take steps to save water in your home. To start with, use the same glass for your drinking water all day. Wash it only once a day. Run your dishwasher ( 洗碗机) only when it is full. Help your parents fix any leaks in your home. You can even help to keep our water supply clean by recycling batteries instead of throwing them away. 1. Which of the following is most likely to lead to less groundwater? A. Using river water. B. Throwing batteries away. C. Paving parking lots. D. Throwing rubbish into lakes. 2. What can be inferred from the text? 68 A. All water shortages are due to human behavior. B. It takes a lot of effort to meet our water needs. C. There is much we can do to reduce family size. D. The average family in America makes proper use of water. 3. The last paragraph is intended to ____. A. show us how to fix leaks at home B. tell us how to run a dishwasher C. prove what drinking glass is best for us D. suggest what we do to save water at home 4. The text is mainly about ____. A. why paving roads reduces our water B. how much we depend on water to live C. why droughts occur more in dry climates D. how human activity affects our water supply Passage 10 2012 北京,8 分词数:369 "In wilderness (荒野) is the preservation of the world." This is a famous saying from a writer regarded as one of the fathers of environmentalism. The frequency with which it is borrowed mirrors a heated debate on environmental protection: whether to place wilderness at the heart of what is to be preserved. As John Sauven of Greenpeace UK points out, there is a strong appeal in images of the wild, the untouched; more than anything else, they speak of the nature that many people value most dearly. The urge to leave the subject of such images untouched is strong, and the danger exploitation ( 开发) brings to such landscapes ( 景观) is real. Some of these wildernesses also perform functions that humans need — the rainforests, for example, store carbon in vast quantities. To Mr. Sauven, these "ecosystem services" far outweigh the gains from exploitation. Lee Lane, a visiting fellow at the Hudson Institute, takes the opposing view. He acknowledges that wildernesses do provide useful services, such as water conservation. But that is not, he argues, a reason to avoid all human presence, or indeed commercial and industrial exploitation. There are ever more people on the Earth, and they reasonably and rightfully want to have better lives, rather than merely struggle for survival. While the ways of using resources have improved, there is still a growing need for raw materials, and some wildernesses contain them in abundance. If they can be tapped without reducing the services those wildernesses provide, the argument goes, there is no further reason not to do so. Being untouched is not, in itself, a characteristic worth valuing above all others. 69 I look forward to seeing these views taken further, and to their being challenged by the other participants. One challenge that suggests itself to me is that both cases need to take on the question of spiritual value a little more directly. And there is a practical question as to whether wildernesses can be exploited without harm. This is a topic that calls for not only free expression of feelings, but also the guidance of reason. What position wilderness should enjoy in the preservation of the world obviously deserves much more serious thinking. 1.John Sauven holds that ____. A. many people value nature too much B. exploitation of wildernesses is harmful C. wildernesses provide humans with necessities D. the urge to develop the ecosystem services is strong 2.What is the main idea of Para. 3? A. The exploitation is necessary for the poor people. B. Wildernesses cannot guarantee better use of raw materials. C. Useful services of wildernesses are not the reason for no exploitation. D. All the characteristics concerning the exploitation should be treated equally. 3.What is the author's attitude towards this debate? A. Objective. B. Disapproving. C. Sceptical. D. Optimistic. 4.Which of the following shows the structure of the passage? CP: Central Point P: Point Sp: Sub-point (次要点) C: Conclusion 专题四 社会生活类 2018 年 70 Passage 1 2018 全国 I,8 分 话题:少花钱,健康饮食 词数:265 Good Morning Britain’s Susanna Reid is used to grilling guests on the sofa every morning, but she is cooking up a storm in her latest role — showing families how to prepare delicious and nutritious meals on a tight budget. In Save Money: Good Food, she visits a different home each week and with the help of chef Matt Tebbutt offers top tips on how to reduce food waste, while preparing recipes for under £5 per family a day. And the Good Morning Britain presenter says she ’s been able to put a lot of what she’s learnt into practice in her own home, preparing meals for sons, Sam, 14, Finn, 13, and Jack, 11. "We love Mexican churros, so I buy them on my phone from my local Mexican takeaway restaurant," she explains. "I pay £5 for a portion(一份),but Matt makes them for 26p a portion, because they are flour, water, sugar and oil. Everybody can buy takeaway food, but sometimes we’ re not aware how cheaply we can make this food ourselves." The eight-part series(系列节目), Save Money: Good Food, follows in the footsteps of ITV’s Save Money: Good Health, which gave viewers advice on how to get value from the vast range of health products on the market. With food our biggest weekly household expense, Susanna and Matt spend time with a different family each week. In tonight’s Easter special they come to the aid of a family in need of some delicious inspiration on a budget. The team transforms the family ’ s long weekend of celebration with less expensive but still tasty recipes. 1. What do we know about Susanna Reid? A. She enjoys embarrassing her guests. B. She has started a new programme. C. She dislikes working early in the morning. D. She has had a tight budget for her family. 2. How does Matt Tebbutt help Susanna? A. He buys cooking materials for her. B. He prepares food for her kids. C. He assists her in cooking matters. D. He invites guest families for her. 3. What does the author intend to do in paragraph 4? A. Summarize the previous paragraphs. B. Provide some advice for the readers. C. Add some background information. D. Introduce a new topic for discussion. 4. What can be a suitable title for the text? A. Keeping Fit by Eating Smart B. Balancing Our Daily Diet C. Making Yourself a Perfect Chef D. Cooking Well for Less Passage 2 2018 全国 I,8 分 话题:全球语言多样性面临危机 词数:298 Languages have been coming and going for thousands of years, but in recent times there has been less coming and a lot more going. When the world was still populated by hunter-gatherers, 71 small, tightly knit (联系) groups developed their own patterns of speech independent of each other. Some language experts believe that 10,000 years ago, when the world had just five to ten million people, they spoke perhaps 12,000 languages between them. Soon afterwards, many of those people started settling down to become farmers, and their languages too became more settled and fewer in number. In recent centuries, trade, industrialisation, the development of the nation-state and the spread of universal compulsory education, especially globalisation and better communications in the past few decades, all have caused many languages to disappear, and dominant languages such as English, Spanish and Chinese are increasingly taking over.At present, the world has about 6,800 languages. The distribution of these languages is hugely uneven. The general rule is that mild zones have relatively few languages, often spoken by many people, while hot, wet zones have lots, often spoken by small numbers. Europe has only around 200 languages; the Americas about 1,000; Africa 2,400; and Asia and the Pacific perhaps 3,200, of which Papua New Guinea alone accounts for well over 800. The median number(中位数) of speakers is a mere 6,000, which means that half the world ’ s languages are spoken by fewer people than that. Already well over 400 of the total of 6,800 languages are close to extinction (消亡), with only a few elderly speakers left. Pick, at random, Busuu in Cameroon (eight remaining speakers), Chiapaneco in Mexico (150), Lipan Apache in the United States (two or three) or Wadjigu in Australia(one, with a question-mark): none of these seems to have much chance of survival. 1.What can we infer about languages in hunter-gatherer times? A. They developed very fast. B. They were large in number. C. They had similar patterns. D. They were closely connected. 2.Which of the following best explains "dominant" underlined in paragraph 2? A. Complex. B. Advanced. C. Powerful. D. Modern. 3.How many languages are spoken by less than 6,000 people at present? A. About 6,800. B. About 3,400. C. About 2,400. D. About 1,200. 4.What is the main idea of the text? A. New languages will be created. B. People’s lifestyles are reflected in languages. C. Human development results in fewer languages. D. Geography determines language evolution. Passage 3 2018 全国 II,8 分 话题:七月成熟的浆果和核果 词数:252 Many of us love July because it’s the month when nature’s berries and stone fruits are in abundance. These colourful and sweet jewels from British Columbia ’ s fields are little powerhouses of nutritional protection. Of the common berries, strawberries are highest in vitamin C, although, because of their seeds, raspberries contain a little more protein(蛋白质), iron and zinc (not that fruits have much protein). Blueberries are particularly high in antioxidants( 抗氧化物质). The yellow and orange stone fruits such as peaches are high in the carotenoids we turn into vitamin A and which are antioxidants. As for cherries( 樱桃), they are so delicious who cares? However, they are rich in 72 vitamin C. When combined with berries or slices of other fruits, frozen bananas make an excellent base for thick, cooling fruit shakes and low fat "ice cream". For this purpose, select ripe bananas for freezing as they are much sweeter. Remove the skin and place them in plastic bags or containers and freeze. If you like, a squeeze of fresh lemon juice on the bananas will prevent them turning brown. Frozen bananas will last several weeks, depending on their ripeness and the temperature of the freezer. If you have a juicer, you can simply feed in frozen bananas and some berries of sliced fruit. Out comes a "soft-serve" creamy dessert, to be eaten right away. This makes a fun activity for a children’s party; they love feeding the fruit and frozen bananas into the top of the machine and watching the ice cream come out below. 1.What does the author seem to like about cherries? A. They contain protein. B. They are high in vitamin A. C. They have a pleasant taste. D. They are rich in antioxidants. 2.Why is fresh lemon juice used in freezing bananas? A. To make them smell better. B. To keep their colour. C. To speed up their ripening. D. To improve their nutrition. 3.What is "a juicer" in the last paragraph? A. A dessert. B. A drink. C. A container. D. A machine. 4.From which is the text probably taken? A. A biology textbook. B. A health magazine. C. A research paper. D. A travel brochure. Passage 4 2018 全国 II,8 分 话题:闲聊的作用 词数:309 We ’ve all been there: in a lift, in line at the bank or on an airplane, surrounded by people who are, like us, deeply focused on their smartphones or, worse, struggling with the uncomfortable silence. What ’s the problem? It ’s possible that we all have compromised conversational intelligence. It ’s more likely that none of us start a conversation because it ’s awkward and challenging, or we think it ’s annoying and unnecessary. But the next time you find yourself among strangers, consider that small talk is worth the trouble. Experts say it ’s an invaluable social practice that results in big benefits. Dismissing small talk as unimportant is easy, but we can ’t forget that deep relationships wouldn’t even exist if it weren’t for casual conversation. Small talk is the grease(润滑剂) for social communication, says Bernardo Carducci, director of the Shyness Research Institute at Indiana University Southeast."Almost every great love story and each big business deal begins with small talk," he explains."The key to successful small talk is learning how to connect with others, not just communicate with them." In a 2014 study, Elizabeth Dunn, associate professor of psychology at UBC, invited people 73 on their way into a coffee shop. One group was asked to seek out an interaction( 互动)with its waiter; the other, to speak only when necessary. The results showed that those who chatted with their server reported significantly higher positive feelings and a better coffee shop experience. "It’ s not that talking to the waiter is better than talking to your husband," says Dunn. "But interactions with peripheral(边缘的)members of our social network matter for our well-being also." Dunn believes that people who reach out to strangers feel a significantly greater sense of belonging, a bond with others. Carducci believes developing such a sense of belonging starts with small talk. "Small talk is the basis of good manners," he says. 1.What phenomenon is described in the first paragraph? A. Addiction to smartphones. B. Inappropriate behaviours in public places. C. Absence of communication between strangers. D. Impatience with slow service. 2.What is important for successful small talk according to Carducci? A. Showing good manners. B. Relating to other people. C. Focusing on a topic. D. Making business deals. 3.What does the coffee-shop study suggest about small talk? A. It improves family relationships. B. It raises people’s confidence. C. It matters as much as a formal talk. D. It makes people feel good. 4.What is the best title for the text? A. Conversation Counts B. Ways of Making Small Talk C. Benefits of Small Talk D. Uncomfortable Silence Passage 5 2018 全国 III,8 分 话题:简约生活 词数:305 Adults understand what it feels like to be flooded with objects. Why do we often assume that more is more when it comes to kids and their belongings? The good news is that I can help my own kids learn earlier than I did how to live more with less. I found the pre-holidays a good time to encourage young children to donate less-used things, and it worked. Because of our efforts, our daughter Georgia did decide to donate a large bag of toys to a little girl whose mother was unable to pay for her holiday due to illness. She chose to sell a few larger objects that were less often used when we promised to put the money into her school fund(基金) (our kindergarten daughter is serious about becoming a doctor). For weeks, I’ve been thinking of bigger, deeper questions: How do we make it a habit for them? And how do we train ourselves to help them live with, need, and use less? Yesterday, I sat with my son, Shepherd, determined to test my own theory on this. I decided to play with him with only one toy for as long as it would keep his interest. I expected that one toy would keep his attention for about five minutes, ten minutes, max. I chose a red rubber ball — simple, universally available. We passed it, he tried to put it in his mouth, he tried bouncing it, rolling it, sitting on it, 74 throwing it. It was totally, completely enough for him. Before I knew it an hour had passed and it was time to move on to lunch. We both became absorbed in the simplicity of playing together. He had my full attention and I had his. My little experiment to find joy in a single object worked for both of us. 1.What do the words "more is more" in paragraph 1 probably mean? A. The more, the better. B. Enough is enough. C. More money, more worries. D. Earn more and spend more. 2.What made Georgia agree to sell some of her objects? A. Saving up for her holiday. B. Raising money for a poor girl. C. Adding the money to her fund. D. Giving the money to a sick mother. 3.Why did the author play the ball with Shepherd? A. To try out an idea. B. To show a parent’s love. C. To train his attention. D. To help him start a hobby. 4.What can be a suitable title for the text? A. Take It or Leave It B. A Lesson from Kids C. Live More with Less D. The Pleasure of Giving 2017 年 Passage 1 2017 全国卷Ⅲ,8 分 话题:广场剧院即将关闭 词数:277 Minutes after the last movie ended yesterday at the Plaza Theater, employees were busy sweeping up popcorns and gathering coke cups. It was a scene that had been repeated many times in the theater’s 75-year history. This time, however, the cleanup was a little different. As one group of workers carried out the rubbish, another group began removing seats and other theater equipment in preparation for the building’s end. The film classic The Last Picture Show was the last movie shown in the old theater. Though the movie is 30 years old, most of the 250 seats were filled with teary-eyed audience wanting to say good-bye to the old building. Theater owner Ed Bradford said he chose the movie because it seemed appropriate. The movie is set in a small town where the only movie theater is preparing to close down. Bradford said that large modern theaters in the city made it impossible for the Plaza to compete. 75 He added that the theater ’s location( 位置) was also a reason. "This used to be the center of town," he said. "Now the area is mostly office buildings and warehouses." Last week some city officials suggested the city might be interested in turning the old theater into a museum and public meeting place. However, these plans were abandoned because of financial problems. Bradford sold the building and land to a local development firm, which plans to build a shopping complex on the land where the theater is located. The theater audience said good-bye as Bradford locked the doors for the last time. After 75 years the Plaza Theater had shown its last movie. The theater will be missed. 1.In what way was yesterday’s cleanup at the Plaza special? A. It made room for new equipment. B. It signaled the closedown of the theater. C. It was done with the help of the audience. D. It marked the 75th anniversary of the theater. 2.Why was The Last Picture Show put on? A. It was an all-time classic. B. It was about the history of the town. C. The audience requested it. D. The theater owner found it suitable. 3.What will probably happen to the building? A. It will be repaired. B. It will be turned into a museum. C. It will be knocked down. D. It will be sold to the city government. 4.What can we infer about the audience? A. They are disappointed with Bradford. B. They are sad to part with the old theater. C. They are supportive of the city officials. D. They are eager to have a shopping center. Passage 2 2017 北京,8 分 话题:麻疹疫苗 词数:346 Measles( 麻疹), which once killed 450 children each year and disabled even more, was nearly 76 wiped out in the United States 14 years ago by the universal use of the MMR vaccine( 疫苗). But the disease is making a comeback, caused by a growing anti-vaccine movement and misinformation that is spreading quickly. Already this year, 115 measles cases have been reported in the USA, compared with 189 for all of last year. The numbers might sound small, but they are the leading edge of a dangerous trend. When vaccination rates are very high, as they still are in the nation as a whole, everyone is protected. This is called "herd immunity", which protects the people who get hurt easily, including those who can’t be vaccinated for medical reasons, babies too young to get vaccinated and people on whom the vaccine doesn’t work. But herd immunity works only when nearly the whole herd joins in. When some refuse vaccination and seek a free ride, immunity breaks down and everyone is in even bigger danger. That ’ s exactly what is happening in small neighborhoods around the country from Orange County, California, where 22 measles cases were reported this month, to Brooklyn, N.Y., where a 17-year-old caused an outbreak last year. The resistance to vaccine has continued for decades, and it is driven by a real but very small risk. Those who refuse to take that risk selfishly make others suffer. Making things worse are state laws that make it too easy to opt out( 决定不参加) of what are supposed to be required vaccines for all children entering kindergarten. Seventeen states allow parents to get an exemption( 豁 免 ), sometimes just by signing a paper saying they personally object to a vaccine. Now, several states are moving to tighten laws by adding new regulations for opting out. But no one does enough to limit exemptions. Parents ought to be able to opt out only for limited medical or religious reasons. But personal opinions? Not good enough. Everyone enjoys the life-saving benefits vaccines provide, but they’ ll exist only as long as everyone shares in the risks. 1.The first two paragraphs suggest that . A. a small number of measles cases can start a dangerous trend B. the outbreak of measles attracts the public attention C. anti-vaccine movement has its medical reasons D. information about measles spreads quickly 77 2.Herd immunity works well when . A. exemptions are allowed B. several vaccines are used together C. the whole neighborhood is involved in D. new regulations are added to the state laws 3.What is the main reason for the comeback of measles? A. The overuse of vaccine. B. The lack of medical care. C. The features of measles itself. D. The vaccine opt-outs of some people. 4.What is the purpose of the passage? A. To introduce the idea of exemption. B. To discuss methods to cure measles. C. To stress the importance of vaccination. D. To appeal for equal rights in medical treatment. Passage 3 2017 天津,12.5 分 话题:共享 词数:348 Fifteen years ago, I took a summer vacation in Lecce in southern Italy. After climbing up a hill for a panoramic ( 全景的) view of the blue sea, white buildings and green olive trees, I paused to catch my breath and then positioned myself to take the best photo of this panorama. Unfortunately, just as I took out my camera, a woman approached from behind, and planted herself right in front of my view. Like me, this woman was here to stop, sigh and appreciate the view. Patient as I was, after about 15 minutes, my camera scanning the sun and reviewing the shot I would eventually take, I grew frustrated. Was it too much to ask her to move so I could take just one picture of the landscape? Sure, I could have asked her, but something prevented me from doing so. She seemed so content in her observation. I didn’t want to mess with that. Another 15 minutes passed and I grew bored. The woman was still there. I decided to take the photo anyway. And now when I look at it, I think her presence in the photo is what makes the image interesting. The landscape, beautiful on its own, somehow comes to life and breathes because this woman is engaging with it. 78 This photo, with the unique beauty that unfolded before me and that woman who "ruined" it, now hangs on a wall in my bedroom. What would she think if she knew that her figure is captured (捕 捉) and frozen on some stranger’s bedroom wall? A bedroom, after all, is a very private space, in which some woman I don’t even know has been immortalized (使……永存). In some ways, she lives in my house. Perhaps we all live in each others’ spaces. Perhaps this is what photos are for: to remind us that we all appreciate beauty, that we all share a common desire for pleasure, for connection, for something that is greater than us. That photo is a reminder, a captured moment, an unspoken conversation between two women, separated only by a thin square of glass. 1.What happened when the author was about to take a photo? A. Her camera stopped working. B. A woman blocked her view. C. Someone asked her to leave. D. A friend approached from behind. 2.According to the author, the woman was probably . A. enjoying herself B. losing her patience C. waiting for the sunset D. thinking about her past 3.In the author’s opinion, what makes the photo so alive? A. The rich color of the landscape. B. The perfect positioning of the camera. C. The woman’s existence in the photo. D. The soft sunlight that summer day. 4.The photo on the bedroom wall enables the author to better understand . A. the need to be close to nature B. the importance of private space C. the joy of the vacation in Italy D. the shared passion for beauty 79 5.The passage can be seen as the author’s reflections upon . A. a particular life experience B. the pleasure of traveling C. the art of photography D. a lost friendship 2012—2016 年 Passage 1 2016 全国卷Ⅰ,8 分 词数:291 Grandparents Answer a Call As a third-generation native of Brownsville, Texas, Mildred Garza never planned to move away. Even when her daughter and son asked her to move to San Antonio to help with their children, she politely refused. Only after a year of friendly discussion did Ms. Garza finally say yes. That was four years ago. Today all three generations regard the move as a success, giving them a closer relationship than they would have had in separate cities. No statistics show the number of grandparents like Garza who are moving closer to adult children and grandchildren. Yet there is evidence suggesting that the trend is growing. Even President Obama's mother-in-law, Marian Robinson, has agreed to leave Chicago and move into the White House to help care for her granddaughters. According to a study by grandparents.com, 83 percent of the people said Mrs. Robinson's decision will influence grandparents in the American family. Two-thirds believe more families will follow the example of Obama's family. "In the 1960s we were all a little wild and couldn't get away from home far enough or fast enough to prove we could do it on our own,"says Christine Crosby, publisher of Grand, a magazine for grandparents."We now realize how important family is and how important it is to be near them, especially when you're raising children." Moving is not for everyone. Almost every grandparent wants to be with his or her grandchildren and is willing to make sacrifices, but sometimes it is wiser to say no and visit frequently instead. Having your grandchildren far away is hard, especially knowing your adult child is struggling, but giving up the life you know may be harder. 1.Why was Garza's move a success? A. It strengthened her family ties. B. It improved her living conditions. 80 C. It enabled her to make more friends. D. It helped her know more new places. 2.What was the reaction of the public to Mrs.Robinson's decision? A. 17% expressed their support for it. B. Few people responded sympathetically. C. 83% believed it had a bad influence. D. The majority thought it was a trend. 3.What did Crosby say about people in the 1960s? A. They were unsure of themselves. B. They were eager to raise more children. C. They wanted to live away from their parents. D. They had little respect for their grandparents. 4.What does the author suggest the grandparents do in the last paragraph? A. Make decisions in the best interests of their own. B. Ask their children to pay more visits to them. C. Sacrifice for their struggling children. D. Get to know themselves better. Passage 2 2016 全国卷Ⅱ,8 分词数:247 Reading can be a social activity. Think of the people who belong to book groups. They choose books to read and then meet to discuss them. Now, the website BookCrossing.com turns the page on the traditional idea of a book group. Members go on the site and register the books they own and would like to share. BookCrossing provides an identification number to stick inside the book. Then the person leaves it in a public place, hoping that the book will have an adventure, traveling far and wide with each new reader who finds it. Bruce Pederson, the managing director of BookCrossing, says, "The two things that change your life are the people you meet and books you read. BookCrossing combines both." Members leave books on park benches and buses, in train stations and coffee shops. Whoever finds their book will go to the site and record where they found it. People who find a book can also leave a journal entry describing what they thought of 81 it.E-mails are then sent to the BookCrossers to keep them updated about where their books have been found. Bruce Pederson says the idea is for people not to be selfish by keeping a book to gather dust on a shelf at home. BookCrossing is part of a trend among people who want to get back to the "real" and not the virtual( 虚 拟 ). The site now has more than one million members in more than one hundred thirty-five countries. 1.Why does the author mention book groups in the first paragraph? A. To explain what they are. B. To introduce BookCrossing. C. To stress the importance of reading. D. To encourage readers to share their ideas. 2.What does the underlined word "it" in Paragraph 2 refer to? A. The book. B. An adventure. C. A public place. D. The identification number. 3.What will a BookCrosser do with a book after reading it? A. Meet other readers to discuss it. B. Keep it safe in his bookcase. C. Pass it on to another reader. D. Mail it back to its owner. 4.What is the best title for the text? A. Online Reading:A Virtual Tour B. Electronic Books:A New Trend C. A Book Group Brings Tradition Back D. A Website Links People through Books Passage 3 2016 北京,8 分词数:425 Why College Is Not Home The college years are supposed to be a time for important growth in autonomy(自主性) and the development of adult identity. However, now they are becoming an extended period of adolescence, during which many of today's students are not shouldered with adult responsibilities. For previous generations, college was a decisive break from parental control; guidance and support needed to come from people of the same age and from within. In the past two decades, however, continued connection with and dependence on family, thanks to cell phones, email and social media, have increased significantly. Some parents go so far as to help with coursework. Instead of promoting the idea of college as a passage from the shelter of the family to autonomy 82 and adult responsibility, universities have given in to the idea that they should provide the same environment as that of the home. To prepare for increased autonomy and responsibility, college needs to be a time of exploration and experimentation. This process involves "trying on" new ways of thinking about oneself both intellectually (在思维方面) and personally. While we should provide "safe spaces" within colleges, we must also make it safe to express opinions and challenge majority views. Intellectual growth and flexibility are fostered by strict debate and questioning. Learning to deal with the social world is equally important. Because a college community(群 体) differs from the family, many students will struggle to find a sense of belonging. If students rely on administrators to regulate their social behavior and thinking pattern, they are not facing the challenge of finding an identity within a larger and complex community. Moreover, the tendency for universities to monitor and shape student behavior runs up against another characteristic of young adults: the response to being controlled by their elders. If acceptable social behavior is too strictly defined( 规 定 ) and controlled, the insensitive or aggressive behavior that administrators are seeking to minimize may actually be encouraged. It is not surprising that young people are likely to burst out, particularly when there are reasons to do so. Our generation once joined hands and stood firm at times of national emergency. What is lacking today is the conflict between adolescents' desire for autonomy and their understanding of an unsafe world. Therefore, there is the desire for their dorms to be replacement homes and not places to experience intellectual growth. Every college discussion about community values, social climate and behavior should include recognition of the developmental importance of student autonomy and self-regulation, of the necessary tension between safety and self-discovery. 1.What's the author's attitude toward continued parental guidance to college students? A. Sympathetic. B. Disapproving. C. Supportive. D. Neutral. 2.The underlined word "passage" in Paragraph 2 means____. A. change B. choice C. text D. extension 3.According to the author, what role should college play? A. To develop a shared identity among students. B. To define and regulate students' social behavior. 83 C. To provide a safe world without tension for students. D. To foster students' intellectual and personal development. 4.Which of the following shows the development of ideas in the passage? A. B. C. D. I:Introduction P:Point Sp:Sub-point(次要点) C:Conclusion Passage 4 2016 天津,12.5 分词数:394 Failure is probably the most exhausting experience a person ever has. There is nothing more tiring than not succeeding. We experience this tiredness in two ways: as start-up fatigue(疲惫) and performance fatigue. In the former case, we keep putting off a task because it is either too boring or too difficult. And the longer we delay it, the more tired we feel. Such start-up fatigue is very real, even if not actually physical, not something in our muscles and bones. The solution is obvious though perhaps not easy to apply: always handle the most difficult job first. Years ago, I was asked to write 102 essays on the great ideas of some famous authors. Applying my own rule, I determined to write them in alphabetical order( 按字母顺序) ,never letting myself leave out a tough idea. And I always started the day's work with the difficult task of essay-writing. Experience proved that the rule works. Performance fatigue is more difficult to handle. Though willing to get started, we cannot seem to do the job right. Its difficulties appear so great that, however hard we work, we fail again and again. In such a situation, I work as hard as I can — then let the unconscious take over. When planning Encyclopaedia Britannica(《大英百科全书》), I had to create a table of contents based on the topics of its articles. Nothing like this had ever been done before, and day after day I kept coming up with solutions, but none of them worked. My fatigue became almost 84 unbearable. One day, mentally exhausted, I wrote down all the reasons why this problem could not be solved. I tried to convince myself that the trouble was with the problem itself, not with me. Relieved, I sat back in an easy chair and fell asleep. An hour later, I woke up suddenly with the solution clearly in mind. In the weeks that followed, the solution which had come up in my unconscious mind proved correct at every step. Though I worked as hard as before, I felt no fatigue. Success was now as exciting as failure had been depressing. Human beings, I believe, must try to succeed. Success, then, means never feeling tired. 1.People with start-up fatigue are most likely to ____. A. delay tasks B. work hard C. seek help D. accept failure 2.What does the author recommend doing to prevent start-up fatigue? A. Writing essays in strict order. B. Building up physical strength. C. Leaving out the toughest ideas. D. Dealing with the hardest task first. 3.On what occasion does a person probably suffer from performance fatigue? A. Before starting a difficult task. B. When all the solutions fail. C. If the job is rather boring. D. After finding a way out. 4.According to the author, the unconscious mind may help us ____. A. ignore mental problems B. get some nice sleep C. gain complete relief D. find the right solution 5. What could be the best title for the passage? A. Success Is Built upon Failure B. How to Handle Performance Fatigue C. Getting over Fatigue: A Way to Success D. Fatigue: An Early Sign of Health Problems Passage 5 2016 浙江,10 分词数:415 "Did you hear what happened to Adam last Friday?" Lindsey whispers to Tori. With her eyes shining, Tori brags, "You bet I did. Sean told me two days ago." Who are Lindsey and Tori talking about? It just happens to be yours truly, Adam Freedman. I can tell you that what they are saying is (a) not nice and (b) not even true. Still, Lindsey and Tori 85 aren't very different from most students here at Linton High School, including me. Many of our conversations are gossip( 闲话). I have noticed three effects of gossip: it can hurt people, it can give gossipers a strange kind of satisfaction, and it can cause social pressures in a group. An important negative effect of gossip is that it can hurt the person being talked about. Usually, gossip spreads information about a topic — breakups, trouble at home, even dropping out — that a person would rather keep secret. The more embarrassing or shameful the secret is, the juicier the gossip it makes. Probably the worst type of gossip is the absolute lie. People often think of gossipers as harmless, but cruel lies can cause pain. If we know that gossip can be harmful, then why do so many of us do it? The answer lies in another effect of gossip: the satisfaction it gives us. Sharing the latest rumor( 传言) can make a person feel important because he or she knows something that others don't. Similarly, hearing the latest rumor can make a person feel like part of the "in group." In other words, gossip is satisfying because it gives people a sense of belonging or even superiority(优越感). Gossip also can have a third effect: it strengthens unwritten, unspoken rules about how people should act. Professor David Wilson explains that gossip is important in policing behaviors in a group. Translated into high school terms, this means that if everybody you hang around with is laughing at what John wore or what Jane said, then you can bet that wearing or saying something similar will get you the same kind of negative attention. The do's and don'ts conveyed through gossip will never show up in any student handbook. The effects of gossip vary depending on the situation. The next time you feel the urge to spread the latest news, think about why you want to gossip and what effects your "juicy story" might have. 1.The author uses a conversation at the beginning of the passage to ____. A. introduce a topic B. present an argument C. describe the characters D. clarify his writing purpose 2.An important negative effect of gossip is that it ____. A. breaks up relationships B. embarrasses the listener C. spreads information around D. causes unpleasant experiences 3.In the author's opinion, many people like to gossip because it ____. A. gives them a feeling of pleasure B. helps them to make more friends 86 C. makes them better at telling stories D. enables them to meet important people 4.Professor David Wilson thinks that gossip can ____. A. provide students with written rules B. help people watch their own behaviors C. force schools to improve student handbooks D. attract the police's attention to group behaviors 5.What advice does the author give in the passage? A. Never become a gossiper. B. Stay away from gossipers. C. Don't let gossip turn into lies. D. Think twice before you gossip. Passage 6 2015 安徽,8 分词数:304 Food serves as a form of communication in two fundamental ways. Sharing bread or other foods is a common human tradition that can promote unity and trust. Food can also have a specific meaning, and play a significant role in a family or culture's celebrations or traditions. The foods we eat — and when and how we eat them — are often unique to a particular culture or may even differ between rural (农村的) and urban areas within one country. Sharing bread, whether during a special occasion ( 时刻) or at the family dinner table, is a common symbol of togetherness. Many cultures also celebrate birthdays and marriages with cakes that are cut and shared among the guests. Early forms of cake were simply a kind of bread, so this tradition has its roots in the custom of sharing bread. Food also plays an important role in many New Year celebrations. In the southern United States, pieces of corn bread represent blocks of gold for prosperity ( 兴旺) in the New Year. In Greece, people share a special cake called vasilopita. A coin is put into the cake, which signifies (预示) success in the New Year for the person who receives it. Many cultures have ceremonies to celebrate the birth of a child, and food can play a significant role. In China, when a baby is one month old, families name and welcome their child in a celebration that includes giving red colored eggs to guests. In many cultures, round foods such as grapes, bread, and moon cakes are eaten at welcome celebrations to represent family unity. Nutrition is necessary for life, so it is not surprising that food is such an important part of different cultures around the world. 87 1. According to the passage, sharing bread ____. A. indicates a lack of food B. can help to develop unity C. is a custom unique to rural areas D. has its roots in birthday celebrations 2. What does the coin in vasilopita signify for its receiver in the New Year? A. Trust. B. Success. C. Health. D. Togetherness. 3. The author explains the role of food in celebrations by. A. using examples B. making comparisons C. analyzing causes D. describing processes 4. What is the passage mainly about? A. The custom of sharing food. B. The specific meaning of food. C. The role of food in ceremonies. D. The importance of food in culture. Passage 7 2015 广东,10 分词数:340 It was once common to regard Britain as a society with class distinction. Each class had unique characteristics. In recent years, many writers have begun to speak of the ‘decline of class' and ‘classless society’ in Britain. And in modern day consumer society everyone is considered to be middle class. But pronouncing the death of class is too early. A recent wide-ranging study of public opinion found 90 percent of people still placing themselves in a particular class; 73 percent agreed that class was still a vital part of British society; and 52 percent thought there were still sharp class differences. Thus, class may not be culturally and politically obvious, yet it remains an important part of British society. Britain seems to have a love of stratification. One unchanging aspect of a British person's class position is accent. The words a person speaks tell her or his class. A study of British accents during the 1970s found that a voice sounding like a BBC newsreader was viewed as the most attractive voice. Most people said this accent sounded‘educated’ and ‘soft’. The accents placed at the bottom in this study, on the other hand, were regional ( 地 区 的 ) city accents. These accents were seen as ‘common’ and ‘ugly’.However, a similar study of British accents in the US turned these results upside down and placed some regional accents as the most attractive and BBC English as the least. This suggests that British attitudes towards accent have deep roots and are based on class prejudice. 88 In recent years, however,young upper middle-class people in London, have begun to adopt some regional accents, in order to hide their class origins. This is an indication of class becoming unnoticed .However, the 1995 pop song ‘Common People’ puts forward the view that though a middle-class person may ‘want to live like common people’ they can never appreciate the reality of a working-class life. 1.A recent study of public opinion shows that in modern Britain ____. A. it is time to end class distinction B. most people belong to middle class C. it is easy to recognize a person's class D. people regard themselves socially different 2.The word stratification in Paragraph 3 is closest in meaning to ____. A. variety B. division C. authority D. qualification 3.The study in the US showed that BBC English was regarded as. A. regional B. educated C. prejudiced D. unattractive 4.British attitudes towards accent _____. A. have a long tradition B. are based on regional status C. are shared by the Americans D. have changed in recent years 5.What is the main idea of the passage? A. The middle class is expanding. B. A person's accent reflects his class. C. Class is a key part of British society. D. Each class has unique characteristics. Passage 8 2015 重庆,8 分词数:271 The values of artistic works, according to cultural relativism ( 相 对 主 义 ), are simply reflections of local social and economic conditions. Such a view, however, fails to explain the ability of some works of art to excite the human mind across cultures and through centuries. History has witnessed the endless productions of Shakespearean plays in every major language of the world. It is never rare to find that Mozart packs Japanese concert halls, as Japanese painter Hiroshige does Paris galleries. Unique works of this kind are different from today's popular art, even if they began as works of popular art. They have set themselves apart in their timeless appeal and will probably be enjoyed for centuries into the future. In a 1757 essay, the philosopher David Hume argued that because "the general principles of 89 taste are uniform ( 不变的) in human nature," the value of some works of art might be essentially permanent. He observed that Homer was still admired after two thousand years. Works of this type, he believed, spoke to deep and unvarying features of human nature and could continue to exist over centuries. Now researchers are applying scientific methods to the study of the universality of art. For example, evolutionary psychology is being used by literary scholars to explain the long-lasting themes and plot devices in fiction. The structures of musical pieces are now open to experimental analysis as never before. Research findings seem to indicate that the creation by a great artist is as permanent an achievement as the discovery by a great scientist. 1.According to the passage, what do we know about cultural relativism? A. It introduces different cultural values. B. It explains the history of artistic works. C. It relates artistic values to local conditions. D. It excites the human mind throughout the world. 2.In Paragraph 2,the artists are mentioned in order to show that ____. A. great works of art can go beyond national boundaries B. history gives art works special appeal to set them apart C. popular arts are hardly distinguishable from great arts D. great artists are skilled at combining various cultures 3.According to Hume,some works of art can exist for centuries because ____. A. they are results of scientific study B. they establish some general principles of art C. they are created by the world's greatest artists D. they appeal to unchanging features of human nature 4.Which of the following can best serve as the title of the passage? A. Are Artistic Values Universal? B. Are Popular Arts Permanent? C. Is Human Nature Uniform? D. Is Cultural Relativism Scientific? Passage 9 2014 新课标全国Ⅰ,8 分词数:281 As more and more people speak the global languages of English,Chinese,Spanish,and Arabic,other languages are rapidly disappearing.In fact,half of the 6,000 — 7,000 languages 90 spoken around the world today will likely die out by the next century, according to the United Nations Educational,Scientific, and Cultural Organization(UNESCO). In an effort to prevent language loss,scholars from a number of organizations — UNESCO and National Geographic among them — have for many years been documenting dying languages and the cultures they reflect. Mark Turin,a scientist at the Macmillan Center,Yale University, who specializes in the languages and oral traditions of the Himalayas, is following in that tradition. His recently published book, A Grammar of Thangmi with an Ethnolinguistic Introduction to the Speakers and Their Culture, grows out of his experience living, working,and raising a family in a village in Nepal. Documenting the Thangmi language and culture is just a starting point for Turin,who seeks to include other languages and oral traditions across the Himalayan reaches of India, Nepal, Bhutan, and China. But he is not content to simply record these voices before they disappear without record. At the University of Cambridge Turin discovered a wealth of important materials — including photographs,films,tape recordings, and field notes — which had remained unstudied and were badly in need of care and protection. Now, through the two organizations that he has founded — the Digital Himalaya Project and the World Oral Literature Project — Turin has started a campaign to make such documents, found in libraries and stores around the world,available not just to scholars but to the younger generations of communities from whom the materials were originally collected.Thanks to digital technology and the widely available Internet,Turin notes,the endangered languages can be saved and reconnected with speech communities. 1.Many scholars are making efforts to ____. A. promote global languages B. rescue disappearing languages C. search for language communities D. set up language research organizations 2.What does "that tradition" in Paragraph 3 refer to? A. Having full records of the languages. B. Writing books on language teaching. C. Telling stories about language users. D. Living with the native speakers. 3.What is Turin's book based on? 91 A. The cultural studies in India. B. The documents available at Yale. C. His language research in Bhutan. D. His personal experience in Nepal. 4.Which of the following best describes Turin's work? A. Write, sell and donate. B. Record, repair and reward. C. Collect, protect and reconnect. D. Design, experiment and report. Passage 10 2014 新课标全国Ⅱ,6 分词数:274 One of the latest trends ( 趋 势 ) in American childcare is Chinese au pairs. Au Pair in Stamford, Conn., for example, has got increasing numbers of requests for Chinese au pairs from zero to around 4,000 since 2004. And that's true all across the country. "I thought it would be very useful for him to learn Chinese at an early age," Joseph Stocke, the managing director of a company, says of his 2-year-old son. "I would at least like to give him the chance to use the language in the future." After only six months of being cared for by a 25-year-old woman from China, the boy can already understand basic Chinese daily expressions, his dad says. Li Drake, a Chinese native raising two children in Minnesota with an American husband, had another reason for looking for an au pair from China: She didn't want her children to miss out on their roots. "Because I am Chinese, my husband and I wanted the children to keep exposed to (接 触) the language and culture," she says. "Staying with a native speaker is better for children than simply sitting in a classroom," says Suzanne Flynn, a professor in language education of children. "But parents must understand that just one year with an au pair is unlikely to produce wonders. Complete mastery demands continued learning until the age of 10 or 12." The popularity of au pairs from China has been strengthened by the increasing numbers of American parents who want their children to learn Chinese. It is expected that American demand for au pairs will continue to rise in the next few years. 1.What does the term "au pair" in the text mean? A. A mother raising her children on her own. B. A child learning a foreign language at home. C. A professor in language education of children. D. A young foreign woman taking care of children. 92 2.Li Drake has her children study Chinese because she wants them . A. to live in China some day B. to speak the language at home C. to catch up with other children D. to learn about the Chinese culture 3.What can we infer from the text? A. Learning Chinese is becoming popular in America. B. Educated women do better in looking after children. C. Chinese au pairs need to improve their English skills. D. Children can learn a foreign language well in six months. Passage 11 2013 辽宁,6 分词数:288 "Indeed," George Washington wrote in his diary in 1785, "some kind of fly, or bug, had begun to eat the leaves before I left home." But the father of America was not the father of bug.When Washington wrote that, Englishmen had been referring to insects as bugs for more than a century,and Americans had already created lightning-bug (萤火虫). But the English were soon to stop using the bugs in their language, leaving it to the Americans to call a bug a bug in the nineteenth and twentieth centuries. The American bug could also be a person, referring to someone who was crazy about a particular activity. Although fan became the usual term,sports fans used to be called racing bugs, baseball bugs, and the like. Or the bug could be a small machine or object, for example, a bug-shaped car.The bug could also be a burglar alarm, from which comes the expression to bug, that is, "to install (安装) an alarm".Now it means a small piece of equipment that people use for listening secretly to others' conversations.Since the 1840s, to bug has long meant "to cheat", and since the 1940s it has been annoying. We also know the bug as a flaw in a computer program or other design. That meaning dates back to the time of Thomas Edison.In 1878 he explained bugs as "little problems and difficulties" that required months of study and labor to overcome in developing a successful product.In 1889 it was recorded that Edison "had been up the two previous nights discovering ‘a bug’ in his invented record player." 1.We learn from Paragraph 1 that ____. A. Americans had difficulty in learning to use the word bug 93 B. George Washington was the first person to call an insect a bug C. the word bug was still popularly used in England in the nineteenth century D. both Englishmen and Americans used the word bug in the eighteenth century 2.What does the word "flaw"in the last paragraph probably mean? A. Explanation B.Finding C. Origin D.Fault 3.The passage is mainly concerned with ____. A. the misunderstanding of the word bug B. the development of the word bug C. the public views of the word bug D. the special characteristics of the word bug Passage 12 2012 安徽,8 分词数:262 Why is pink or purple a color for girls and blue or brown for boys? The answer depends largely on cultural values as well as personal experiences. To the Egyptians, green was a color that represented the hope and joy of spring, while for Muslims, it means heaven. Red is a symbol of good luck in many cultures. In China, children are given money in a red envelope to bring good fortune in the New Year. For many nations, blue is a symbol of protection and religious beliefs. Greek people often wear a blue necklace hoping to protect themselves against evils(灾祸). People's choice of colors is also influenced by their bodies' reactions( 反应) toward them. Green is said to be the most restful color. It has the ability to reduce pain and relax people both mentally and physically. People who work in green environments have been found to have fewer stomach aches. Red can cause a person's blood pressure to rise and increase people's appetites(食欲). Many decorators will include different shades of red in the restaurant. Similarly, many commercial websites will have a red "Buy Now" button because red is a color that easily catches a person's eye. Blue is another calming color. Unlike red, blue can cause people to lose appetite. So if you want to eat less, some suggest that eating from blue plates can help.The next time you are deciding on what to wear or what color to decorate your room, think about the color carefully. 1.Muslims regard green as a symbol of heaven mainly because of their ____. A. cultural values B. commercial purposes C. personal experiences D. physical reactions to the color 94 2.Why will many commercial websites have a red "Buy Now" button? A. To relax people physically. B. To increase people's appetites. C. To encourage people to make a purchase. D. To cause a person's blood pressure to rise. 3.What color might help lose weight according to the text? A. Red. B. Green. C. Blue. D. Purple. 4.Which of the following would be the most proper title for the text? A. Colors and Human Beings B. The Cultural Meaning of Color C. Colors and Personal Experiences D. The Meaning and Function of Color 专题五 时事新闻类 2018 年 Passage 2018 全国 III,8 分 话题:中国传统建筑艺术 词数:361 While famous foreign architects are invited to lead the designs of landmark buildings in China such as the new CCTV tower and the National Center for the Performing Arts, many excellent Chinese architects are making great efforts to take the center stage. Their efforts have been proven fruitful. Wang Shu, a 49-year-old Chinese architect, won the 2012 Pritzker Architecture Prize — which is often referred to as the Nobel Prize in architecture — on February 28. He is the first Chinese citizen to win this award. Wang serves as head of the Architecture Department at the China Academy of Art(CAA).His office is located at the Xiangshan campus(校园) of the university in Hangzhou, Zhejiang Province. Many buildings on the campus are his original creations. The style of the campus is quite different from that of most Chinese universities. Many visitors were amazed by the complex architectural space and abundant building types. The curves(曲线) of the buildings perfectly match the rise and fall of hills, forming a unique view. Wang collected more than 7 million abandoned bricks of different ages. He asked the workers to use traditional techniques to make the bricks into walls, roofs and corridors. This creation attracted a lot of attention thanks to its mixture of modern and traditional Chinese elements(元素). Wang’s works show a deep understanding of modern architecture and a good knowledge of traditions. Through such a balance, he had created a new type of Chinese architecture, said Tadao Ando, the winner of the 1995 Pritzker Prize. Wang believes traditions should not be sealed in glass boxes at museums. "That is only evidence that traditions once existed," he said. "Many Chinese people have a misunderstanding of traditions. They think tradition means old things from the past. In fact, tradition also refers to the things that have been developing and that are still being created," he said. "Today, many Chinese people are learning Western styles and theories rather than focusing on Chinese traditions. Many people tend to talk about traditions without knowing what they really are," said Wang. The study of traditions should be combined with practice. Otherwise, the recreation of traditions would be artificial and empty, he said. 95 1.Wang’s winning of the prize means that Chinese architects are . A. following the latest world trend B. getting international recognition C. working harder than ever before D. relying on foreign architects 2.What impressed visitors to the CAA Xiangshan campus most? A. Its hilly environment. B. Its large size. C. Its unique style. D. Its diverse functions. 3.What made Wang’s architectural design a success? A. The mixture of different shapes. B. The balance of East and West. C. The use of popular techniques. D. The harmony of old and new. 4.What should we do about Chinese traditions according to Wang? A. Spread them to the world. B. Preserve them at museums. C. Teach them in universities. D. Recreate them in practice. 2017 年 Passage 2017 全国卷Ⅱ,8 分 话题:飞行汽车 词数:317 Terrafugia Inc. said Monday that its new flying car has completed its first flight, bringing the company closer to its goal of selling the flying car within the next year. The vehicle — named the Transition — has two seats, four wheels and wings that fold up so it can be driven like a car. The Transition, which flew at 1,400 feet for eight minutes last month, can reach around 70 miles per hour on the road and 115 in the air. It flies using a 23-gallon tank of gas and burns 5 gallons per hour in the air. On the ground, it gets 35 miles per gallon. Around 100 people have already put down a ﹩10,000 deposit to get a Transition when they go on sale, and those numbers will likely rise after Terrafugia introduces the Transition to the public later this week at the New York Auto Show. But don ’ t expect it to show up in too many driveways. It’s expected to cost ﹩279,000. And it won’t help if you’re stuck in traffic. The car needs a runway. Inventors have been trying to make flying cars since the 1930s, according to Robert Mann, an airline industry expert. But Mann thinks Terrafugia has come closer than anyone to making the 96 flying car a reality. The government has already permitted the company to use special materials to make it easier for the vehicle to fly. The Transition is now going through crash tests to make sure it meets federal safety standards. Mann said Terrafugia was helped by the Federal Aviation Administration’s decision five years ago to create a separate set of standards for light sport aircraft, which are lower than those for pilots of larger planes. Terrafugia says an owner would need to pass a test and complete 20 hours of flying time to be able to fly the Transition, a requirement pilots would find relatively easy to meet. 1.What is the first paragraph mainly about? A. The basic data of the Transition. B. The advantages of flying cars. C. The potential market for flying cars. D. The designers of the Transition. 2.Why is the Transition unlikely to show up in too many driveways? A. It causes traffic jams. B. It is difficult to operate. C. It is very expensive. D. It burns too much fuel. 3.What is the government’s attitude to the development of the flying car? A. Cautious. B. Favorable. C. Ambiguous. D. Disapproving. 4.What is the best title for the text? A. Flying Car at Auto Show B. The Transition’s First Flight C. Pilots’ Dream Coming True D. Flying Car Closer to Reality 2012—2016 年 Passage 1 2016 北京,8 分词数:373 97 Surviving Hurricane Sandy(飓风桑迪) Natalie Doan, 14, has always felt lucky to live in Rockaway, New York. Living just a few blocks from the beach. Natalie can see the ocean and hear the waves from her house. "It's the ocean that makes Rockaway so special," she says. On October 29, 2012, that ocean turned fierce. That night, Hurricane Sandy attacked the East Coast, and Rockaway was hit especially hard. Fortunately, Natalie's family escaped to Brooklyn shortly before the city's bridges closed. When they returned to Rockaway the next day, they found their neighborhood in ruins. Many of Natalie's friends had lost their homes and were living far away. All around her, people were suffering, especially the elderly. Natalie's school was so damaged that she had to temporarily attend a school in Brooklyn. In the following few days, the men and women helping Rockaway recover inspired Natalie. Volunteers came with carloads of donated clothing and toys. Neighbors devoted their spare time to helping others rebuild. Teenagers climbed dozens of flights of stairs to deliver water and food to elderly people trapped in powerless high-rise buildings. "My mom tells me that I can't control what happens to me," Natalie says, "but I can always choose how I deal with it." Natalie's choice was to help. She created a website page, matching survivors in need with donors who wanted to help. Natalie posted information about a boy named Patrick, who lost his baseball card collection when his house burned down. Within days, Patrick's collection was replaced. In the coming months, her website page helped lots of kids: Christopher, who received a new basketball; Charlie, who got a new keyboard. Natalie also worked with other organizations to bring much-needed supplies to Rockaway. Her efforts made her a famous person. Last April, she was invited to the White House and honored as a Hurricane Sandy Champion of Change. Today, the scars(创痕) of destruction are still seen in Rockaway, but hope is in the air. The streets are clear, and many homes have been rebuilt. "I can't imagine living anywhere but Rockaway," Natalie declares. "My neighborhood will be back, even stronger than before." 1.When Natalie returned to Rockaway after the hurricane, she found ____. A. some friends had lost their lives B. her neighborhood was destroyed 98 C. her school had moved to Brooklyn D. the elderly were free from suffering 2. According to Paragraph 4, who inspired Natalie most? A. The people helping Rockaway rebuild. B. The people trapped in high-rise buildings. C. The volunteers donating money to survivors. D. Local teenagers bringing clothing to elderly people. 3.How did Natalie help the survivors? A. She gave her toys to other kids. B. She took care of younger children. C. She called on the White House to help. D. She built an information sharing platform. 4.What does the story intend to tell us? A. Little people can make a big difference. B. A friend in need is a friend indeed. C. East or west, home is best. D. Technology is power. Passage 2 2015 新课标全国Ⅱ,8 分词数:264 More students than ever before are taking a gap year(间隔年) before going to university. It used to be called the "year off" between school and university. The gap-year phenomenon originated( 起 源 ) with the months left over to Oxbridge applicants between entrance exams in November and the start of the next academic year. This year, 25,310 students who have accepted places in higher education institutions have put off their entry until next year, according to statistics on university entrance provided by the University and College Admissions Service(UCAS). That is a record 14.7% increase in the number of students taking a gap year. Tony Higgins from UCAS said that the statistics are good news for everyone in higher education. "Students who take a well-planned year out are more likely to be satisfied with, and complete, their chosen course. Students who take a gap year are often more mature and responsible," he said. But not everyone is happy. Owain James, the president of the National Union of Students (NUS), argued that the increase is evidence of student hardship — young people are being forced into earning money before finishing their education. "New students are now aware that they are likely to leave university up to £15,000 in debt. It is not surprising that more and more students are taking a gap year to earn money to support their study for the degree. NUS statistics show that over 40% of students are forced to work during term-time and the figure increases to 99 90% during vacation periods," he said. 1.What do we learn about the gap year from the text? A. It is flexible in length. B. It is a time for relaxation. C. It is increasingly popular. D. It is required by universities. 2.According to Tony Higgins, students taking a gap year ____. A. are better prepared for college studies B. know a lot more about their future jobs C. are more likely to leave university in debt D. have a better chance to enter top universities 3.How does Owain James feel about the gap-year phenomenon? A. He's puzzled. B. He's worried. C. He's surprised. D. He's annoyed. 4.What would most students do on their vacation according to NUS statistics? A. Attend additional courses. B. Make plans for the new term. C. Earn money for their education. D. Prepare for their graduate studies. Passage 3 2015 天津,12.5 分词数:322 Whether in the home or the workplace, social robots are going to become a lot more common in the next few years. Social robots are about to bring technology to the everyday world in a more humanized way, said Cynthia Breazeal, chief scientist at the robot company Jibo. While household robots today do the normal housework, social robots will be much more like companions than mere tools. For example, these robots will be able to distinguish when someone is happy or sad. This allows them to respond more appropriately to the user. The Jibo robot, arranged to ship later this year, is designed to be a personalized assistant. You can talk to the robot, ask it questions, and make requests for it to perform different tasks. The robot doesn't just deliver general answers to questions; it responds based on what it learns about each individual in the household. It can do things such as reminding an elderly family member to take medicine or taking family photos. Social robots are not just finding their way into the home. They have potential applications in everything from education to health care and are already finding their way into some of these spaces. Fellow Robots is one company bringing social robots to the market. The company's "Oshbot" robot is built to assist customers in a store, which can help the customers find items and help guide 100 them to the product's location in the store. It can also speak different languages and make recommendations for different items based on what the customer is shopping for. The more interaction the robot has with humans, the more it learns. But Oshbot, like other social robots, is not intended to replace workers, but to work alongside other employees. "We have technologies to train social robots to do things not for us, but with us," said Breazeal. 1.How are social robots different from household robots? A. They can control their emotions. B. They are more like humans. C. They do the normal housework. D. They respond to users more slowly. 2.What can a Jibo robot do according to Paragraph 3? A. Communicate with you and perform operations. B. Answer your questions and make requests. C. Take your family pictures and deliver milk. D. Obey your orders and remind you to take pills. 3.What can Oshbot work as? A. A language teacher. B. A tour guide. C. A shop assistant. D. A private nurse. 4.We can learn from the last paragraph that social robots will ____. A. train employees B. be our workmates C. improve technologies D. take the place of workers 5.What does the passage mainly present? A. A new design idea of household robots. B. Marketing strategies for social robots. C. Information on household robots. D. An introduction to social robots. Passage 4 2014 四川,8 分词数:303 A schoolgirl saved her father's life by kicking him in the chest after he suffered a serious allergic (过敏的) reaction which stopped his heart. Izzy, nine, restarted father Colm's heart by stamping ( 踩) on his chest after he fell down at home and stopped breathing. Izzy's mother, Debbie, immediately called 999 but Izzy knew doctors would never arrive in 101 time to save her father, so decided to use CPR. However, she quickly discovered her arms weren't strong enough, so she stamped on her father's chest instead. Debbie then took over with some more conventional chest compressions ( 按 压 ) until the ambulance arrived. Izzy, who has been given a bravery award by her school, said: "I just kicked him really hard. My mum taught me CPR but I knew I wasn't strong enough to use hands. I was quite scared. The doctor said I might as well be a doctor or a nurse. My mum said that Dad was going to hospital with a big footprint on his chest." "She's a little star," said Debbie. "I was really upset but Izzy just took over. I just can't believe what she did. I really think all children should be taught first aid. Izzy did CPR then the doctor turned up. Colm had to have more treatment on the way to the hospital and we've got to see an expert." Truck driver Colm, 35, suffered a mystery allergic reaction on Saturday and was taken to hospital, but was sent home only for it to happen again the next day. The second attack was so serious that his airway swelled, preventing him from breathing, his blood pressure dropped suddenly, and his heart stopped for a moment. He has now made a full recovery from his suffering. 1.Izzy kicked her father in the chest ____. A. to express her helplessness B. to practise CPR on him C. to keep him awake D. to restart his heart 2.What's the right order of the events? ①Izzy kicked Colm. ②Debbie called 999. ③Izzy learned CPR. ④Colm's heart stopped . A. ③①②④ B. ④②③① C. ③④②① D. ④③①② 3.What does Paragraph 8 mainly talk about? A. What Colm suffered. 102 B. Colm's present condition. C. What caused Colm's allergy. D. Symptoms of Colm's allergic reaction. 4.Why does the author write the news? A. To describe a serious accident. B. To prove the importance of CPR. C. To report a 9-year-old girl's brave act. D. To call people's attention to allergic reaction. Passage 5 2012 全国Ⅱ,8 分词数:215 ADDIS ABABA, Ethiopia — One of the world's most famous fossils ( 化 石 ) — the 3.2 million-year-old Lucy skeleton (骨骼) unearthed in Ethiopia in 1974 — will go on an exhibition tour abroad for the first time in the United States, officials said Tuesday. Even the Ethiopian public has only seen Lucy twice. The Lucy on exhibition at the Ethiopian National Museum in the capital, Addis Ababa, is a replica while the real remains are usually locked in a secret storeroom. A team from the Museum of National Science in Houston, Texas, spent four years discussing with the Ethiopians for the U.S. tour, which will start in Houston next September. "Ethiopia's rich culture of both the past and today, is one of the best kept secrets in the world," said Joel Bartsch, director of the Houston museum. The six-year tour will also go to Washington, New York, Denver and Chicago. Officials said six other U.S. cities may be on the tour. But they said plans had not yet been worked out. Traveling with Lucy will be 190 other fossils. Lucy, her name taken from a Beatles song that played in a camp the night of her discovery, is part of the skeleton of what was once a 3 1/2-foot-tall ape-man (猿人). 1.The author writes this text mainly to ____. A. introduce a few U.S. museums B. describe some research work C. discuss the value of an ape-man D. report a coming event 2.What do the words "a replica" in Paragraph 2 refer to? 103 A. A painting of the skeleton. B. A photograph of Lucy. C. A copy of the skeleton. D. A written record of Lucy. 3.How many cities has Lucy's U.S. tour plan already included? A. Four. B. Five. C. Six. D. Eleven. 4.What was the skeleton named after? A. An ape-man. B. A song. C. A singer. D. A camp. 专题六 人物故事类 2018 年 Passage 1 2018 北京,8 分 话题:马拉松比赛 词数:315 My First Marathon(马拉松) A month before my first marathon, one of my ankles was injured and this meant not running for two weeks, leaving me only two weeks to train. Yet, I was determined to go ahead. I remember back to my 7th year in school. In my first P.E. class, the teacher required us to run laps and then hit a softball. I didn’t do either well. He later informed me that I was "not athletic". The idea that I was "not athletic" stuck with me for years. When I started running in my 30s, I realized running was a battle against myself, not about competition or whether or not I was athletic. It was all about the battle against my own body and mind. A test of wills! The night before my marathon, I dreamt that I couldn’t even find the finish line. I woke up sweating and nervous, but ready to prove something to myself. Shortly after crossing the start line, my shoe laces( 鞋 带 ) became untied. So I stopped to readjust. Not the start I wanted! At mile 3, I passed a sign:"GO FOR IT,RUNNERS!" By mile 17, I became out of breath and the once injured ankle hurt badly. Despite the pain, I stayed the course walking a bit and then running again. By mile 21, I was starving! As I approached mile 23, I could see my wife waving a sign. She is my biggest fan. She never minded the alarm clock sounding at 4 a.m. or questioned my expenses on running. I was one of the final runners to finish. But I finished! And I got a medal. In fact, I got the same medal as the one that the guy who came in first place had. Determined to be myself, move forward, free of shame and worldly labels(世俗标签), I can now call myself a "marathon winner". 1.A month before the marathon, the author . A. was well trained B. felt scared C. made up his mind to run D. lost hope 2.Why did the author mention the P.E. class in his 7th year? 104 A. To acknowledge the support of his teacher. B. To amuse the readers with a funny story. C. To show he was not talented in sports. D. To share a precious memory. 3.How was the author’s first marathon? A. He made it. B. He quit halfway. C. He got the first prize. D. He walked to the end. 4.What does the story mainly tell us? A. A man owes his success to his family support. B. A winner is one with a great effort of will. C. Failure is the mother of success. D. One is never too old to learn. Passage 2 2018 浙江,6 分 话题:英国作家狄更斯 词数:314 In 1812, the year Charles Dickens was born, there were 66 novels published in Britain. People had been writing novels for a century — most experts date the first novel to Robinson Crusoe in 1719 — but nobody wanted to do it professionally. The steam-powered printing press was still in its early stages; the literacy( 识字)rate in England was under 50%. Many works of fiction appeared without the names of the authors, often with something like "By a lady." Novels, for the most part, were looked upon as silly, immoral or just plain bad.In 1870, when Dickens died, the world mourned him as its first professional writer and publisher, famous and beloved, who had led an explosion in both the publication of novels and their readership and whose characters — from Oliver Twist to Tiny Tim — were held up as moral touchstones. Today Dickens’ greatness is unchallenged. Removing him from the pantheon ( 名人堂) of English literature would make about as much sense as the Louvre selling off the Mona Lisa. How did Dickens get to the top? For all the feelings readers attach to stories, literature is a numbers game, and the test of time is extremely difficult to pass. Some 60,000 novels were published during the Victorian age, from 1837 to 1901; today a casual reader might be able to name a half-dozen of them. It’s partly true that Dickens’ style of writing attracted audiences from all walks of life. It’s partly that his writings rode a wave of social, political and scientific progress. But it’s also that he rewrote the culture of literature and put himself at the center. No one will ever know what mix of talent, ambition, energy and luck made Dickens such a distinguished writer. But as the 200th anniversary of his birth approaches, it is possible — and important for our own culture — to understand how he made himself a lasting one. 1.Which of the following best describes British novels in the 18th century? A. They were difficult to understand. B. They were popular among the rich. C. They were seen as nearly worthless. D. They were written mostly by women. 2.Dickens is compared with the Mona Lisa in the text to stress. A. his reputation in France B. his interest in modern art C. his success in publication D. his importance in literature 105 3.What is the author’s purpose in writing the text? A. To remember a great writer. B. To introduce an English novel. C. To encourage studies on culture. D. To promote values of the Victorian age. 2017 年 Passage 1 2017 全国卷Ⅱ,8 分 话题:与 Paul Newman 的友谊 词数:283 I first met Paul Newman in 1968, when George Roy Hill, the director of Butch Cassidy and the Sundance Kid, introduced us in New York City. When the studio didn’t want me for the film — it wanted somebody as well known as Paul — he stood up for me. I don’t know how many people would have done that; they would have listened to their agents or the studio powers. The friendship that grew out of the experience of making that film and The Sting four years later had its root in the fact that although there was an age difference, we both came from a tradition of theater and live TV. We were respectful of craft(技艺) and focused on digging into the characters we were going to play. Both of us had the qualities and virtues that are typical of American actors: humorous, aggressive, and making fun of each other — but always with an underlying affection. Those were also at the core(核心) of our relationship off the screen. We shared the belief that if you’re fortunate enough to have success, you should put something back — he with his Newman’s Own food and his Hole in the Wall camps for kids who are seriously ill, and me with Sundance and the institute and the festival. Paul and I didn’t see each other all that regularly, but sharing that brought us together. We supported each other financially and by showing up at events. I last saw him a few months ago. He’d been in and out of the hospital. He and I both knew what the deal was, and we didn’t talk about it. Ours was a relationship that didn’t need a lot of words. 1.Why was the studio unwilling to give the role to the author at first? A. Paul Newman wanted it. B. The studio powers didn’t like his agent. C. He wasn’t famous enough. 106 D. The director recommended someone else. 2.Why did Paul and the author have a lasting friendship? A. They were of the same age.B. They worked in the same theater. C. They were both good actors. D. They had similar characteristics. 3.What does the underlined word "that" in paragraph 3 refer to? A. Their belief.B. Their care for children. C. Their success.D. Their support for each other. 4.What is the author’s purpose in writing the text? A. To show his love of films. B. To remember a friend. C. To introduce a new movie. D. To share his acting experience. Passage 2 2017 北京,6 分 话题:利用急救知识救人 词数:339 It was a cold March day in High Point, North Carolina. The girls on the Wesleyan Academy softball team were waiting for their next turns at bat during practice, stamping their feet to stay warm. Eighth grader Taylor Bisbee shivered( 发抖) a little as she watched her teammate Paris White play. The two didn’t know each other well — Taylor had just moved to town a month or so before. Suddenly, Paris fell to the ground. "Paris’s eyes rolled back," Taylor says. "She started shaking. I knew it was an emergency." It certainly was. Paris had suffered a sudden heart failure. Without immediate medical care, Paris would die. At first, no one moved. The girls were in shock. Then the softball coach shouted out, "Does anyone know CPR?" CPR is a life-saving technique. To do CPR, you press on the sick person ’s chest so that blood moves through the body and takes oxygen to organs. Without oxygen, the brain is damaged quickly. Amazingly, Taylor had just taken a CPR course the day before. Still, she hesitated.She didn ’t think she knew it well enough. But when no one else came forward, Taylor ran to Paris and began 107 doing CPR. "It was scary. I knew it was the difference between life and death," says Taylor. Taylor’s swift action helped her teammates calm down. One girl called 911. Two more ran to get the school nurse, who brought a defibrillator, an electronic device(器械) that can shock the heart back into work. Luck stayed with them: Paris’s heartbeat returned. "I know I was really lucky," Paris says now. "Most people don’t survive this. My team saved my life." Experts say Paris is right: For a sudden heart failure, the single best chance for survival is having someone nearby step in and do CPR quickly. Today, Paris is back on the softball team. Taylor will apply to college soon. She wants to be a nurse. "I feel more confident in my actions now," Taylor says. "I know I can act under pressure in a scary situation." 1.What happened to Paris on a March day? A. She caught a bad cold. B. She had a sudden heart problem. C. She was knocked down by a ball. D. She shivered terribly during practice. 2. Why does Paris say she was lucky? A. She made a worthy friend. B. She recovered from shock. C. She received immediate CPR. D. She came back on the softball team. 3. Which of the following words can best describe Taylor? A. Enthusiastic and kind. B. Courageous and calm. C. Cooperative and generous. D. Ambitious and professional. Passage 3 2017 浙江,10 分 话题:绘画之父本杰明 词数:318 Benjamin West, the father of American painting, showed his talent for art when he was only six years of age. But he did not know about brushes before a visitor told him he needed one. In those days, a brush was made from camel’s hair. There were no camels nearby. Benjamin decided that 108 cat hair would work instead. He cut some fur from the family cat to make a brush. The brush did not last long. Soon Benjamin needed more fur. Before long, the cat began to look ragged(蓬乱). His father said that the cat must be sick. Benjamin was forced to admit what he had been doing. The cat’s lot was about to improve. That year, one of Benjamin’s cousins, Mr. Pennington, came to visit. He was impressed with Benjamin’s drawings. When he went home, he sent Benjamin a box of paint and some brushes. He also sent six engravings(版画) by an artist. These were the first pictures and first real paint and brushes Benjamin had ever seen. In 1747, when Benjamin was nine years old, Mr.Pennington returned for another visit. He was amazed at what Benjamin had done with his gift. He asked Benjamin’s parents if he might take the boy back to Philadelphia for a visit. In the city, Mr.Pennington gave Benjamin materials for creating oil paintings. The boy began a landscape( 风 景 ) painting. William Williams, a well  known painter, came to see him work. Williams was impressed with Benjamin and gave him two classic books on painting to take home. The books were long and dull. Benjamin could read only a little, having been a poor student. But he later said, "Those two books were my companions by day, and under my pillow at night." While it is likely that he understood very little of the books, they were his introduction to classical paintings. The nine year old boy decided then that he would be an artist. 1.What is the text mainly about? A. Benjamin’s visit to Philadelphia. B. Williams’ influence on Benjamin. C. The beginning of Benjamin’s life as an artist. D. The friendship between Benjamin and Pennington. 2. What does the underlined sentence in paragraph 3 suggest? A. The cat would be closely watched. B. The cat would get some medical care. C. Benjamin would leave his home shortly. D. Benjamin would have real brushes soon. 3. What did Pennington do to help Benjamin develop his talent? A. He took him to see painting exhibitions. 109 B. He provided him with painting materials. C. He sent him to a school in Philadelphia. D. He taught him how to make engravings. 4.Williams’ two books helped Benjamin to . A. master the use of paints B. appreciate landscape paintings C. get to know other painters D. make up his mind to be a painter 2012—2016 年 Passage 1 2016 全国卷Ⅰ,6 分词数:266 I am Peter Hodes, a volunteer stem cell courier. Since March 2012, I've done 89 trips — of those, 51 have been abroad. I have 42 hours to carry stem cells (干细胞)in my little box because I've got two ice packs and that's how long they last. In all, from the time the stem cells are harvested from a donor( 捐献者) to the time they can be implanted in the patient, we've got 72 hours at most. So I am always conscious of time. I had one trip last year where I was caught by a hurricane in America. I picked up the stem cells in Providence, Rhode Island, and was meant to fly to Washington then back to London. But when I arrived at the check-in desk at Providence, the lady on the desk said:"Well, I'm really sorry, I've got some bad news for you — there are no flights from Washington." So I took my box and put it on the desk and I said:"In this box are some stem cells that are urgently needed for a patient — please, please, you've got to get me back to the United Kingdom." She just dropped everything. She arranged for a flight on a small plane to be held for me, re-routed(改道)me through Newark and got me back to the UK even earlier than originally scheduled. For this courier job, you're consciously aware that in that box you've got something that is potentially going to save somebody's life. 1.Which of the following can replace the underlined word "courier" in Paragraph 1? A. provider B. delivery man C. collector D. medical doctor 2.Why does Peter have to complete his trip within 42 hours? A. He cannot stay away from his job too long. B. The donor can only wait for that long. 110 C. The operation needs that much time. D. The ice won't last any longer. 3.Which flight did the woman put Peter on first? A. To London. B. To Newark. C. To Providence. D. To Washington. Passage 2 2016 全国卷Ⅲ,6 分词数:324 On one of her trips to New York several years ago, Eudora Welty decided to take a couple of New York friends out to dinner. They settled in at a comfortable East Side cafe and within minutes, another customer was approaching their table. "Hey, aren't you from Mississippi?" the elegant, white-haired writer remembered being asked by the stranger. "I'm from Mississippi too." Without a second thought, the woman joined the Welty party. When her dinner partner showed up, she also pulled up a chair. "They began telling me all the news of Mississippi," Welty said. "I didn't know what my New York friends were thinking." Taxis on a rainy New York night are rarer than sunshine. By the time the group got up to leave, it was pouring outside. Welty's new friends immediately sent a waiter to find a cab. Heading back downtown toward her hotel, her big-city friends were amazed at the turn of events that had changed their Big Apple dinner into a Mississippi state reunion(团聚). "My friend said: ‘Now we believe your stories,’" Welty added. "And I said:‘Now you know. These are the people that make me write them.’" Sitting on a sofa in her room, Welty, a slim figure in a simple gray dress, looked pleased with this explanation. "I don't make them up," she said of the characters in her fiction these last 50 or so years."I don't have to." Beauticians, bartenders, piano players and people with purple hats, Welty's people come from afternoons spent visiting with old friends, from walks through the streets of her native Jackson, Miss., from conversations overheard on a bus. It annoys Welty that, at 78, her left ear has now given out. Sometimes, sitting on a bus or a train, she hears only a fragment(片断)of a particularly interesting story. 111 1.What happened when Welty was with her friends at the cafe? A. Two strangers joined her. B. Her childhood friends came in. C. A heavy rain ruined the dinner. D. Some people held a party there. 2.The underlined word "them" in Paragraph 6 refers to Welty's. A. readers B. parties C. friends D. stories 3.What can we learn about the characters in Welty's fiction? A. They live in big cities. B. They are mostly women. C. They come from real life. D. They are pleasure seekers. Passage 3 2016 北京,6 分词数:329 December 15,2014 Dear Alfred, I want to tell you how important your help is to my life. Growing up, I had people telling me I was too slow, though, with an IQ of 150+ at 17, I'm anything but stupid. The fact was that I was found to have ADHD( 注 意 力 缺 陷 多 动 障 碍 ). Anxious all the time, I was unable to keep focused for more than an hour at a time. However, when something did interest me, I could become absorbed. In high school, I became curious about the computer, and built my first website. Moreover, I completed the senior course of Computer Basics, plus five relevant pre-college courses. While I was exploring my curiosity, my disease got worse. I wanted to go to college after high school, but couldn't. So, I was killing my time at home until June 2012 when I discovered the online computer courses of your training center. Since then, I have taken courses like Data Science and Advanced Mathematics. Currently, I'm learning your Probability course. I have hundreds of printer paper, covered in self-written notes from your videos. This has given me a purpose. Last year, I spent all my time looking for a job where, without dealing with the public, I could work alone, but still have a team to talk to. Luckily, I discovered the job — Data Analyst — this month and have been going full steam ahead. I want to prove that I can teach myself a respectful profession, without going to college, and be just as good as, if not better than, my competitors. 112 Thank you. You've given me hope that I can follow my heart. For the first time, I feel good about myself because I'm doing something, not because someone told me I was doing good. I feel whole. This is why you're saving my life. Yours, Tanis 1.Why didn't Tanis go to college after high school? A. She had learned enough about computer science. B. She had more difficulty keeping focused. C. She preferred taking online courses. D. She was too slow to learn. 2.As for the working environment, Tanis prefers ____. A. working by herself B. dealing with the public C. competing against others D. staying with ADHD students 3.Tanis wrote this letter in order to ____. A. explain why she was interested in the computer B. share the ideas she had for her profession C. show how grateful she was to the center D. describe the courses she had taken so far Passage 4 2016 江苏,12 分词数:699 Not so long ago, most people didn't know who Shelly-Ann Fraser-Pryce was going to become. She was just an average high athlete. There was every indication that she was just another Jamaican teenager without much of a future. However, one person wanted to change this. Stephen Francis observed then eighteen-year-old Shelly-Ann at a track meet and was convinced that he had seen the beginnings of true greatness. Her times were not exactly impressive, but even so, he sensed there was something trying to get out, something the other coaches had overlooked when they had assessed her and found her lacking. He decided to offer Shelly-Ann a place in his very strict training sessions. Their cooperation quickly produced results, and a few years later at 113 Jamaica's Olympic trials in early 2008, Shelly-Ann, who at that time only ranked number 70 in the world, beat Jamaica's unchallenged queen of the sprint (短跑). "Where did she come from?" asked an astonished sprinting world, before concluding that she must be one of those one-hit wonders that spring up from time to time, only to disappear again without signs. But Shelly-Ann was to prove that she was anything but a one-hit wonder. At the Beijing Olympics she swept away any doubts about her ability to perform consistently by becoming the first Jamaican woman ever to win the 100 metres Olympic gold. She did it again one year on at the World Championships in Berlin, becoming world champion with a time of 10.73 — the fourth fastest time ever. Shelly-Ann is a little woman with a big smile. She has a mental toughness that did not come about by chance. Her journey to becoming the fastest woman on earth has been anything but smooth and effortless. She grew up in one of Jamaica's toughest inner-city communities known as Waterhouse, where she lived in a one-room apartment, sleeping four in a bed with her mother and two brothers. Waterhouse, one of the poorest communities in Jamaica, is a really violent and overpopulated place. Several of Shelly-Ann's friends and family were caught up in the killings; one of her cousins was shot dead only a few streets away from where she lived. Sometimes her family didn't have enough to eat. She ran at the school championships barefooted because she couldn't afford shoes. Her mother Maxime, one of a family of fourteen, had been an athlete herself as a young girl but, like so many other girls in Waterhouse, had to stop after she had her first baby. Maxime's early entry into the adult world with its responsibilities gave her the determination to ensure that her kids would not end up in Waterhouse's roundabout of poverty. One of the first things Maxime used to do with Shelly-Ann was taking her to the track, and she was ready to sacrifice everything. It didn't take long for Shelly-Ann to realize that sports could be her way out of Waterhouse. On a summer evening in Beijing in 2008, all those long, hard hours of work and commitment finally bore fruit. The barefoot kid who just a few years previously had been living in poverty, surrounded by criminals and violence, had written a new chapter in the history of sports. But Shelly-Ann's victory was far greater than that. The night she won Olympic gold in Beijing, the routine murders in Waterhouse and the drug wars in the neighbouring streets stopped. The dark cloud above one of the world's toughest criminal neighbourhoods simply disappeared for 114 a few days. "I have so much fire burning for my country," Shelly said. She plans to start a foundation for homeless children and wants to build a community centre in Waterhouse. She hopes to inspire the Jamaicans to lay down their weapons. She intends to fight to make it a woman's as well as a man's world. As Muhammad Ali puts it, "Champions aren't made in gyms. Champions are made from something they have deep inside them. A desire, a dream, a vision." One of the things Shelly-Ann can be proud of is her understanding of this truth. 1.Why did Stephen Francis decide to coach Shelly-Ann? A. He had a strong desire to free her family from trouble. B. He sensed a great potential in her despite her weaknesses. C. She had big problems maintaining her performance. D. She suffered a lot of defeats at the previous track meets. 2.What did the sprinting world think of Shelly-Ann before the 2008 Olympic Games? A. She would become a promising star. B. She badly needed to set higher goals. C. Her sprinting career would not last long. D. Her talent for sprinting was known to all. 3.What made Maxime decide to train her daughter on the track? A. Her success and lessons in her career. B. Her interest in Shelly-Ann's quick profit. C. Her wish to get Shelly-Ann out of poverty. D. Her early entrance into the sprinting world. 4.What can we infer from Shelly-Ann's statement underlined in Paragraph 5? A. She was highly rewarded for her efforts. B. She was eager to do more for her country. C. She became an athletic star in her country. D. She was the envy of the whole community. 5.By mentioning Muhammad Ali's words, the author intends to tell us that ____. A. players should be highly inspired by coaches B. great athletes need to concentrate on patience 115 C. hard work is necessary in one's achievements D. motivation allows great athletes to be on the top 6.What is the best title for the passage? A. The Making of a Great Athlete B. The Dream for Championship C. The Key to High Performance D. The Power of Full Responsibility Passage 5 2016 浙江,12 分词数:647 Two things changed my life: my mother and a white plastic bike basket. I have thought long and hard about it and it's true. I would be a different person if my mom hadn't turned a silly bicycle accessory into a life lesson I carry with me today. My mother and father were united in their way of raising children, but it mostly fell to my mother to actually carry it out. Looking back, I honestly don't know how she did it. Managing the family budget must have been a very hard task, but she made it look effortless. If we complained about not having what another kid did, we'd hear something like, "I don't care what so and so got for his birthday, you are not getting a TV in your room/a car for your birthday/a lavish sweet 16 party." We had to earn our allowance( 零 用 钱 ) by doing chores around the house. I can still remember how long it took to polish the legs of our coffee table. My brothers can no doubt remember hours spent cleaning the house. Like the two little girls growing up at the White House, we made our own beds (no one left the house until that was done) and picked up after ourselves. We had to keep track of our belongings, and if something was lost, it was not replaced. It was summer and, one day, my mother drove me to the bike shop to get a tire fixed — and there it was in the window. White, shiny, plastic and decorated with flowers, the basket winked at me and I knew — I knew — I had to have it. "It's beautiful," my mother said when I pointed it out to her. "What a neat basket." I tried to hold off at first. I played it cool for a short while. But then I guess I couldn't stand it any longer: "Mom, please can I please, please get it? I'll do extra chores for as long as you say. I'll do anything, but I need that basket. I love that basket. Please, Mom. Please?" I was desperate. "You know," she said, gently rubbing my back while we both stared at what I believed was 116 the coolest thing ever, "If you save up you could buy this yourself." "By the time I make enough it'll be gone!" "Maybe Roger here could hold it for you," she smiled at Roger, the bike guy. "He can't hold it for that long, Mom. Someone else will buy it. Please, Mom, please?""There might be another way," she said. And so our paying plan unfolded. My mother bought the beautiful basket and put it safely in some hiding place I couldn't find. Each week I eagerly counted my growing savings increased by extra work here and there (washing the car, helping my mother make dinner, delivering or collecting things on my bike that already looked naked without the basket in front). And then, weeks later, I counted, re-counted and jumped for joy. Oh, happy day! I made it! I finally had the exact amount we'd agreed upon.... Days later the unthinkable happened. A neighborhood girl I'd played with millions of times appeared with the exact same basket fixed to her shiny, new bike that already had all the bells and whistles. I rode hard and fast home to tell my mother about this disaster. This horrible turn of events. And then came the lesson I've taken with me through my life:"Honey, your basket is extra-special," Mom said, gently wiping away my hot tears. "Your basket is special because you paid for it yourself." 1.What can we learn from the first two paragraphs? A. The children enjoyed doing housework. B. The author came from a well-off family. C. The mother raised her children in an unusual way. D. The children were fond of the US president's daughters. 2.When the author saw the basket in the window, she ____ . A. fell in love with it B. stared at her mother C. recognized it at once D. went up to the bike guy 3.Why did the author say many "pleases" to her mother? A. She longed to do extra work. B. She was eager to have the basket. C. She felt tired after standing too long. D. She wanted to be polite to her mother. 4.By using "naked" (Paragraph 12), the author seems to stress that the basket was ____. 117 A. something she could afford B. something important to her C. something impossible to get D. something she could do without 5.To the author, it seemed to be a horrible turn of events that ____. A. something spoiled her paying plan B. the basket cost more than she had saved C. a neighborhood girl had bought a new bike D. someone else had got a basket of the same kind 6.What is the life lesson the author learned from her mother? A. Save money for a rainy day. B. Good advice is beyond all price. C. Earn your bread with your sweat. D. God helps those who help themselves. Passage 6 2015 新课标全国Ⅰ,8 分词数:232 Salvador Dali (1904—1989)was one of the most popular of modern artists. The Pompidou Centre in Paris is showing its respect and admiration for the artist and his powerful personality with an exhibition bringing together over 200 paintings, sculptures, drawings and more. Among the works and masterworks on exhibition the visitor will find the best pieces, most importantly The Persistence of Memory.There is also L'Enigme sans Fin from 1938, works on paper, objects, and projects for stage and screen and selected parts from television programmes reflecting the artist's showman qualities. The visitor will enter the World of Dali through an egg and is met with the beginning, the world of birth. The exhibition follows a path of time and subject with the visitor exiting through the brain. The exhibition shows how Dali draws the viewer between two infinities (无限), "From the infinity small to the infinity large, contraction and expansion coming in and out of focus: amazing Flemish accuracy and the showy Baroque of old painting that he used in his museum-theatre in Figueras," explains the Pompidou Centre. The fine selection of the major works was done in close collaboration ( 合作) with the Museo Nacional Reina Sofia in Madrid, Spain, and with contributions from other institutions like the Salvador Dali Museum in St.Petersburg, Florida. 118 1.Which of the following best describes Dali according to Paragraph 1? A. Optimistic. B. Productive. C. Generous. D. Traditional. 2.What is Dali's The Persistence of Memory considered to be? A. One of his masterworks. B. A successful screen adaptation. C. An artistic creation for the stage. D. One of the best TV programmes. 3.How are the exhibits arranged at the World of Dali? A. By popularity. B. By importance. C. By size and shape. D. By time and subject. 4.What does the word "contributions" in the last paragraph refer to? A. Artworks. B. Projects. C. Donations. D. Documents. Passage 7 2015 湖北,8 分词数:330 Science has a lot of uses.It can uncover laws of nature,cure diseases,make bombs,and help bridges to stand up.Indeed science is so good at what it does that there's always a temptation (诱惑) to drag it into problems where it may not be helpful.David Brooks,author of The Social Animal:The Hidden Sources of Love, Character, and Achievement, appears to be the latest in a long line of writers who have failed to resist the temptation. Brooks gained fame for several books.His latest book The Social Animal,however,is more ambitious and serious than his earlier books.It is an attempt to deal with a set of weighty topics.The book focuses on big questions:What has science revealed about human nature? What are the sources of character? And why are some people happy and successful while others aren't? To answer these questions,Brooks surveys a wide range of disciplines ( 学科).Considering this,you might expect the book to be a dry recitation of facts.But Brooks has structured his book in an unorthodox ( 非 常 规 的 ) , and perhaps unfortunate,way.Instead of introducing scientific theories,he tells a story,within which he tries to make his points,perhaps in order to keep the reader's attention. So as Harold and Erica,the hero and heroine in his story, live through childhood,we hear about the science of child development,and as they begin to date we hear about 119 the theory of sexual attraction.Brooks carries this through to the death of one of his characters. On the whole,Brooks's story is acceptable if uninspired.As one would expect,his writing is mostly clear and,to be fair,some chapters stand out above the rest.I enjoyed,for instance,the chapter in which Harold discovers how to think on his own.While Harold and Erica are certainly not strong or memorable characters,the more serious problems with The Social Animal lie elsewhere.These problems partly involve Brooks's attempt to translate his tale into science. 1.The author mentions the functions of science at the beginning of the passage to ____. A. illustrate where science can be applied B. demonstrate the value of Brooks's new book C. remind the reader of the importance of science D. explain why many writers use science in their works 2.According to the author,which of the following could be a strength of the book? A. Its strong basis. B. Its convincing points. C. Its clear writing. D. Its memorable characters. 3.What is the author's general attitude towards the book? A. Contradictory. B. Supportive. C. Cautious. D. Critical. 4.What is the author likely to write about after the last paragraph? A. Problems with the book. B. Brooks's life experiences. C. Death of the characters. D. Brooks's translation skills. Passage 8 2014 安徽,8 分词数:315 You may not have heard of Ashoka, but for the past 27 years, this association,founded by Bill Drayton, has fought poverty ( 贫 困 ) and sickness,promoted education and encouraged small businesses. To support these worthy causes, Ashoka provides money for the world's most promising "changemakers" seeking to solve ( 解决) urgent problems and would like to create a world in which every citizen is a changemaker. Drayton believes that anyone can become an agent for change. The important thing is to simply give yourself permission. If you see a problem that you care about, you can help solve it. The young in particular are willing to accept this concept because at heart every child wants to grow into a happy, healthy, contributing adult. In fact, it is many young people's ambition to set up 120 programmes or businesses that improve social conditions. An excellent example is an Ashoka project started in 1995 in Dhaka, which handled the rubbish problem facing the city, helped local farmers and provided an income for poor people there. When Masqsood and Iftekhar began to study the problem of all the uncollected rubbish that lay in Dhaka's streets, attracting rats and disease, they discovered that 80% of it was natural waste. So they educated the poor people in the city to compost (把……制成堆肥) this waste. They knew that they would have a market for the end product because local farmers were struggling with chemical fertilisers (化肥) which were expensive and had reduced the natural minerals in they soil over they years. At first, they were refused, but once they were able to persuade them that there was money to be made, the project took off. In 2009 sales were $14,000. Drayton is optimistic that in ten years Ashoka will be making really serious, practical progress in bringing about social change by changing the way we look at economic development. 1.Which of the following could be the best title for the passage? A. Changemakers B. Businessmen C. Social Conditions D. Rubbish Problem 2.The underlined word "them" in Paragraph 3 probably refers to "____". A. the local farmers B. Masqsood and Iftekhar C. Drayton and his team D. the poor people in Dhaka 3.It can be concluded from the passage that anyone can become a changemaker if he ____. A. considers Drayton's concept B. gets permission from Ashoka C. tries to improve social conditions D. is a young, happy and healthy adult 4.The author's attitude towards Ashoka's program can be described as ____. A. changing B. forgiving C. cautious D. positive Passage 9 2014 广东,10 分词数:372 Like many new graduates, I left university full of hope for the future but with no real idea of what I wanted to do. My degree, with honors, in English literature had not really prepared me for anything practical. I knew I wanted to make a difference in the world somehow, but I had no idea how to do that. That's when I learned about the Lighthouse Project. I started my journey as a Lighthouse Project volunteer by reading as much as I could about 121 the experiences of previous volunteers. I knew it would be a lot of hard work, and that I would be away from my family and friends for a very long time. In short, I did not take my decision to apply for the Lighthouse Project lightly. Neither did my family. Eventually, however, I won the support of my family, and I sent in all the paperwork needed for application. After countless interviews and presentations, I managed to stand out among the candidates and survive the test alone. Several months later, I finally received a call asking me to report for duty. I would be going to a small village near Abuja, Nigeria. Where? What? Nigeria? I had no idea. But I was about to find out. After completing my training, I was sent to the village that was small and desperately in need of proper accommodation. Though the local villagers were poor, they offered their homes, hearts, and food as if I were their own family. I was asked to lead a small team of local people in building a new schoolhouse. For the next year or so, I taught in that same schoolhouse. But I sometimes think I learned more from my students than they did from me. Sometime during that period, I realized that all those things that had seemed so strange or unusual to me no longer did, though I did not get anywhere with the local language, and I returned to the United States a different man. The Lighthouse Project had changed my life forever. 1.What do we know about the author? A. His university education focused on theoretical knowledge. B. His dream at university was to become a volunteer. C. He took pride in having contributed to the world. D. He felt honored to study English literature. 2.According to Paragraph 2, it is most likely that the author ____. A. discussed his decision with his family B. asked previous volunteers about voluntary work C. attended special training to perform difficult tasks D. felt sad about having to leave his family and friends 3.In his application for the volunteer job, the author ____. A. participated in many discussions B. went through challenging survival tests C. wrote quite a few papers on voluntary work 122 D. faced strong competition from other candidates 4.On arrival at the village, the author was ____. A. asked to lead a farming team B. sent to teach in a schoolhouse C. received warmly by local villagers D. arranged to live in a separate house 5.What can we infer from the author' s experiences in Nigeria? A. He found some difficulty adapting to the local culture. B. He had learned to communicate in the local language. C. He had overcome all his weaknesses before he left for home. D. He was chosen as the most respectable teacher by his students. Passage 10 2013 山东,10 分词数:348 George Gershwin, born in 1898, was one of America's greatest composers. He published his first song when he was eighteen years old. During the next twenty years he wrote more than five hundred songs. Many of Gershwin's songs were first written for musical plays performed in theatres in New York City. These plays were a popular form of entertainment in the 1920s and 1930s. Many of his songs have remained popular as ever. Over the years they have been sung and played in every possible way — from jazz to country. In the 1920s there was a debate in the United States about jazz music. Could jazz, some people asked, be considered serious music? In 1924 jazz musician and orchestra leader Paul Whiteman decided to organize a special concert to show that jazz was serious music. Gershwin agreed to compose something for the concert before he realized he had just a few weeks to do it. And in that short time, he composed a piece for piano and orchestra which he called Rhapsody in Blue.Gershwin himself played the piano at the concert. The audience were thrilled when they heard his music. It made him world-famous and showed that jazz music could be both serious and popular. In 1928, Gershwin went to Paris. He applied to study composition ( 作 曲 ) with the well-known musician Nadia Boulanger, but she rejected him. She was afraid that classical study would ruin his jazz-influenced style. While there, Gershwin wrote An American in Paris. When it was first performed, critics (评论家) were divided over the music. Some called it happy and full of life, to others it was silly and boring. But it quickly became popular in Europe and the United 123 States. It still remains one of his most famous works. George Gershwin died in 1937, just days after doctors learned he had brain cancer. He was only thirty-nine years old. Newspapers all over the world reported his death on their front pages. People mourned the loss of the man and all the music he might have still written. 1.Many of Gershwin's musical works were ____ . A. written about New Yorkers B. composed for Paul Whiteman C. played mainly in the countryside D. performed in various ways 2.What do we know about the concert organized by Whiteman? A. It attracted more people to theatres. B. It proved jazz could be serious music. C. It made Gershwin leader of the orchestra. D. It caused a debate among jazz musicians. 3.What did Gershwin do during his stay in Paris? A. He created one of his best works. B. He studied with Nadia Boulanger. C. He argued with French critics. D. He changed his music style. 4.What do we learn from the last paragraph? A. Many of Gershwin's works were lost. B. The death of Gershwin was widely reported. C. A concert was held in memory of Gershwin. D. Brain cancer research started after Gershwin's death. 5.Which of the following best describes Gershwin? A. Talented and productive. B. Serious and boring. C. Popular and unhappy. D. Friendly and honest. Passage 11 2013 湖北,8 分词数:387 Mothers and daughters go through so much — yet when was the last time a mother and daughter sat down to write a book together about it all? Perri Klass and her mother, Sheila Solomon Klass, both gifted professional writers, prove to be ideal co-writers as they examine their decades of motherhood, daughterhood, and the wonderful ways their lives have overlapped (重叠). 124 Perri notes with amazement how closely her own life has mirrored her mother's: both have full-time careers; both have published books, articles, and stories; each has three children; they both love to read. They also love to travel — in fact, they often take trips together. But in truth, the harder they look at their lives, the more they acknowledge their big differences in circumstance and basic nature.   A child of the Depression ( 大 萧 条 ), Sheila was raised in Brooklyn by parents who considered education a luxury for girls. Starting with her college education, she has fought for everything she's ever accomplished. Perri, on the other hand, grew up privileged in the New Jersey suburbs of the 1960s and 1970s. For Sheila, wasting time or money is a crime, and luxury is unthinkable while Perri enjoys the occasional small luxury, but has not been successful at trying to persuade her mother into enjoying even the tiniest thing she likes.   Each writing in her own unmistakable voice, Perri and Sheila take turns exploring the joys and pains, the love and bitterness, the minor troubles and lasting respect that have always bonded them together. Sheila describes the adventure of giving birth to Perri in a tiny town in Trinidad where her husband was doing research fieldwork. Perri admits that she can't sort out all the mess in the households, even though she knows it drives her mother crazy. Together they compare thoughts on bringing up children and working, admit long-hidden sorrows, and enjoy precious memories.   Looking deep into the lives they have lived separately and together, Perri and Sheila tell their mother-daughter story with honesty, humor, enthusiasm, and admiration for each other. A written account in two voices, Every Mother Is a Daughter is a duet (二重奏) that produces a deep, strong sound with the experiences that all mothers and daughters will recognize. 1.Why does Perri think that her own life has mirrored her mother's? A. They both have gone through difficult times. B.They have strong emotional ties with each other. C. They have the same joys and pains, and love and bitterness. D.They both have experiences as daughter, mother and writer. 2.The word "luxury" in Paragraph 3 means ____. A. something rare but not pleasant B.something that cannot be imagined 125 C. something expensive but not necessary D.something that can only be enjoyed by boys 3.What is Paragraph 4 mainly about? A. The content of the book. B.The purpose of the book. C. The influence of the book. D.The writing style of the book. 4.How are women's lives explored in this book? A. In a musical form. B.Through field research. C. With unique writing skills. D.From different points of view. Passage 12 2012 安徽,8 分词数:255 When Frida Kahlo's paintings were on show in London, a poet described her paintings as "a ribbon (丝带) around a bomb". Such comments seem to suggest Kahlo had a big influence on the art world of her time. Sadly, she is actually a much bigger name today than she was during her time. Born in 1907 in a village near Mexico City, Kahlo suffered from polio (小儿麻痹症) at the age of seven. Her spine ( 脊 柱 ) became bent as she grew older. Then, in 1925, her back was broken in several places in a school-bus accident. Throughout the rest of her life, the artist had many operations, but nothing was able to cure the terrible pain in her back. However, the accident had an unexpected side effect. While lying in her bed recovering, Kahlo taught herself to paint. In 1929, she got married to Diego Rivera, another famous Mexican artist. Rivera's strong influence on Kahlo's style can be seen in her early works, but her later works from the 1940s, known today as her best works, show less influence from her husband. Unfortunately, her works did not attract much attention in the 1930s and 1940s, even in her home country. Her first one-woman show in Mexico was not held until 1953. For more than a decade after her death in 1954, Kahlo's works remained largely unnoticed by the world, but in the 1970s her works began to gain international fame at last. 1.What does the phrase "a much bigger name" in Paragraph 1 most nearly mean? 126 A. a far better artist B. a far more gifted artist C. a much stronger person D. a much more famous person 2.The terrible pain Kahlo suffered was caused by . A. polio B. her bent spine C. back injuries D. the operations she had 3.Kahlo's style had become increasingly independent since the . A. 1930s B. 1940s C. 1950s D. 1970s 4.What is the author's attitude toward Kahlo? A. Devotion. B. Sympathy. C. Worry. D. Encouragement. 专题七 逸闻趣事类 2018 年 Passage 2018 天津,10 分 话题:参观博物馆,误闯民宅 词数:397 When I was 17, I read a magazine article about a museum called the McNay, once the home of a watercolorist named Marian McNay. She had requested the community to turn it into a museum upon her death. On a sunny Saturday, Sally and I drove over to the museum. She asked, "Do you have the address?" "No, but I’ll recognize it, there was a picture in the magazine." "Oh, stop. There it is!" The museum was free. We entered, excited. A group of people sitting in the hall stopped talking and stared at us." May I help you?" a man asked. "No," I said. "We’re fine." Tour guides got on my nerves. What if they talked a long time about a painting you weren’t that interested in? Sally had gone upstairs. The people in the hall seemed very nosy( 爱 窥 探 的 ), keeping their eyes on me with curiosity. What was their problem? I saw some nice sculptures in one room. Suddenly I sensed a man standing behind me. "Where do you think you are?" he asked. I turned sharply. "The McNay Art Museum!" He smiled, shaking his head. "Sorry, the McNay is on New Braunfels Street." "What’s this place?" I asked, still confused. "Well, it’s our home." My heart jolted(震颤). I raced to the staircase and called out, "Sally! Come down immediately!" "There’s some really good stuff(艺术作品) up there." She stepped down, looking confused. I pushed her toward the front door, waving at the family, saying, "Sorry, please forgive us, you have a really nice place." Outside, when I told Sally what happened, she covered her mouth, laughing. She couldn’t believe how long they let us look around without saying anything. The real McNay was splendid, but we felt nervous the whole time we were there. Van Gogh, Picasso. This time, we stayed together, in case anything else unusual happened. Thirty years later, a woman approached me in a public place. "Excuse me, did you ever enter a residence, long ago, thinking it was the McNay Museum?" "Yes. But how do you know? We never told anyone." "That was my home. I was a teenager sitting in the hall. Before you came over, I never realized what a beautiful place I lived in. I never felt lucky before. You thought it was a museum. 127 My feelings about my home changed after that. I’ve always wanted to thank you." 1.What do we know about Marian McNay? A. She was a painter. B. She was a community leader. C. She was a museum director. D. She was a journalist. 2.Why did the author refuse the help from the man in the house? A. She disliked people who were nosy. B. She felt nervous when talking to strangers. C. She knew more about art than the man. D. She mistook him for a tour guide. 3.How did the author feel about being stared at by the people in the hall? A. Puzzled. B. Concerned. C. Frightened. D. Delighted. 4. Why did the author describe the real McNay museum in just a few words? A. The real museum lacked enough artwork to interest her. B. She was too upset to spend much time at the real museum. C. The McNay was disappointing compared with the house. D. The event happening in the house was more significant. 5.What could we learn from the last paragraph? A. People should have good taste to enjoy life. B. People should spend more time with their family. C. People tend to be blind to the beauty around them. D. People tend to educate teenagers at a museum. 2017 年 Passage 2017 江苏,6 分 话题:鸟儿"胎教" 词数:322 Before birth, babies can tell the difference between loud sounds and voices. They can even distinguish their mother’s voice from that of a female stranger. But when it comes to embryonic learning( 胎教) ,birds could rule the roost . As recently reported in The Auk:Ornithological Advances , some mother birds may teach their young to sing even before they hatch( 孵化). New-born chicks can then imitate their mom’s call within a few days of entering the world. 128 This educational method was first observed in 2012 by Sonia Kleindorfer, a biologist at Flinders University in South Australia, and her colleagues. Female Australian superb fairy wrens were found to repeat one sound over and over again while hatching their eggs. When the eggs were hatched, the baby birds made the similar chirp to their mothers — a sound that served as their regular "feed me!" call. To find out if the special quality was more widespread in birds, the researchers sought the red-backed fairy wren, another species of Australian songbird. First they collected sound data from 67 nests in four sites in Queensland before and after hatching. Then they identified begging calls by analyzing the order and number of notes. A computer analysis blindly compared calls produced by mothers and chicks, ranking them by similarity. It turns out that baby red-backed fairy wrens also emerge chirping like their moms. And the more frequently mothers had called to their eggs, the more similar were the babies’ begging calls. In addition, the team set up a separate experiment that suggested that the baby birds that most closely imitated their mom’s voice were rewarded with the most food. This observation hints that effective embryonic learning could signal neurological(神经系统的) strengths of children to parents. An evolutionary inference can then be drawn. "As a parent, do you invest in quality children, or do you invest in children that are in need?"Kleindorfer asks. "Our results suggest that they might be going for quality." 1.The underlined phrase in Paragraph 1 means " ". A. be the worstB. be the best C. be just as badD. be just as good 2.What are Kleindorfer’s findings based on? A. Similarities between the calls of moms and chicks. B. The observation of fairy wrens across Australia. C. The data collected from Queensland’s locals. D. Controlled experiments on wrens and other birds. 3.Embryonic learning helps mother birds to identify the baby birds which . A. can receive quality signals B. are in need of training C. fit the environment better 129 D. make the loudest call 2012—2016 年 Passage 1 2016 全国卷Ⅱ,8 分词数:291 Five years ago, when I taught art at a school in Seattle, I used Tinkertoys as a test at the beginning of a term to find out something about my students. I put a small set of Tinkertoys in front of each student, and said: "Make something out of the Tinkertoys. You have 45 minutes today — and 45 minutes each day for the rest of the week." A few students hesitated to start. They waited to see what the rest of the class would do. Several others checked the instructions and made something according to one of the model plans provided. Another group built something out of their own imaginations. Once I had a boy who worked experimentally with Tinkertoys in his free time. His constructions filled a shelf in the art classroom and a good part of his bedroom at home. I was delighted at the presence of such a student. Here was an exceptionally creative mind at work. His presence meant that I had an unexpected teaching assistant in class whose creativity would infect(感染)other students. Encouraging this kind of thinking has a downside. I ran the risk of losing those students who had a different style of thinking. Without fail one would declare, "But I'm just not creative." "Do you dream at night when you're asleep?" "Oh, sure." "So tell me one of your most interesting dreams." The student would tell something wildly imaginative. Flying in the sky or in a time machine or growing three heads."That's pretty creative. Who does that for you?" "Nobody. I do it. "Really — at night, when you're asleep?" "Sure." "Try doing it in the daytime, in class, okay?" 1.The teacher used Tinkertoys in class in order to ____. A. know more about the students B. make the lessons more exciting C. raise the students' interest in art 130 D. teach the students about toy design 2.What do we know about the boy mentioned in Paragraph 3? A. He liked to help his teacher. B. He preferred to study alone. C. He was active in class. D. He was imaginative. 3.What does the underlined word "downside" in Paragraph 4 probably mean? A. Mistake. B. Drawback. C. Difficulty. D. Burden. 4.Why did the teacher ask the students to talk about their dreams? A. To help them to see their creativity. B. To find out about their sleeping habits. C. To help them to improve their memory. D. To find out about their ways of thinking. Passage 2 2016 全国卷Ⅱ,6 分词数:297 A new collection of photos brings an unsuccessful Antarctic voyage back to life. Frank Hurley's pictures would be outstanding — undoubtedly first-rate photo-journalism — if they had been made last week. In fact, they were shot from 1914 through 1916, most of them after a disastrous shipwreck( 海难) , by a cameraman who had no reasonable expectation of survival. Many of the images were stored in an ice chest, under freezing water, in the damaged wooden ship. The ship was the Endurance, a small, tight, Norwegian-built three-master that was intended to take Sir Ernest Shackleton and a small crew of seamen and scientists, 27 men in all, to the southernmost shore of Antarctica's Weddell Sea. From that point Shackleton wanted to force a passage by dog sled (雪橇) across the continent. The journey was intended to achieve more than what Captain Robert Falcon Scott had done. Captain Scott had reached the South Pole early in 1912 but had died with his four companions on the march back. As writer Caroline Alexander makes clear in her forceful and well-researched story The Endurance, adventuring was even then a thoroughly commercial effort. Scott's last journey, completed as he lay in a tent dying of cold and hunger, caught the world's imagination, and a film made in his honor drew crowds. Shackleton, a onetime British merchant-navy officer who had got to within 100 miles of the South Pole in 1908, started a business before his 1914 voyage to make money from movie and still photography. Frank Hurley, a confident and gifted Australian 131 photographer who knew the Antarctic, was hired to make the images, most of which have never before been published. 1.What do we know about the photos taken by Hurley? A. They were made last week. B. They showed undersea sceneries. C. They were found by a cameraman. D. They recorded a disastrous adventure. 2.Who reached the South Pole first according to the text? A. Frank Hurley. B. Ernest Shackleton. C. Robert Falcon Scott. D. Caroline Alexander. 3.What does Alexander think was the purpose of the 1914 voyage? A. Artistic creation. B. Scientific research. C. Money making. D. Treasure hunting. Passage 3 2016 天津,12.5 分词数:415 Every man wants his son to be somewhat of a clone, not in features but in footsteps. As he grows you also age, and your ambitions become more unachievable. You begin to realize that your boy, in your footsteps, could probably accomplish what you hoped for. But footsteps can be muddied and they can go off in different directions. My son Jody has hated school since day one in kindergarten. Science projects waited until the last moment. Book reports weren't written until the final threat. I've been a newspaperman all my adult life. My daughter is a university graduate working toward her master's degree in English. But Jody? When he entered the tenth grade he became a "vo-tech" student(技校学生). They're called "motorheads" by the rest of the student body. When a secretary in my office first called him "motorhead", I was shocked. "Hey, he's a good kid," I wanted to say. "And smart, really." I learned later that motorheads are, indeed, different. They usually have dirty hands and wear dirty work clothes. And they don't often make school honor rolls(光荣榜). But being the parent of a motorhead is itself an experience in education. We who labor in clean shirts in offices don't have the abilities that motorheads have. I began to learn this when I had my car crashed. The cost to repair it was estimated at $800. "Hey, I can fix it," said Jody. I 132 doubted it, but let him go ahead, for I had nothing to lose. My son, with other motorheads, fixed the car. They got parts( 零件) from a junkyard, and ability from vo-tech classes. The cost was $25 instead of $800. Since that first repair job, a broken air-conditioner, a non-functioning washer and a non-toasting toaster have been fixed. Neighbors and co-workers trust their car repairs to him. These kids are happiest when doing repairs. They joke and laugh and are living in their own relaxed world. And their minds are bright despite their dirty hands and clothes. I have learned a lot from my motorhead: publishers need printers, engineers need mechanics, and architects need builders. Most important, I have learned that fathers don't need clones in footsteps or anywhere else. My son may never make the school honor roll. But he made mine. 1.What used to be the author's hope for his son? A. To avoid becoming his clone. B. To resemble him in appearance. C. To develop in a different direction. D. To reach the author's unachieved goals. 2.What can we learn about the author's children? A. His daughter does better in school. B. His daughter has got a master's degree. C. His son tried hard to finish homework. D. His son couldn't write his book reports. 3.The author let his son repair the car because he believed that ____. A. his son had the ability to fix it B. it would save him much time C. it wouldn't cause him any more loss D. other motorheads would come to help 4.In the author's eyes, motorheads are ____ . A. tidy and hardworking B. cheerful and smart C. lazy but bright 133 D. relaxed but rude 5.What did the author realize in the end? A. It is unwise to expect your child to follow your path. B. It is important for one to make the honor roll. C. Architects play a more important role than builders. D. Motorheads have greater ability than office workers. Passage 4 2015 重庆,8 分词数:307 At thirteen, I was diagnosed (诊断) with a kind of attention disorder. It made school difficult for me. When everyone else in the class was focusing on tasks, I could not. In my first literature class, Mrs. Smith asked us to read a story and then write on it, all within 45 minutes. I raised my hand right away and said, "Mrs. Smith, you see, the doctor said I have attention problems. I might not be able to do it." She glanced down at me through her glasses, "You are no different from your classmates, young man." I tried, but I didn't finish the reading when the bell rang. I had to take it home. In the quietness of my bedroom, the story suddenly all became clear to me. It was about a blind person, Louis Braille. He lived in a time when the blind couldn't get much education. But Louis didn't give up. Instead, he invented a reading system of raised dots (点), which opened up a whole new world of knowledge to the blind. Wasn't I the "blind" in my class, being made to learn like the "sighted" students? My thoughts spilled out and my pen started to dance. I completed the task within 40 minutes. Indeed, I was no different from others; I just needed a quieter place. If Louis could find his way out of his problems, why should I ever give up? I didn't expect anything when I handed in my paper to Mrs. Smith, so it was quite a surprise when it came back to me the next day — with an "A" on it. At the bottom of the paper were these words: "See what you can do when you keep trying?" 1.The author didn't finish the reading in class because ____. A. he was new to the class B. he was tired of literature C. he had an attention disorder D. he wanted to take the task home 2.What do we know about Louis Braille from the passage? 134 A. He had good sight. B. He made a great invention. C. He gave up reading. D. He learned a lot from school. 3.What was Mrs. Smith's attitude to the author at the end of the story? A. Angry. B. Impatient. C. Sympathetic. D. Encouraging. 4.What is the main idea of the passage? A. The disabled should be treated with respect. B. A teacher can open up a new world to students. C. One can find his way out of difficulties with efforts. D. Everyone needs a hand when faced with challenges. Passage 5 2015 安徽,8 分词数:316 When her five daughters were young, Helene An always told them that there was strength in unity( 团结). To show this, she held up one chopstick, representing one person. Then she easily broke it into two pieces. Next, she tied several chopsticks together, representing a family. She showed the girls it was hard to break the tied chopsticks. This lesson about family unity stayed with the daughters as they grew up. Helene An and her family own a large restaurant business in California. However, when Helene and her husband Danny left their home in Vietnam in 1975, they didn't have much money. They moved their family to San Francisco. There they joined Danny's mother, Diana, who owned a small Italian sandwich shop. Soon afterwards, Helene and Diana changed the sandwich shop into a small Vietnamese restaurant. The five daughters helped in the restaurant when they were young. However, Helene did not want her daughters to always work in the family business because she thought it was too hard. Eventually the girls all graduated from college and went away to work for themselves, but one by one, the daughters returned to work in the family business. They opened new restaurants in San Francisco and Los Angeles. Even though family members sometimes disagreed with each other, they worked together to make the business successful. Daughter Elizabeth explains, "Our mother taught us that to succeed we must have unity, and to have unity we must have peace. Without the strength of the family, there is no business." Their expanding business became a large corporation in 1996, with three generations of Ans working together.Now the Ans' corporation makes more than $20 million each year. Although 135 they began with a small restaurant, they had big dreams, and they worked together. Now they are a big success. 1.Helene tied several chopsticks together to show ____. A. the strength of family unity B. the difficulty of growing up C. the advantage of chopsticks D. the best way of giving a lesson 2.We can learn from Paragraph 2 that the An family____. A. started a business in 1975 B. left Vietnam without much money C. bought a restaurant in San Francisco D. opened a sandwich shop in Los Angeles 3.What can we infer about the An daughters? A. They did not finish their college education. B. They could not bear to work in the family business. C. They were influenced by what Helene taught them. D. They were troubled by disagreement among family members. 4.Which of the following can be the best title for the passage? A. How to Run a Corporation B. Strength Comes from Peace C. How to Achieve a Big Dream D. Family Unity Builds Success Passage 6 2015 新课标全国Ⅱ,8 分词数:277 My color television has given me nothing but a headache. I was able to buy it a little over a year ago because I had my relatives give me money for my birthday instead of a lot of clothes that wouldn't fit. I let a salesclerk fool me into buying a discontinued model. I realized this a day later, when I saw newspaper advertisements for the set at seventy-five dollars less than I had paid. The set worked so beautifully when I first got it home that I would keep it on until stations signed off for the night. Fortunately, I didn't get any channels showing all-night movies or I would never have gotten to bed. Then I started developing a problem with the set that involved static (静电) noise. For some reason, when certain shows switched into a commercial, a loud noise would sound for a few seconds. Gradually, this noise began to appear during a show, and to get rid of it, I had to change to another channel and then change it back. Sometimes this technique would not work, and I had to pick up the set and shake it to remove the sound. I actually began to build up my arm muscles (肌肉) shaking my set. 136 When neither of these methods removed the static noise, I would sit helplessly and wait for the noise to go away. At last I ended up hitting the set with my fist, and it stopped working altogether. My trip to the repair shop cost me $62, and the set is working well now, but I keep expecting more trouble. 1.Why did the author say he was fooled into buying the TV set? A. He got an older model than he had expected. B. He couldn't return it when it was broken. C. He could have bought it at a lower price. D. He failed to find any movie shows on it. 2.Which of the following can best replace the phrase "signed off" in Paragraph 1? A. ended all their programs B. provided fewer channels C. changed to commercials D. showed all-night movies 3.How did the author finally get his TV set working again? A. By shaking and hitting it. B. By turning it on and off. C. By switching channels. D. By having it repaired. 4.How does the author sound when telling the story? A. Curious. B. Anxious. C. Cautious. D. Humorous. Passage 7 2015 广东,10 分词数:330 When I was nine years old, I loved to go fishing with my dad. But the only thing that wasn't very fun about it was that he could catch many fish while I couldn't catch anything. I usually got pretty upset and kept asking him why. He always answered, "Son, if you want to catch a fish, you have to think like a fish." I remember being even more upset then because,"I'm not a fish!" I didn't know how to think like a fish. Besides, I reasoned, how could what I think influence what a fish does? As I got a little older I began to understand what my dad really meant. So, I read some books on fish. And I even joined the local fishing club and started attending the monthly meetings. I learned that a fish is a cold-blooded animal and therefore is very sensitive to water temperature. That is why fish prefer shallow water to deep water because the former is warmer. Besides, water is usually warmer in direct sunlight than in the shade. Yet, fish don't have any eyelids (眼皮) and the sun hurts their eyes... The more I understood fish, the more I became effective at finding and 137 catching them. When I grew up and entered the business world, I remember hearing my first boss say, "We all need to think like salespeople." But it didn't completely make sense. My dad never once said, "If you want to catch a fish you need to think like a fisherman." What he said was, "You need to think like a fish." Years later, with great efforts to promote long-term services to people much older and richer than me, I gradually learned what we all need is to think more like customers. It is not an easy job. I will show you how in the following chapters. 1.Why was the author upset in the fishing trips when he was nine? A. He could not catch a fish. B. His father was not patient with him. C. His father did not teach him fishing. D. He could not influence a fish as his father did. 2.What did the author's father really mean? A. To read about fish. B. To learn fishing by oneself. C. To understand what fish think. D. To study fishing in many ways. 3.According to the author, fish are most likely to be found ____. A. in deep water on sunny days B. in deep water on cloudy days C. in shallow water under sunlight D. in shallow water under waterside trees 4.After entering the business world, the author found ____. A. it easy to think like a customer B. his father's fishing advice inspiring C. his first boss's sales ideas reasonable D. it difficult to sell services to poor people 5.This passage most likely comes from ____. A. a fishing guide B. a popular sales book C. a novel on childhood D. a millionaire's biography Passage 8 2014 重庆,8 分词数:290 I was never very neat, while my roommate Kate was extremely organized. Each of her objects had its place, but mine always hid somewhere. She even labeled ( 贴标签) everything. I always looked for everything. Over time, Kate got neater and I got messier. She would push my dirty clothing over, and I would lay my books on her tidy desk. We both got tired of each other. War broke out one evening. Kate came into the room. Soon, I heard her screaming, "Take your shoes away! Why under my bed!" Deafened, I saw my shoes flying at me. I jumped to my feet and started yelling. She yelled back louder. 138 The room was filled with anger. We could not have stayed together for a single minute but for a phone call. Kate answered it. From her end of the conversation, I could tell right away her grandma was seriously ill. When she hung up, she quickly crawled (爬) under her covers, sobbing. Obviously, that was something she should not go through alone. All of a sudden, a warm feeling of sympathy rose up in my heart. Slowly ,I collected the pencils, took back the books, made my bed, cleaned the socks and swept the floor, even on her side. I got so into my work that I even didn't notice Kate had sat up. She was watching, her tears dried and her expression one of disbelief. Then, she reached out her hands to grasp mine. I looked up into her eyes. She smiled at me, "Thanks." Kate and I stayed roommates for the rest of the year. We didn't always agree, but we learned the key to living together: giving in, cleaning up and holding on. 1.What made Kate so angry one evening? A. She couldn't find her books. B. She heard the author shouting loud. C. She got the news that her grandma was ill. D. She saw the author's shoes beneath her bed. 2.The author tidied up the room most probably because ____. A. she was scared by Kate's anger B. she hated herself for being so messy C. she wanted to show her care D. she was asked by Kate to do so 3.How is Paragraph 1 mainly developed? A. By analyzing causes. B. By showing differences. C. By describing a process. D. By following time order. 4.What might be the best title for the story? A. My Friend Kate B. Hard Work Pays Off C. How to Be Organized D. Learning to Be Roommates Passage 9 2014 福建,8 分词数:332 It was Mother's Day morning last year and I was doing shopping at our local supermarket with my five-year-old son, Tenyson. As we were leaving, we found that only minutes earlier an elderly woman had fallen over at the entrance and had hit her head on the concrete. Her husband was with her, but there was blood everywhere and the woman was embarrassed and clearly in shock. Walking towards the scene, Tenyson became very upset about what had happened to the 139 couple. He said to me, "Mum, it's not much fun falling over in front of everyone." At the front of the supermarket, a charity ( 慈善) group had set up a stand selling cooked sausages and flowers to raise funds. Tenyson suggested that we should buy the lady a flower. "It will make her feel better," he said. I was amazed that he'd come up with this sweet idea. So we went over to the flower seller and asked her if we could buy a flower for the lady to cheer her up. "Just take it," she replied. "I can't take your money for such a wonderful gesture." By now paramedics (急救人员) had arrived, and were attending to the injured woman. As we walked up to her, my son became intimidated by all the blood and medical equipment. He said he was just too scared to go up to her. Instead I gave the flower to the woman's husband and told him, "My son was very upset for your wife and wanted to give her this flower to make her feel better." At that, the old man started crying and said, "Thank you so much,you have a wonderful son. Happy Mother's Day to you." The man then bent down and gave his wife the flower, telling her who it was from. Though badly hurt and shaken, the old lady looked up at Tenyson with love in her eyes and gave him a little smile. 1.What does the author intend to tell us? A. One can never be too careful. B. Actions speak louder than words. C. Love begins with a little smile. D. A small act of kindness brings a great joy. 2.Which of the following is TRUE according to the passage? A. The elderly woman was knocked down by Tenyson. B. Tenyson's idea of buying a flower gained his mother's support. C. Tenyson's care for the elderly woman puzzled the flower seller. D. The elderly woman was moved to tears by Tenyson's gesture. 3.The underlined word "intimidated" in the fourth paragraph probably means "____". A. astonished B. struck C. frightened D. excited 4.What would be the best title for the passage? A. Flower Power B. Mother's Day C. An Accidental Injury D. An Embarrassing Moment 140 Passage 10 2014 四川,8 分词数:315 In 1943,when I was 4,my parents moved from Coeur d'Alene,Idaho,to Fairbanks, Alaska,where adventure was never very far away. We arrived in the summer,just in time to enjoy the midnight sun.All that sunlight was fantastic for Mom's vegetable garden.Working in the garden at midnight tended to throw her timing off, so she didn't care much about my bedtime. Dad was a Railway Express agent and Mom was his clerk. That left me in a mess. I usually managed to find some trouble to get into. Once I had a little fire going in the dirt basement of a hotel. I had tried to light a barrel (桶) of paint but couldn't really get a good fire going. The smoke got pretty bad, though,and when I made my exit, a crowd and the police were there to greet me.The policemen took my matches and drove me home. Mom and Dad were occupied in the garden and Dad told the police to keep me, and they did! I had a tour of the prison before Mom rescued me. I hadn't turned 5 yet. As I entered kindergarten, the serious cold began to set in. Would it surprise you to know that I soon left part of my tongue on a metal handrail at school? As for Leonhard Seppala, famous as a dog sledder ( 驾雪橇者), I think I knew him well because I was taken for a ride with his white dog team one Sunday. At the time I didn't realize what a superstar he was, but I do remember the ride well. I was wrapped (包裹) heavily and well sheltered from the freezing and blowing weather. In 1950, we moved back to Coeur d'Alene, but we got one more Alaskan adventure when Leonhard invited us eight years later by paying a visit to Idaho to attend a gathering of former neighbors of Alaska. 1.What can be inferred about the author's family? A. His father was a cruel man. B. His parents didn't love him. C. His parents used to be very busy. D. His mother didn't have any jobs. 2.What happened when the author was 4? A. He learned to smoke. B. He was locked in a basement. C. He was arrested by the police. D. He nearly caused a fire accident. 3.Which of the following is true? A. Leonhard was good at driving dog sleds. 141 B. The author spent his whole childhood in Alaska. C. Leonhard often visited the author's family after 1950. D. The author suffered a lot while taking the dog sled in Alaska. 4.What is the author's purpose of writing the text? A. To look back on his childhood with adventures. B. To describe the extreme weather of Alaska. C. To express how much he misses Leonhard. D. To show off his pride in making trouble. Passage 11 2013 福建,8 分词数:332 When I was 12, all I wanted was a signet (图章) ring. They were the "in" thing and it seemed every girl except me had one. On my 13th birthday, my Mum gave me a signet ring with my initials (姓名首字母) carved into it. I was in heaven. What made it even more special was that it was about the only thing that wasn't being "replaced". We'd been burnt out in fires that swept through our area earlier that year and had lost everything — so most of the "new" stuff (东西) we got was really just to replace what we'd lost. But not my ring. My ring was new. Then, only one month later, I lost it. I took it off before bed and it was missing in the morning. I was sad and searched everywhere for it. But it seemed to have disappeared. Eventually, I gave up and stopped looking for it. And two years later, we sold the house and moved away. Years passed, and a couple of moves later, I was visiting my parents' when Mum told me that she had something for me. It wasn't my birthday, nor was it Easter or Christmas or any other gift-giving occasion. Mum noticed my questioning look. "You'll recognize this one," she said, smiling. Then she handed me a small ring box. I took it from her and opened it to find my beautiful signet ring inside. The family who had bought our house 13 years earlier had recently decided to do some redecoration, which included replacing the carpets. When they pulled the carpet up in my old bedroom, they found the ring. As it had my initials carved into it, they realized who owned the ring. They'd had it professionally cleaned up by a jeweler before sending it to my mother. And it still fits me. 142 1.The underlined word "in" in the first paragraph probably means "____". A. fashionable B. available C. practical D. renewable 2.When she got the ring back, the writer was about ____. A. 13 years old B. 15 years old C. 26 years old D. 28 years old 3.Which of the following is TRUE according to the passage? A. The writer's family moved several times. B. The writer never stopped looking for her ring. C. The writer's ring was cleaned up by the new house owner. D. The writer lost her ring in the morning when she took it off. 4.What would be the best title for the passage? A. My New Ring B. Lost and Found C. Lost and Replaced D. An Expensive Ring 专题八 史地人文类 2018 年 Passage 1 2018 全国卷 III,8 分 话题:影响城市选址的因素 词数:310 Cities usually have a good reason for being where they are, like a nearby port or river. People settle in these places because they are easy to get to and naturally suited to communications and trade. New York City, for example, is near a large harbour at the mouth of the Hudson River. Over 300 years its population grew gradually from 800 people to 8 million. But not all cities develop slowly over a long period of time. Boom towns grow from nothing almost overnight. In 1896, Dawson, Canada, was unmapped wilderness( 荒野). But gold was discovered there in 1897, and two years later, it was one of the largest cities in the West, with a population of 30,000. Dawson did not have any of the natural conveniences of cities like London or Paris. People went there for gold. They travelled over snow-covered mountains and sailed hundreds of miles up icy rivers. The path to Dawson was covered with thirty feet of wet snow that could fall without warning. An avalanche( 雪崩) once closed the path, killing 63 people. For many who made it to Dawson, however, the rewards were worth the difficult trip. Of the first 20,000 people who dug for gold, 4,000 got rich. About 100 of these stayed rich men for the rest of their lives. But no matter how rich they were, Dawson was never comfortable. Necessities like food and wood were very expensive. But soon, the gold that Dawson depended on had all been found. The city was crowded with disappointed people with no interest in settling down, and when they heard there were new gold discoveries in Alaska, they left Dawson City as quickly as they had come. Today, people still come and go — to see where the Canadian gold rush happened. Tourism is now the chief industry of Dawson City — its present population is 762. 1.What attracted the early settlers to New York City? 143 A. Its business culture. B. Its small population. C. Its geographical position. D. Its favourable climate. 2.What do we know about those who first dug for gold in Dawson? A. Two-thirds of them stayed there. B. One out of five people got rich. C. Almost everyone gave up. D. Half of them died. 3.What was the main reason for many people to leave Dawson? A. They found the city too crowded. B. They wanted to try their luck elsewhere. C. They were unable to stand the winter. D. They were short of food. 4.What is the text mainly about? A. The rise and fall of a city. B. The gold rush in Canada. C. Journeys into the wilderness. D. Tourism in Dawson. Passage 2 2018 浙江,6 分 话题:美国汽车业的发展 词数:292 As cultural symbols go, the American car is quite young. The Model T Ford was built at the Piquette Plant in Michigan a century ago, with the first rolling off the assembly line(装配线) on September 27, 1908. Only eleven cars were produced the next month. But eventually Henry Ford would build fifteen million of them. Modern America was born on the road, behind a wheel. The car shaped some of the most lasting aspects of American culture: the roadside diner, the billboard, the motel, even the hamburger. For most of the last century, the car represented what it meant to be American — going forward at high speed to find new worlds. The road novel, the road movie, these are the most typical American ideas, born of abundant petrol, cheap cars and a never-ending interstate highway system, the largest public works project in history. In 1928 Herbert Hoover imagined an America with "a chicken in every pot and a car in every garage." Since then, this society has moved onward, never looking back, as the car transformed America from a farm-based society into an industrial power. The cars that drove the American Dream have helped to create a global ecological disaster. In America the demand for oil has grown by 22 percent since 1990. The problems of excessive( 过 度 的 ) energy consumption, climate change and population growth have been described in a book by the American writer Thomas L. Friedman. He fears the worst, but hopes for the best. Friedman points out that the green economy( 经济) is a chance to keep American strength. "The ability to design, build and export green technologies for producing clean water, clean air and healthy and abundant food is going to be the currency of power in the new century." 1.Why is hamburger mentioned in paragraph 2? A. To explain Americans’ love for travelling by car. 144 B. To show the influence of cars on American culture. C. To stress the popularity of fast food with Americans. D. To praise the effectiveness of America’s road system. 2.What has the use of cars in America led to? A. Decline of economy. B. Environmental problems. C. A shortage of oil supply. D. A farm-based society. 3.What is Friedman’s attitude towards America’s future? A. Ambiguous. B. Doubtful. C. Hopeful. D. Tolerant. 2017 年 Passage 1 2017 全国卷Ⅰ,8 分 话题:爵士乐 词数:325 Some of the world ’ s most famous musicians recently gathered in Paris and New Orleans to celebrate the first annual International Jazz Day. UNESCO (United Nations Educational, Scientific and Cultural Organization) recently set April 30 as a day to raise awareness of jazz music, its significance, and its potential as a unifying (联合) voice across cultures. Despite the celebrations, though, in the U.S. the jazz audience continues to shrink and grow older, and the music has failed to connect with younger generations. It ’s Jason Moran ’s job to help change [ZZ(Z]that[ZZ)]. As the Kennedy Center ’s artistic adviser for jazz, Moran hopes to widen the audience for jazz, make the music more accessible, and preserve its history and culture. "Jazz seems like it ’s not really a part of the American appetite," Moran tells National Public Radio ’ s reporter Neal Conan. "What I ’ m hoping to accomplish is that my generation and younger start to reconsider and understand that jazz is not black and white anymore. It’s actually color, and it’s actually digital." Moran says one of the problems with jazz today is that the entertainment aspect of the music has been lost. "The music can’t be presented today the way it was in 1908 or 1958. It has to continue to move, because the way the world works is not the same," says Moran. Last year, Moran worked on a project that arranged Fats Waller’s music for a dance party, "just to kind of put it back in the mind that Waller is dance music as much as it is concert music," says Moran. "For me, it’s the recontextualization.In music, where does the emotion (情感) lie? Are we, 145 as humans, gaining any insight ( 感悟) on how to talk about ourselves and how something as abstract as a Charlie Parker record gets us into a dialogue about our emotions and our thoughts? Sometimes we lose sight that the music has a wider context," says Moran, "so I want to continue those dialogues. Those are the things I want to foster." 1.Why did UNESCO set April 30 as International Jazz Day? A. To remember the birth of jazz. B. To protect cultural diversity. C. To encourage people to study music. D. To recognize the value of jazz. 2.What does the underlined word "that" in paragraph 3 refer to? A. Jazz becoming more accessible. B. The production of jazz growing faster. C. Jazz being less popular with the young. D. The jazz audience becoming larger. 3.What can we infer about Moran’s opinion on jazz? A. It will disappear gradually. B. It remains black and white. C. It should keep up with the times. D. It changes every 50 years. 4.Which of the following can be the best title for the text? A. Exploring the Future of Jazz B. The Rise and Fall of Jazz C. The Story of a Jazz Musician D. Celebrating the Jazz Day Passage 2 2017 江苏,8 分 话题:反垄断管理 词数:439 A new commodity brings about a highly profitable, fast-growing industry, urging antitrust(反垄断) regulators to step in to check those who control its flow. A century ago, the resource in question was oil. Now similar concerns are being raised by the giants(巨头) that deal in data, the oil of the digital age. The most valuable firms are Google, Amazon, Facebook and Microsoft. All look 146 unstoppable. Such situations have led to calls for the tech giants to be broken up. But size alone is not a crime. The giants’ success has benefited consumers. Few want to live without search engines or a quick delivery. Far from charging consumers high prices, many of these services are free (users pay, in effect, by handing over yet more data). And the appearance of new-born giants suggests that newcomers can make waves, too. But there is cause for concern. The internet has made data abundant, all-present and far more valuable, changing the nature of data and competition. Google initially used the data collected from users to target advertising better. But recently it has discovered that data can be turned into new services: translation and visual recognition, to be sold to other companies. Internet companies’ control of data gives them enormous power. So they have a "God ’s eye view" of activities in their own markets and beyond. This nature of data makes the antitrust measures of the past less useful. Breaking up firms like Google into five small ones would not stop remaking themselves: in time, one of them would become great again. A rethink is required — and as a new approach starts to become apparent, two ideas stand out. The first is that antitrust authorities need to move from the industrial age into the 21st century. When considering a merger( 兼并), for example, they have traditionally used size to determine when to step in. They now need to take into account the extent of firms’ data assets(资产) when assessing the impact of deals. The purchase price could also be a signal that an established company is buying a new-born threat. When this takes place, especially when a new-born company has no revenue to speak of, the regulators should raise red flags. The second principle is to loosen the control that providers of on-line services have over data and give more to those who supply them. Companies could be forced to reveal to consumers what information they hold and how much money they make from it. Governments could order the sharing of certain kinds of data, with users’ consent. Restarting antitrust for the information age will not be easy. But if governments don’t want a data economy controlled by a few giants, they must act soon. 1.Why is there a call to break up giants? A. They have controlled the data market. 147 B. They collect enormous private data. C. They no longer provide free services. D. They dismissed some new-born giants. 2.What does the technological innovation in Paragraph 3 indicate? A. Data giants’ technology is very expensive. B. Google’s idea is popular among data firms. C. Data can strengthen giants’ controlling position. D. Data can be turned into new services or products. 3.[JP4]By paying attention to firms’ data assets, antitrust regulators could . A. kill a new threat B. avoid the size trap C. favour bigger firms D. charge higher prices 4.What is the purpose of loosening the giants’ control of data? A. Big companies could relieve data security pressure. B. Governments could relieve their financial pressure. C. Consumers could better protect their privacy. D. Small companies could get more opportunities. Passage 3 2017 浙江,7.5 分 话题:移民语言测试项目 词数:314 FLORENCE, Italy — Svetlana Cojochru feels hurt. The Moldovan has lived here seven years as a caregiver to Italian kids and the elderly, but in order to stay she’s had to prove her language skills by taking a test which requires her to write a postcard to an imaginary friend and answer a fictional job ad. Italy is the latest Western European country trying to control a growing immigrant( 移 民 ) population by demanding language skills in exchange for work permits, or in some cases, citizenship. Some immigrant advocates worry that as hard financial times make it more difficult for natives to keep jobs, such measures will become more a vehicle for intolerance than integration( 融 合).Others say it’s only natural that newcomers learn the language of their host nation, seeing it as a condition to ensure they can contribute to society. 148 Other European countries laid down a similar requirement for immigrants, and some terms are even tougher. The governments argue that this will help foreigners better join the society and promote understanding across cultures. Italy, which has a much weaker tradition of immigration, has witnessed a sharp increase in immigration in recent years. In 1990, immigrants numbered some 1.14 million out of Italy’s then 56.7 million people, or about 2 percent. At the start of this year, foreigners living in Italy amounted to 4.56 million of a total population of 60.6  million, or 7.5  percent, with immigrants’ children accounting for an ever larger percentage of births in Italy. Cojochru, the Moldovan caregiver, hoped obtaining permanent residence (居住权) would help her bring her two children to Italy; they live with her sister in Moldova, where salaries are among the lowest in Europe. She was skeptical that the language requirement would encourage integration. Italians always"see me as a foreigner,"an outsider, even though she ’s stayed in the country for years and can speak the local language fluently, she said. 1.Why does Cojochru have to take a language test? A. To continue to stay in Italy. B. To teach her children Italian. C. To find a better job in Italy. D. To better mix with the Italians. 2.Some people worry that the new language requirement may . A. reduce Italy’s population quickly B. cause conflicts among people C. lead to financial difficulties D. put pressure on schools 3.What do we know about Cojochru? A. She lives with her sister now in Italy. B. She enjoys learning the Italian language. C. She speaks Italian well enough for her job. D. She wishes to go back to her home country. 2012—2016 年 149 Passage 1 2016 四川,8 分词数:249 If you could have one superpower, what would it be? Dreaming about whether you would want to read minds, see through walls, or have superhuman strength may sound silly, but it actually gets to the heart of what really matters in your life. Every day in our work, we are inspired by the people we meet doing extraordinary things to improve the world. They have a different kind of superpower that all of us possess: the power to make a difference in the lives of others. We're not saying that everyone needs to contribute their lives to the poor. Your lives are busy enough doing homework, playing sports, making friends, seeking after your dreams. But we do think that you can live a more powerful life when you devote some of your time and energy to something much larger than yourself. Find an issue you are interested in and learn more. Volunteer or, if you can, contribute a little money to a cause. Whatever you do, don't be a bystander. Get involved. You may have the opportunity to make your biggest difference when you're older. But why not start now? Our own experience working together on health, development, and energy the last twenty years has been one of the most rewarding parts of our lives. It has changed who we are and continues to fuel our optimism about how much the lives of the poorest people will improve in the years ahead. 1.What does the underlined part in Paragraph 2 refer to? A. Your life style. B. Your life value. C. Your trouble in life. D. Your life experience. 2.Why does the author say they are inspired every day? A. They possess different kinds of superpowers. B. They have got the power to change the world. C. Some people around them are making the world better. D. There are many powerful people in their life and work. 3.What does the author stress in Paragraph 5? A. Learning more and contributing more to a cause. 150 B. Rising above self and acting to help others. C. Working hard to get a bigger opportunity. D. Trying your best to help the poor. 4.What can be inferred from the last paragraph? A. The author believes the lives of the poorest will get better. B. Much more progress will be made in the near future. C. The work on health is the most valuable experience. D. People's efforts have been materially rewarded. Passage 2 2016 全国卷Ⅰ,8 分 词数:333 The meaning of silence varies among cultural groups. Silences may be thoughtful, or they may be empty when a person has nothing to say. A silence in a conversation may also show stubbornness, uneasiness, or worry. Silence may be viewed by some cultural groups as extremely uncomfortable; therefore attempts may be made to fill every gap( 间 隙 ) with conversation. Persons in other cultural groups value silence and view it as necessary for understanding a person's needs. Many Native Americans value silence and feel it is a basic part of communicating among people, just as some traditional Chinese and Thai persons do. Therefore, when a person from one of these cultures is speaking and suddenly stops, what may be implied( 暗示)is that the person wants the listener to consider what has been said before continuing. In these cultures, silence is a call for reflection. Other cultures may use silence in other ways, particularly when dealing with conflicts among people or in relationships of people with different amounts of power. For example, Russian, French, and Spanish persons may use silence to show agreement between parties about the topic under discussion. However, Mexicans may use silence when instructions are given by a person in authority rather than be rude to that person by arguing with him or her. In still another use, persons in Asian cultures may view silence as a sign of respect, particularly to an elder or a person in authority. Nurses and other care-givers need to be aware of the possible meanings of silence when they come across the personal anxiety their patients may be experiencing. Nurses should recognize their own personal and cultural construction of silence so that a patient's silence is not interrupted 151 too early or allowed to go on unnecessarily. A nurse who understands the healing(治愈) value of silence can use this understanding to assist in the care of patients from their own and from other cultures. 1.What does the author say about silence in conversations? A. It implies anger. B. It promotes friendship. C. It is culture-specific. D. It is content-based. 2.Which of the following people might regard silence as a call for careful thought? A. The Chinese. B. The French. C. The Mexicans. D. The Russians. 3.What does the author advise nurses to do about silence? A. Let it continue as the patient pleases. B. Break it while treating patients. C. Evaluate its harm to patients. D. Make use of its healing effects. 4.What may be the best title for the text? A. Sound and Silence B. What It Means to Be Silent C. Silence to Native Americans D. Speech Is Silver; Silence Is Gold Passage 3 2016 四川,8 分词数:318 In the depths of the French Guianese rainforest, there still remain unusual groups of indigenous ( 土著的) people. Surprisingly, these people live largely by their own laws and their own social customs. And yet, people in this area are in fact French citizens because it has been a colony (殖民地) of the French Republic since 1946. In theory, they should live by the French law. However, their remote locations mean that the French law is often ignored or unknown, thus making them into an interesting area of "lawlessness" in the world. The lives of these people have finally been recorded thanks to the efforts of a Frenchman from Paris called Gin. Gin spent five months in early 2015 exploring the most remote corners of this area, which sits on the edge of the Amazon rainforest, with half its population of only 250,000 living in its capital, Cayenne. 152 "I have a special love for the French Guianese people. I have worked there on and off for almost ten years," says Gin. "I've been able to keep firm friendships with them. Thus I have been allowed to gain access to their living environment. I don't see it as a lawless land. But rather I see it as an area of freedom." "I wanted to show the audience a photographic record touching upon the uncivilized life," continues Gin. "I prefer to work in black and white, which allows me to show different specific worlds more clearly." His black-and-white pictures present a world almost lost in time. These pictures show people seemingly pushed into a world that they were unprepared for. These local citizens now have to balance their traditional self-supporting hunting lifestyle with the lifestyle offered by the modern French Republic, which brings with it not only necessary state welfare, but also alcoholism, betrayal and even suicide. 1.Why does the author feel surprised about the indigenous people in French Guiana? A. They seldom follow the French law. B. They often ignore the Guianese law. C. They are separated from the modern world. D. They are both Guianese and French citizens. 2.Gin introduces the special world of the indigenous Guianese as . A. a tour guide B. a geographer C. a film director D. a photographer 3.What is Gin's attitude towards the lives of the indigenous Guianese? A. Cautious. B. Doubtful. C. Uninterested. D. Appreciative. 4.What does the underlined word "it" in the last paragraph refer to? A. The modern French lifestyle. B. The self-supporting hunting. C. The uncivilized world. D. The French Republic. Passage 4 2015 新课标全国Ⅰ,8 分词数:371 The freezing Northeast hasn't been a terribly fun place to spend time this winter, so when the 153 chance came for a weekend to Sarasota, Florida, my bags were packed before you could say "sunshine". I left for the land of warmth and vitamin C(维生素 C), thinking of beaches and orange trees. When we touched down to blue skies and warm air, I sent up a small prayer of gratefulness. Swimming pools, wine tasting, and pink sunsets (at normal evening hours, not 4 in the afternoon) filled the weekend, but the best part — particularly to my taste, dulled by months of cold-weather root vegetables — was a 7 a.m. adventure to the Sarasota farmers' market that proved to be more than worth the early wake-up call. The market, which was founded in 1979, sets up its tents every Saturday from 7 a.m. to 1 p.m., rain or shine, along North Lemon and State streets. Baskets of perfect red strawberries; the red-painted sides of the Java Dawg coffee truck; and most of all, the tomatoes: amazing, large, soft and round red tomatoes. Disappointed by many a broken, vine-ripened( 蔓上成熟的) promise, I've refused to buy winter tomatoes for years. No matter how attractive they look in the store, once I get them home they're unfailingly dry, hard, and tasteless. But I homed in, with uncertainty, on one particular table at the Brown's Grove Farm's stand, full of fresh and soft tomatoes the size of my fist. These were the real deal — and at that moment, I realized that the best part of Sarasota in winter was going to be eating things that back home in New York I wouldn't be experiencing again for months. Delighted as I was by the tomatoes in sight, my happiness deepened when I learned that Brown's Grove Farm is one of the suppliers for Jack Dusty, a newly opened restaurant at the Sarasota Ritz Carlton, where — luckily for me — I was planning to have dinner that very night. Without even seeing the menu, I knew I'd be ordering every tomato on it. 1.What did the author think of her winter life in New York? A. Exciting. B. Boring. C. Relaxing. D. Annoying. 2.What made the author's getting up early worthwhile? A. Having a swim. B. Breathing in fresh air. C. Walking in the morning sun. D. Visiting a local farmers' market. 3.What can we learn about tomatoes sold in New York in winter? A. They are soft. B. They look nice. C. They taste great. D. They are juicy. 4.What was the author going to do that evening? 154 A. Go to a farm. B. Check into a hotel. C. Eat in a restaurant. D. Buy fresh vegetables. Passage 5 2015 湖南,10 分词数:388 In its early history,Chicago had floods frequently, especially in the spring, making the streets so muddy that people, horses, and carts got stuck. An old joke that was popular at the time went something like this: A man is stuck up to his waist in a muddy Chicago street. Asked if he needs help,he replies, "No,thanks. I've got a good horse under me." The city planners decided to build an underground drainage(排水)system,but there simply wasn't enough difference between the height of the ground level and the water level. The only two options were to lower the Chicago River or raise the city. An engineer named Ellis Chesbrough convinced the city that it had no choice but to build the pipes above ground and then cover them with dirt.This raised the level of the city's streets by as much as 12 feet. This of course created a new problem:dirt practically buried the first floors of every building in Chicago.Building owners were faced with a choice: either change the first floors of their buildings into basements, and the second stories into main floors, or hoist the entire buildings to meet the new street level. Small wood-frame buildings could be lifted fairly easily. But what about large, heavy structures like the Tremont Hotel, which was a six-story brick building? That's where George Pullman came in. He had developed some house-moving skills successfully. To lift a big structure like the Tremont Hotel, Pullman would place thousands of jackscrews ( 螺旋千斤顶)beneath the building's foundation. One man was assigned to operate each section of roughly 10 jackscrews. At Pullman's signal each man turned his jackscrew the same amount at the same time, thereby raising the building slowly and evenly. Astonishingly, the Tremont Hotel stayed open during the entire operation, and many of its guests didn't even notice anything was happening. Some people like to say that every problem has a solution. But in Chicago's early history, every engineering solution seemed to create a new problem. Now that Chicago's waste water was draining efficiently into the Chicago River, the city's next step was to clean the polluted river. 1. The author mentions the joke to show____. A. horses were fairly useful in Chicago B. Chicago's streets were extremely muddy 155 C. Chicago was very dangerous in the spring D. the Chicago people were particularly humorous 2. The city planners were convinced by Ellis Chesbrough to . A. get rid of the street dirt B. lower the Chicago River C. fight against heavy floods D. build the pipes above ground 3. The underlined word "hoist" in Paragraph 4 means "____". A. change B. lift C. repair D. decorate 4. What can we conclude about the moving operation of the Tremont Hotel? A. It went on smoothly as intended. B. It interrupted the business of the hotel. C. It involved Pullman turning ten jackscrews. D. It separated the building from its foundation. 5. The passage is mainly about early Chicago's____. A. popular lifestyles and their influences B. environmental disasters and their causes C. engineering problems and their solutions D. successful businessmen and their achievements Passage 6 2015 陕西,8 分词数:364 The production of coffee beans is a huge, profitable business, but, unfortunately, full sun production is taking over the industry and bringing about a lot of damage. The change in how coffee is grown from shade grown production to full sun production endangers the very existence of certain animals and birds, and even disturbs the world's ecological balance. On a local level, the damage of the forest required by full sun fields affects the area's birds and animals. The shade of the forest trees provides a home for birds and other species (物种) that depend on the trees' flowers and fruits. Full-sun coffee growers destroy this forest home. As a result, many species are quickly dying out. On a more global level, the destruction of the rainforest for full-sun coffee fields also threatens (威胁) human life. Medical research often makes use of the forests' plant and animal life, and the destruction of such species could prevent researchers from finding cures for certain diseases. In addition, new coffee growing techniques are poisoning the water locally, and 156 eventually the world's groundwater. Both locally and globally, the continued spread of fullsun coffee plantations (种植园) could mean the destruction of the rainforest ecology. The loss of shade trees is already causing a slight change in the world's climate, and studies show that the loss of oxygen-giving trees also leads to air pollution and global warming. Moreover, the new growing techniques are contributing to acidic (酸性的) soil conditions. It is obvious that the way much coffee is grown affects many aspects of life, from the local environment to the global ecology. But consumers do have a choice. They can purchase shade grown coffee whenever possible, although at a higher cost. The future health of the planet and mankind is surely worth more than an inexpensive cup of coffee. 1.What can we learn about fullsun coffee production from Paragraph 4? A.It limits the spread of new growing techniques. B. It leads to air pollution and global warming. C. It slows down the loss of shade trees. D. It improves local soil conditions. 2.The purpose of the text is to ____. A. entertain B. advertise C. instruct D. persuade 3.Where does this text probably come from? A. An agricultural magazine. B. A medical journal. C. An engineering textbook. D. A tourist guide. 4.Which of the following shows the structure of the whole text? (P:Paragraph) Passage 7 2015 重庆,8 分词数:324 There are many places to go on safari (观赏野生动物) in Africa, but riding a horse through the flooded waters of Botswana's Okavango Delta must rank as one of the world's most exciting wildlife journeys. 157 Several safari camps operate as the base for this adventure, providing unique rides twice a day to explore deep into the delta. The camps have excellent horses, professional guides and lots of support workers. They have a reputation for providing a great riding experience. The morning ride, when the guides take you to beautiful, shallow lakes full of water lilies, tends to be more active. It is unlike any other riding experience. With rainbows forming in the splashing water around you and the sound of huge drops of water bouncing off your body and face, it is truly exciting. You are very likely to come across large wild animals, too. On horseback it is possible to get quite close to elephants, giraffes and many other animals. The sense of excitement and tension levels rise suddenly though, as does your heart rate, as you move closer to them. In the evening, rides are usually at a more relaxed and unhurried pace, with golden light streaming across the grassy delta and the animals coming out to eat and drink. Sedate though they are, rides at this time of day are still very impressive. As the sun's rays pass through the dust kicked up by the horses, the romance of Africa comes to life. Back at the camp you can kick off your boots and enjoy excellent food and wine. Looking back on your day, you will find it hard to deny that a horseback safari is as close as you will ever come to answering the call of the wild. 1.What does the underlined word "They" refer to? A. Flooded waters. B. Wildlife journeys. C. Safari camps. D. Unique rides. 2.What does the author find most exciting about a horseback safari? A. Seeing and feeling the real African life. B. Enjoying good food and wine at the camp. C. Hunting large animals just as our ancestors did. D. Being part of the scene and getting close to animals. 3.What does the underlined word "Sedate" probably mean? A. Wild and romantic. B. Slow and peaceful. C. Hungry and thirsty. D. Active and excited. 4.The author introduces the riding experience in the Okavango Delta mainly by . A. following space order B. following time order C. making classifications D. giving examples Passage 8 2014 安徽,8 分词数:282 158 Should we allow modern buildings to be built next to older buildings in a historic area of a city? In order to answer this question, we must first examine whether people really want to preserve the historic feel of an area. Not all historical buildings are attractive. However, there may be other reasons — for example, economic( 经 济 的 ) reasons — why they should be preserved. So,let us assume that historical buildings are both attractive and important to the majority of people. What should we do then if a new building is needed? In my view, new architectural styles can exist perfectly well alongside an older style. Indeed, there are many examples in my own home town of Tours where modern designs have been placed very successfully next to old buildings. As long as the building in question is pleasing and does not dominate (影响) its surroundings too much, it often improves the attractiveness of the area. It is true that there are examples of new buildings which have spoilt (破坏) the area they are in, but the same can be said of some old buildings too. Yet people still speak against new buildings in historic areas. I think this is simply because people are naturally conservative (保守的) and do not like change. Although we have to respect people's feelings as fellow users of the buildings. I believe that it is the duty of the architect and planner to move things forward. If we always reproduced what was there before, we would all still be living in caves. Thus, I would argue against copying previous architectural styles and choose something fresh and different, even though that might be the more risky choice. 1.What does the author say about historical buildings in the first paragraph? A. Some of them are not attractive. B. Most of them are too expensive to preserve. C. They are more pleasing than modern buildings. D. They have nothing to do with the historic feel of an area. 2.Which of the following is true according to the author? A. We should reproduce the same old buildings. B. Buildings should not dominate their surroundings. C. Some old buildings have spoilt the area they are in. D. No one understands why people speak against new buildings. 3.By "move things forward" in the last paragraph, the author probably means "____". 159 A. destroy old buildings B. put things in a different place C. choose new architectural styles D. respect people's feelings for historical buildings 4.What is the main purpose of the passage? A. To explain why people dislike change. B. To warn that we could end up living in caves. C. To admit how new buildings have ruined their surroundings. D. To argue that modern buildings can be built in historic areas. Passage 9 2014 重庆,8 分词数:279 The idea of being able to walk on water has long interested humans greatly. Sadly, biological facts prevent us ever accomplishing such a thing without artificial aid — we simply weigh too much, and all our mass pushes down through our relatively small feet, resulting in a lot of pressure that makes us sink. However, several types of animals can walk on water. One of the most interesting is the common basilisk Basilicus basilicus, a lizard (蜥蜴) native to Central and South America. It can run across water for a distance of several meters, avoiding getting wet by rapidly hitting the water's surface with its feet. The lizard will take as many as 20 steps per second to keep moving forward. For humans to do this, we'd need huge feet that we could bring up to our ears in order to create adequate "hitting". But fortunately there is an alternative: cornflour. By adding enough of this common thickening agent to water (and it does take a lot), you can create a "non-Newtonian" liquid that doesn't behave like normal water. Now, if the surface of the water is hit hard enough, particles (粒 子) in the water group together for a moment to make the surface hard. Move quickly enough and put enough force into each step, and you really can walk across the surface of an adequately thick liquid of cornflour. Fun though all this may sound, it's still rather messy and better read about in theory than carried out in practice. If you must do it, then keep the water wings handy in case you start to sink — and take a shower afterward! 1.Walking on water hasn't become a reality mainly because humans____. 160 A. are not interested in it B. have biological limitations C. have not invented proper tools D. are afraid to make an attempt 2.What do we know about Basilicus basilicus from the passage? A. It is light enough to walk on water. B. Its huge feet enable it to stay above water. C. It can run across water at a certain speed. D. Its unique skin keeps it from getting wet in water. 3.What is the function of the cornflour according to the passage? A. To create a thick liquid. B. To turn the water into solid. C. To help the liquid behave normally. D. To enable the water to move rapidly. 4.What is the author's attitude toward the idea of humans' walking on water? A. It is risky but beneficial. B. It is interesting and worth trying. C. It is crazy and cannot become a reality. D. It is impractical though theoretically possible. Passage 10 2013 安徽,8 分词数:338 Argentina in the late nineteenth century was an exciting place. Around 1870, it was experiencing an economic (经济的) boom, and the capital, Buenos Aires, attracted many people. Farmers, as well as a flood of foreigners from Spain and Italy, came to Buenos Aires seeking jobs. These jobs didn't pay well, and the people felt lonely and disappointed with their new life in the city. As the unhappy newcomers mixed together in the poor parts of the city, the dance known as the tango (探戈舞) came into being. At the beginning the tango was a dance of the lower classes. It was danced in the bars and streets. At that time there were many fewer women than men, so if a man didn't want to be left out, his only choice was to dance with another man so that he could attract the attention of the few available women. Gradually, the dance spread into the upper classes of Argentinean society and became more respectable. In Europe at this time, strong interest in dance from around the world was beginning. This 161 interest in international dance was especially evident in Paris. Every kind of dance from ballet (芭 蕾舞) to belly dancing could be found on the stages of the Paris theaters. After tango dancers from Argentina arrived in Europe, they began to draw the interest of the public as they performed their exciting dance in cafes. Though not everyone approved of the new dance, saying it was a little too shocking, the dance did find enough supporters to make it popular. The popularity ( 流 行 ) of the tango continued to grow in many other parts of the world. Soldiers who returned to the United States from World WarⅠbrought the tango to North America. It reached Japan in 1926, and in 2003 the Argentinean embassy in Seoul hired a local tango dancer to act as a kind of dance ambassador, and promote tango dancing throughout South Korea. 1. The origin of the tango is associated with____. A. belly dancers B. American soldiers C. a Spanish city D. the capital of Argentina 2. Which of the following is true about the tango? A. It was created by foreigners from Spain and Italy. B. People of the upper classes loved the tango most. C. It was often danced by two males in the beginning. D. A dancer in Seoul became the Argentinean ambassador. 3. Before World War Ⅰ, the tango spread to ____. A. America B. Japan C. France D. South Korea 4. What can be the best title for the text? A. How to Dance the Tango B. The History of the Tango C. How to Promote the Tango D. The Modern Tango Boom Passage 11 2012 重庆,8 分词数:313 To take the apple as a forbidden fruit is the most unlikely story the Christians ( 基督教徒) have ever cooked up. For them, the forbidden fruit from Eden is evil ( 邪 恶 的 ). So when Columbus brought the tomato back from South America, a land mistakenly considered to be Eden, everyone jumped to the obvious conclusion. Wrongly taken as the apple of Eden, the tomato was shut out of the door of Europeans. What made it particularly terrifying was its similarity to the mandrake, a plant that was 162 thought to have come from Hell ( 地狱). What earned the plant its awful reputation was its roots which looked like a dried-up human body occupied by evil spirits. Though the tomato and the mandrake were quite different except that both had bright red or yellow fruit, the general population considered them one and the same, too terrible to touch. Cautious Europeans long ignored the tomato, and until the early 1700s most of the Western people continued to drag their feet. In the 1880s, the daughter of a well-known plant expert wrote that the most interesting part of an afternoon tea at her father's house had been the "introduction of this wonderful new fruit — or is it a vegetable?" As late as the twentieth century some writers still classed tomatoes with mandrakes as an "evil fruit". But in the end tomatoes carried the day. The hero of the tomato was an American named Robert Johnson,and when he was publicly going to eat the tomato in 1820, people journeyed for hundreds of miles to watch him drop dead. "What are you afraid of?" he shouted."I'll show you fools that these things are good to eat!" Then he bit into the tomato. Some people fainted. But he survived and, according to a local story, set up a tomato-canning factory. 1.The tomato was shut out of the door of early Europeans mainly because ____. A. it made Christians evil B. it was the apple of Eden C. it came from a forbidden land D. it was religiously unacceptable 2.What can we infer from the underlined part in Paragraph 3? A. The process of ignoring the tomato slowed down. B. There was little progress in the study of the tomato. C. The tomato was still refused in most western countries. D. Most western people continued to get rid of the tomato. 3.What is the main reason for Robert Johnson to eat the tomato publicly? A. To make himself a hero. B. To remove people's fear of the tomato. C. To speed up the popularity of the tomato. D. To persuade people to buy products from his factory. 4.What is the main purpose of the passage? 163 A. To challenge people's fixed concepts of the tomato. B. To give an explanation to people's dislike of the tomato. C. To present the change of people's attitudes to the tomato. D. To show the process of freeing the tomato from religious influence. 阅读理解 专题一 广告信息类答案 2018 年 Passage 1 本文主要介绍了在华盛顿特区骑自行车观光的四条路线。 1.A 细节理解题。根据第一条旅行路线介绍的最后一句"Reserve your spot before availability — and the cherry blossoms — disappear"可知,选择第一条旅行路线需要提前预订。 2.D 细节理解题。 根据第三条旅行路线介绍中的"Knowledgeable guides will entertain you with the most interesting stories about Presidents, Congress, memorials, and parks"可知,选择这条 旅行路线的游客可以在观光过程中听导游讲有趣的故事。 3.D 细节理解题。第四条旅行路线是夜间游览,根据该部分的最后一句"All riders are equipped with reflective vests and safety lights"可知,该路线为夜间骑车观光的游客提供反光背心和安 全灯。 Passage 2 本文是一篇应用文,是一则有关暑期活动的广告,该广告主要向学生推荐了四项不同风格的 活动。 1.A 细节理解题。题干中的 go camping 是关键信息,与 Outdoor Adventure(OUT)活动栏目中 的"wild camping"呼应。 2.D 细节理解题。根据题干中的 Mrs. Wilson 可定位到 WWⅠ Battlefields and Paris(WBP)这 一栏,然后根据"On Monday... After staying overnight in London, we travel on Day 2 to northern France to visit the World War Ⅰ battlefields"可知,这里的 Day 2 就是题干中的 Tuesday,由 此可知应该选 D 项。 【干扰项分析】A 项是周一(Monday)的旅游内容,B 项是周四的旅游内容,C 项是周五 的旅游内容。 3.A 细节理解题。通读 Potty about Potter(POT)一栏的内容可知,参观完 Warner Bros Studio 之后在 Streatley|on|Thames 过夜,第二天在 Oxfords Christchurch 外野餐,据此可知该活 动历时两天。 【长难句解读】Thursday sees us make the short journey to Paris where we will visit Disneyland Paris park, staying until late to see the parade and the fireworks. 分析:这是一个主从复合句。主句谓语动词 sees 后接省略 to 的不定式作宾补;Paris 是先行 词,后接 where 引导的定语从句,在该从句中,staying... fireworks 是现在分词短语作伴随状 语,to see the parade and the fireworks 是不定式短语作目的状语。 译文:星期四我们去巴黎短途旅行,在那里我们将参观巴黎迪斯尼乐园,一直待到很晚去观 看游行和烟花表演。 Passage 3 文章介绍了英国旅游胜地霍克庄园和花园。 164 1.D 细 节 理 解 题 。 根 据 Visitor Information 部 分 中 的 "How to Get to Holker" " By Car" "Manchester — 1 hour 30 minutes"可知,开车从曼彻斯特到霍克庄园需要 1 小时 30 分钟,即 90 分钟。 2.B 细节理解题。根据"Admission Charges"下面的内容可知,参观霍克庄园和花园时,团体 票(Groups)的价格为£9.00, 即每人 9 英镑。 3.D 细节理解题。根据 Special Events 部分中的"Winter Market"和"enjoying a live music show" 可知,Winter Market 为正确答案。 Passage 4 本文是一篇应用文,介绍了纽约大都会艺术博物馆的参观须知。 1.C 细节理解题。根据题意并结合 Admission 部分中的"$25.00 recommended for adults" "free for children under 12 with an adult"可知,他们应付两个成年人的门票费用。故选 C。 2.B 细节理解题。根据 The Cloisters Museum and Gardens 部分的内容可知,它吸引人的地方 在于它的收藏品可追溯到中世纪,故选 B。 【干扰项分析】根据文末的 Closed 的信息可知,它不是全年开放的,因此 A 项错误;根据 文中的"architecture of Europe in the Middle Ages"可知,它不属于现代欧洲风格,故 C 项错误; 文中并未提及它销售玻璃收藏品,因此 D 项错误。 Passage 5 本文从几个方面介绍了阿德莱德大学的消防举措。 1.C 细节理解题。第一节说到阿德莱德大学雇用的全职消防人员的职责:检查校园大楼、检 测并维护各种消防装置、在宿舍楼提供消防安全教育活动。从这些信息我们可以判断,C 项 符合题意。 【干扰项分析】A 项"给学生提供兼职工作"在文中没有提到;第一节最后一句说的是"无论 何时你搬到一个新的区域,你应该确定火警报警器以及离你房间最近的两个出口的位置",B 项与文章内容不符;D 项张冠李戴,文章说的是提供消防安全教育活动,而不是培训老师成 为消防专业人员。 2.B 细节理解题。根据题干中的"the automatic fire alarm systems",我们把答案定位在第二节 Fire Alarms 部分。该节内容说,火灾自动报警系统包括感温探测器、烟雾探测器和喷水灭 火装置(heat detectors, smoke detectors and sprinklers),故 B 项符合本节内容。 【干扰项分析】pipes 和 fire alarm pull stations 是火灾手动报警器的组成部分,而不是火灾自 动报警系统的组成部分,所以 A、C、D 三项均不符合题意。 3.C 细节理解题。根据题干我们把答案定位在第三节。该节内容说,在消防演习中,学生应 该带上房间钥匙、身份证,锁好房门,从最近的紧急出口快速疏散,不能坐电梯,在宿舍楼 外(与同学们)会合,等待进一步的指令。由以上信息我们可以判断选 C。 【干扰项分析】A 项与文中的"do not use a lift"不符;文章说的是从紧急出口疏散到宿舍楼外 等待进一步的指令,故 B 项和 D 项不正确。 4.A 细节理解题。根据题干我们把答案定位在第四节。根据第四节中的"Misuse of a fire extinguisher will result in fines"可知,灭火器使用不当会被罚款。 【干扰项分析】B、C 项曲解文意:该节说灭火器使用不当会给其他居住者带来危险(create a dangerous situation for other residents)和损坏个人财产(damage to personal property);文中没有 提到必须有培训师在场才能使用灭火器,因此 D 项属于主观臆断。 5.D 细节理解题。根据题干信息我们把答案定位在最后一节。该节说,烟雾探测器的红灯亮 了表明烟雾探测器运转正常;如果红灯不闪了就要联系楼管员;在任何情况下都不要覆盖或 165 者堵塞烟雾探测器;如果烟雾探测器在没有火和烟雾的情况下响了,要通知楼管员。从这些 信息我们可以判断选 D。 2017 年 Passage 1 这是一篇应用文,提供了游客去太平洋科学中心的一些参观指南,并鼓励游客为该中心的发 展捐款。 【段意梳理】 第一段:介绍太平洋科学中心纪念品商店的位置。 第二段:介绍太平洋科学中心咖啡馆的服务项目及位置。 第三段:介绍太平洋科学中心的供游客租用的物品的信息。 第四段:鼓励人们支持该中心的发展,为它捐款。 1.B 考查细节理解。根据文章第一段的"The store is located(位于) upstairs in Building 3 right next to the Laser Dome"可知,纪念品商店坐落于 3 号楼,故选 B。 【干扰项分析】 根据文章第二段的"The café is located upstairs in Building 1 and is open daily until one hour before Pacific Science Center closes"可知,咖啡馆在 1 号楼,故 A 项错误;根据 第一段的最后一句可排除 C 项;根据第三段的"Pushchairs and wheelchairs are available to rent at the Information Desk and Denny Way entrance. ID required"可知,在 Denny Way entrance 可 租赁到折叠式婴儿车和轮椅,故 D 项错误。 2.D 考查细节理解。根据第四段的"Today, Pacific Science Center serves more than 1.3 million people a year and brings inquiry-based science education to classrooms and community events"可 知,太平洋科学中心把科学教育带入课堂,故选 D。 3.A 考查目的意图。 根据最后一段的标题"Support Pacific Science Center"和"Visit pacificsciencecenter.org to find various ways you can support Pacific Science Center"可知,本段 旨在鼓励人们为太平洋科学中心的发展捐款。 【干扰项分析】 最后一段并没有为该中心做广告,故 B 项的陈述错误;最后一段没有介绍 特色展品,可排除 C;最后一段提到这个中心创办于 1962 年,但是本段没有介绍它的历史, 可排除 D。  Passage 2 本文是一篇应用文,主要介绍了全球的艺术家将聚集到一起用他们自己的语言表演莎士比亚 的戏剧的相关内容。 1.A 考查细节理解。根据题干中的"the National Theatre of China"可将答案定位于 National Theatre of China 部分中的"Shakespeare’s RichardⅢ",故选 A 项。 【技巧点拨】对于细节理解题主要是采用定位的方法,找到题干的关键词,并与原文的内容 相对应,就可准确地选出答案。 2.C 考查推理判断。根据 Deafinitely Theatre 部分中的"By  translating the rich and humorous text of Love’s Labour’s Lost into the physical language of BSL, Deafinitely Theatre creates a new interpretation of Shakespeare’s comedy"可知,Deafinitely Theatre 的特别 之处就是使用 BSL 表演戏剧,创造出了一种对莎士比亚喜剧的新的诠释。故选 C 项。 3.D 考查细节理解。根据题干中的"Hebrew"可将该题答案定位在最后一部分中。根据最后一 部分中的"Date & Time: Monday 28 May, 7.30pm & Tuesday 29 May, 7.30 pm"可知, 该题应选 D 项。 Passage 3 166 本文是一篇应用文,介绍了旧金山的四种消防车之旅。 1.A 考查细节理解。题干问的是三月份可参加的旅行,而第一个项目 San Francisco Winery Tour 的时间是从 2 月 1 日到 4 月 30 日 。故选 A。 2.C 考查细节理解。第二个项目 Back to the Fifties Tour 中提到在 San Francisco 你能免费品 尝冰淇淋。而其他选项在这个旅行中没有被提到。故选 C。 3.D 考查细节理解。第四个项目 Holiday Lights Tour 的最后一句话提到"Advance  reservations required",由此可知,该旅行需要提前预订。故选 D。 Passage 4 这是一篇应用文。文章主要介绍了 TOKNOW 杂志的内容、订购价格和退订等方面的信息。  1.D 考查细节理解。根据第一段中的"a unique combination of challenging ideas and good fun to young fans every month"可知,TOKNOW 杂志的特殊之处在于它将趣味性与有挑战性 的、复杂的概念结合起来。故选 D。 【干扰项分析】A 项"张冠李戴", TOKNOW 杂志的目标读者是孩子,而不是年轻的父母; B 项"颠倒是非",文中的"Well, it has no ads or promotions inside"表明 TOKNOW 杂志不 刊登广告;文中并未提及 C 项。 2.B 考查细节理解。根据 What’s inside?部分中的"Every month the magazine introduces a fresh new topic with articles, experiments and creative things to make"可知,TOKNOW 杂志刊 载关于新话题的文章。 3.D 考查细节理解。根据 Annual Subscription with Gift Pack 部分中的"Rest of World £70"可知,在中国按年订购有礼包的 TOKNOW 杂志需要£70。 4.B 考查细节理解。根据文中的"Refund Policy — the subscription can be cancelled within 28 days and you can get your money back"可知,28 天之内退订的订购者可以获得全额退款。 【长难句解读】[HT]TOKNOW makes complex ideas attractive and accessible to children, who can become involved in advanced concepts and even philosophy(哲学) — and they will soon discover that TOKNOW feels more like a club than just a magazine. 分析:这是一个主从复合句。who 引导定语从句,修饰 children;that 引导宾语从句,作 discover 的宾语。 译文:TOKNOW 杂志让复杂的概念有吸引力,容易为孩子们所理解;孩子们可接触高级概 念,甚至是哲学,而且他们将很快发现 TOKNOW 杂志不仅仅是一本杂志,而更像是一个俱 乐部。 Passage 5 这是一篇应用文。匆忙之中或心不在焉时发送电子邮件出现错误怎么办?比如,邮件还没写 好就发送了,把收件人的名字拼写错了,个人邮件被你误发给了所有人,或者把埋怨的邮件 误发给了被你吐槽的那个人。本文告诉我们出现这些情况时我们该如何补救。  1.C 考查细节理解。根据题干中的"realising an email accident"可把答案定位在第二段。该 段说,在你点击"发送"数秒钟之后你就会意识到自己的错误,结合下文的"freeze in horror and burn with shame"可知,人们意识到"邮件事故"后,可能会感觉糟糕,因此选 C。  2.A 考查细节理解。根据题干中的"written the wrong name"可把答案定位在第五段,该部 分提出建议说,此时赶紧给对方发一个简短的回复,为自己的错误道歉(apologising for your mistake),而且要注意道歉的语调(don’t handle it too lightly),这与 A 项的陈述是一致的。 3.B 考查细节理解。根据题干的内容可把答案定位至第六段,这部分提到,如果你不小心 点了"回复所有人",可能有人会就此开始回复不太友好的话,此时你最好离开电脑键盘(step away from your keyboard),以便让大家都冷静下来。这与 B 项的陈述相吻合,表示"避免继 167 续卷入(这样的对话)"。 4.D 考查细节理解。根据题干中的"offensive email"可把答案定位在最后一段。这部分提到, 在出现冒犯对方的情况时,要尽快亲自向对方道歉。故选 D。 5.C 考查主旨要义。通读全文,尤其是第三段的内容可知,本文主要介绍的是发送电子邮 件出现问题时人们该如何面对和补救,因此 C 项最能概括本文的主题。 Passage 6 本文是一则应用文,介绍了一本历史方面的书籍。本书老少皆宜,书中通过具体的事例让读 者在愉悦的阅读过程中了解不一样的历史。 1.C 考查推理判断。根据文中的"CHRONOLOGICA is a fascinating journey through time, from the foundation of Rome to the creation of the internet" 可知, 它与历史有关,再根据文中的 "CHRONOLOGICA is an informative...this book also gives an account"可知,它是一本书,故 答案为 C。 2.B 考查细节理解。根据文中的"Roman Emperor"、"Spencer Perceval"、"Thomas Edison and Alexander the Great"等可知,文章通过介绍一些具体的内容来向读者推荐这本书。 故答案为 B。 2012—2016 年 Passage 1 本文主要介绍了在过去的一百年里四名杰出的女性以及她们对社会所做出的贡献与影响。 21.A 细节理解题。根据 Jane Addams(1860—1935)部分的"Anyone who has ever been helped by a social worker has Jane Addams to thank. Addams helped the poor and worked for peace. She encouraged a sense of community(社区)by creating shelters and promoting education and services for people in need"可知, Jane Addams 因她的社会福利工作而出名。故选 A。 22.C 细节理解题。根据 Sandra Day O'Connor(1930—present) 部分的"When Sandra Day O'Connor finished third in her class at Stanford Law School, in 1952, she could not find work at a law firm because she was a woman"可知,她不能在律师事务所工作的原因是女性会受到性别 歧视。故选 C。 23.D 细节理解题。根据 Rosa Parks(1913—2005)部分的"It lasted for more than a year, and kicked off the civil-rights movement"可知,她的举动最终推动了民权运动。故选 D。 24.C 推理判断题。Jane Addams 是社会福利工作的带头人;Rachel Carson 是环境运动的发 起者;Sandra Day O'Connor 是反对性别歧视的领头人;Rosa Parks 是民权运动的推动者。由 此可见,这四位女性都是先驱者,故选 C。 Passage 2 本文介绍了四项即将举办的娱乐活动,包括相关的时间和地点以及具体的活动事项。 21.A 细节理解题。根据第一部分中的"...Jules Skye...He's going to talk about how you can find the right person to produce your music"可知,该题选 A 项。 【技巧点拨】对于细节理解题的解答主要是采用定位的方法,找到题干的关键词"have your music produced",并对比原文的内容,就可准确地选出答案。 22.B 细节理解题。根据第二部分中的"This joyful show will please everyone, from the youngest to the oldest"可知,该题选 B 项。 23.D 细节理解题。 根据文中第三部分中的"Wednesdays at Victoria Stage"可知,Simon's Workshop 每周三举行活动。故选 D 项。 24.C 细节理解题。根据第四部分中的"8.00pm—11.00pm"可知,该题选 C 项。 Passage 3 本文主要对五个网站上的歌剧及管弦乐等乐曲作品的演出时间、内容及购票等细节作了 168 介绍。 1.A 细节理解题。根据第一则信息 "Opera at Music Hall"及本段中的细节 "The Opera honors Enjoy the Arts membership discounts. Phone: 241-2742"可知答案。 2.B 细节理解题。根据第二则信息 "Chamber Orchestra" 并结合"which offers several concerts from March through June"的描述可知,四个选项中只有五月在此时间范围内。 3.C 细 节 理 解 题 。 根 据 第 四 则 信 息 "College Conservatory of Music (CCM)" 中 的 首 句 "Performances are on the main campus( 校 园 ) of the university, usually at Patricia Cobbett Theater"及下文中的"Students with I.D. cards can attend the events for free"可知答案。 4.A 细节理解题。第五则信息中提到了带座位的巨大户外剧场"Large outdoor theater with the closest seats under cover",选项 A 与此描述一致。 【长难句解读】CCM organizes a variety of events, including performances by the well-known LaSalle Quartet, CCM's Philharmonic Orchestra, and various groups of musicians presenting Baroque through modern music.分析:本句为一个含有较长介词短语的简单句, including 至句 末为介词短语,performances 后的"by..."是介词短语作后置定语,presenting Baroque through modern music 为现在分词短语作名词的后置定语。译文:CCM 组织了多样的活动,包括知 名的 LaSalle 四重奏乐团和 CCM 爱乐管弦乐团的演出,还有各种代表巴罗克风格到现代音 乐的音乐家团体所进行的演出。 Passage 4 本文对英国苹果节的举办规模、活动安排、苹果种类及种植环境等进行了介绍。 8.D 细节理解题。从第二段首句"Visiting an apple event...often taste, a wide variety of apples" 可直接获取答案。 9.C 推理判断题。A 选项"Decio 是一种新品种"与第二段中的"it can be...still in existence"不 一致,且由这句话可知 Decio 现在比较罕见,故 C 项正确;文中并未提及 Decio 的外形奇异, 故 B 选项错误;D 选项 "It has a special taste" 与第二段中的原文 "Although it doesn't taste of anything special, it's still worth a try"表述相反。 10.B 词义猜测题。根据第三段可知,Orleans Reinette 品种虽然口感好,但对种植环境和土 壤要求高,因此对于大多数喜欢它的人来说,大饱口福只是一种脱离实际的愿望,这与 B 项 "A vain hope" (徒劳的希望)一致。 11.B 写作意图题。通读全文可知,这是一篇介绍英国苹果节的文章。 【长难句解读】To people who are used to the limited choice of apples such as Golden Delicious and Royal Gala in supermarkets, it can be quite an eye opener to see the range of classical apples still in existence, such as Decio which was grown by the Romans. 分析:本句主干为 it can be quite an eye opener to see the range of classical apples still in existence,句中不定式短语作真正 的主语,it 为形式主语;该句前面为介词短语 To people,而 people 后又接了一个定语从句, 该句后面为 such as 短语,用于举例。译文:对于那些已习惯了选择超市里有限的苹果种类 如 Golden Delicious 和 Royal Gala 的人来说,目睹 Decio 这种过去由罗马人种植的传统苹果 品种如今依然存在可能会令他们大开眼界。 Passage 5 这是针对青少年的一则语言培训项目广告,文章从课程设置、评价体系、接机服务和饮 食四个方面进行了详细介绍。 36.C 细节理解题。根据 Our Courses 部分中的"Our Intensive Course builds on our Standard Course, with 10 additional lessons per week, guaranteeing the fastest possible language learning" 可知强化课程和标准课程的区别在于强化课程还包括一些额外的课程。 37.B 细节理解题。根据表格第一行可知,学生可以在周一到周五的 9 点至 12 点半上标准 169 课程。 38.A 细节理解题。根据 Evaluation 部分的第二句话"The majority of them take an online language test before starting their programme"可知,孩子们在开始语言项目学习前先进行语言 测试。 39.B 推理判断题。根据 Arrivals and Transfer 部分第一句中的"...students are taken good care of from the start through to the very end"可推知,这个项目推出了 full package,组织者会从项 目开始一直到结束全程照顾孩子们。 40.D 细节理解题。根据文章最后一句话"Depending on the type of allergies ...an extra charge may be made for providing special food"可推知,特殊饮食有可能会额外收费。 【长难句解读】We ask that you let us know of any allergies or dietary requirements as well as information about any medicines you take.分析:这是一个含有宾语从句和定语从句的复合句。 We ask 为主句,后面的部分为宾语从句,在宾语从句中又包含一个省略了 that 的定语从句 "you take",先行词为 any medicines。译文:我们请您告知您有无过敏情况、您的饮食要求 以及您的用药信息。 Passage 6 本文主要向读者介绍了一个大学图书馆的数据库的搜索功能以及关于"children"的搜索 结果。 46.A 细节理解题。从文中的"...As a basis for teaching relationship skills"及其所对应的"Circle Time for Young Children"可知,答案选 A 项。 47.D 细节理解题。从文中的"...constructing mental and physical images of what they see..."及 其所对应的"Young Children as Artists: Art and Design in the Early Years and Key Stage 1"可知, 答案选 D 项。 48.B 细节理解题。从文章 Refine Your Search 中提到的 Year Published 中的"2015(90)"可 知,数据库中搜索到 2015 年出版的书共有 90 本,所以答案选 B 项。 49.C 推理判断题。从文章中 Children with School Problems: A Physician's Manual 中的"The physician's guide to diagnosing and treating...and doctors must be..."可推知,该书的读者对象最 有可能为医生,所以选 C 项。 Passage 7 这是一个招聘广告。文章介绍了体育部的招聘要求以及联系人。 21.C 细节理解题。由文中的第一句"The Athletic Department is looking for students to help assist staff during the Fall 2016, Winter 2016-17 and Spring 2017 semesters"可知,工作从 2016 年秋天开始。所以 C 项正确。 22.C 细节理解题。根据文中的招聘条件"Good computer skills"; "Available evenings and weekends"; "Knowing basketball rules and statistics"可知,Ted 满足条件,更有可能会得到这 个工作。 23.B 细节理解题。根据文中"Rockville Athletic Coordinator, Jorge Zuniga"可知,如果申请在 Rockville 的工作需要与 Jorge 联系。所以 B 项正确。 Passage 8 本文介绍了英国几处风景名胜一日游的相关情况。 33.B 细节理解题。在 Tour B 中提到"Oxford: Includes a guided tour of England's oldest university city and colleges.",因此选 B。 34.D 细节理解题。结合本文信息可知,在 3 月 17 日这一天四个景点的票价分别为:£37, £36, £37, £33,因此最低票价在 Cambridge。 35.A 细节理解题。根据 Tour C 中的"With 500 years of history, Hampton Court was once the 170 home of four Kings and one Queen."可知,这里曾经是四个国王和一个女王的住所,因此现在 成为了一个重要的旅游胜地。 【长难句解读】Visit the palace and its various historic gardens, which include the famous maze (迷宫) where it is easy to get lost! 分析:这是一个主从复合句。定语从句 which include the famous maze 修饰先行词 historic gardens,定语从句 where it is easy to get lost 修饰先行词 maze。译文:(你可以)参观宫殿和各种历史花园,花园中有著名的迷宫,在那里(游客) 容易迷失方向! Passage 9 本文向读者介绍了一款产品——电视耳机。 60.B 推理判断题。根据文章第一段可知,这款电视耳机可以根据个人的情况调节音量,同 时其他人也能在令人舒服的音量下看电视,这样就不会影响其他人,故答案为 B 项。 61.C 细节理解题。根据第二段中的"TV Ears patented technology includes a revolutionary noise reduction ear tip, not used in any other commercially available headset." 可 知 ,a revolutionary noise reduction ear tip 让这款耳机与众不同,故选 C 项。 62.A 推理判断题。广告中引用了一位医生和一对夫妇的使用体验以及他们的推荐,这为广 告增加了说服力,故选 A 项。 Passage 10 本文介绍了大学住宿制度中的五项规定:允许和禁止的物品、宿舍的密码锁、做饭的政 策、养宠物的政策和时间管制。 36.D 细节理解题。根据第一部分中的"The following items are approved for use in residential ( 住 宿 的 ) rooms: electric blankets, hair dryers, personal computers, radios, televisions and DVD players." 可知,在学生宿舍里允许使用电热毯、电吹风、个人电脑、收音机、电视和 DVD 播放器,故选 D。 37.A 细节理解题。根据第二部分中的"The Office of Residence Life may change the door lock combination at any time at the expense of the resident if it is found that the student has shared the combination with others."可知,如果发现学生将组合密码告诉他人,the Office of Residence Life 可以随时更改房门锁的组合密码,费用将由学生承担,故选 A。 38.A 细节理解题。根据第三部分中的"With the exception of using a small microwave oven(微 波炉) to heat food, students are not permitted to cook in their rooms."可知,除了可以使用小的 微波炉来加热食物外,不允许学生在房间里做饭,由此可知选 A。 39.C 细节理解题。根据第四部分中的"If, one week from the date of written notice, the pet is not removed, the student is referred to the Student Court."可知,如果学生在接到书面通知后的一周 内没有移走宠物,他将被送交学生法庭处理,故选 C。 40.D 细节理解题。根据最后一部分中的"Quiet hours on Friday and Saturday nights are 1:00 am to 8:00 am."可知,学生应该在周五和周六的凌晨 1:00 到早上 8:00 之间保持安静,由此可 知 D 项正确。 Passage 11 本文介绍的是地铁指南,就零币兑换、地铁运行时间、老年人/残疾人优惠、特别提醒 等方面给予指导说明。 32.D 细节理解题。 依据第二段第二句可知,车站提供最大面值为 5 美元的零钱。 33.B 细节理解题。 依据原文中的"3 a.m. Fri.—Sat. nights"可推断,地铁周六凌晨三点停运。 34.B 推理判断题。文章中反复出现 SmarTrip Card,再根据 Metrobus 下的内容可知,其最 大特点是使用起来有优惠,换言之,能为持卡者省钱。 171 35.A 细节理解题。依据全文最后一句话可知,遗失物品拨打 202-962-1195。 Passage 12 文章由两则广告组成:机器人广告和手表遥控器广告。 56.B 细 节 理 解 题 。 根 据 第 一 则 广 告 中 的 首 句 "Are you having problems finishing your homework on time?" 以 及 第 三 句 "You don't need to worry if you buy a Mr. Helping Hand personal robot."可知选 B。A 项与文中的"includes long-lasting batteries"矛盾;C 项与文中的 "remembers simple instructions"不符;D 项与文中的"Your own personal robot will follow you around, putting away books and objects that you have left on the floor or bed."不符。 57.D 细节理解题。根据第二则广告中所列的该手表控制器功能的第二条"gives you a daily weather forecast"可知选 D。由文章的叙述可知,该产品可遥控电视,而不是修理电视,A 项 错误;该产品可以提醒你交作业的时间,而不是处理你的作业,B 项错误;C 项与第二则广 告中的首句"This is a watch that James Bond would be proud to wear!"不一致。 58.B 细节理解题。根据第一则广告中的"Originally (最初) sold for $ 499 "和"NOW ONLY $ 299"可知,Mr.H 现在售价仅为$ 299。故选 B。 59.C 文章出处题。根据第二则广告中的最后一句"For further information, click here."可知, 这两则广告是刊登在网站上的。故选 C。 Passage 13 无可比拟的高强性能,无与伦比的优质服务,无法想象的价格优惠,一款新型便携式加 热器惊艳亮相!机不可失,失不再来,心动不如行动,此时不买,更待何时? 56.B 段落大意题。本段提到这种加热器没有可以引起火灾的外露部件,它的外部不会烧伤 孩子或宠物,由此可知本段主要讨论的是这种加热器的安全性。 57.A 推理判断题。第三段第一句提到这种加热器不会减少室内的氧气,下一句提到其他的 加热器会因燃烧掉氧气而让你昏昏欲睡,由此可推断出这种加热器不消耗氧气,也不会让人 昏昏欲睡,故 A 项正确,C 项错误;根据第一段最后一句中的"runs almost silent"可判断出 B 项不正确;根据第二段的最后一句可知 D 项错误。故选 A。 58.D 词义猜测题。本段提到这种先进的加热器的覆盖面积多达 350 平方英尺,而其他的加 热器散发的大部分热量只集中在房间的中央,再结合本段最后一句的内容可推测出这种加热 器可以把热量"均衡地"散发到房间中,故选 D。 59.A 写作意图题。文章先是叙述了这种加热器的种种优点,接着提到了优质服务和价格折 扣,再结合最后一段的内容可知,本文意在劝说人们购买这种产品。 Passage 14 想要为孩子寻找新鲜而又刺激的地方吗?形形色色的寓教于乐的博物馆是非常理想的选 择。 56.C 细节理解题。根据"Head to a natural history museum."一段中的第二句话可知,孩子 们在这里可以了解大自然发展的历史,故 C 项正确。 57.C 细节理解题。根据"Go to a Youtheater."一段的内容可知,在这里孩子们可以看到木 偶的制作,了解舞台化妆等内容。 58.B 词义猜测题。根据"Try hands-on science."一段的内容可知,"hands-on science"的意思 是孩子们自己动手来学习和体验科学知识。 59.C 推理判断题。结合文章内容可判断出,本文可能出自一份博物馆指南。 专题二 科普研究类答案 2018 年 Passage 1 本文讨论了有关无人驾驶汽车的问题。 172 1.A 细节理解题。根据第二段中的"While much of the debate...mobility options"以及第三段中 的"Do we want to copy — or even worsen — the traffic of today with driverless cars" 可知,应该 注意随着无人驾驶汽车的出现所产生的问题。故选 A。 2.D 细 节 理 解 题 。 根 据 第 一 段 中 的 "But however long it takes...how the transformation is regulated" 可知,作者最关心自动驾驶的监管问题。故选 D。 3.A 词义猜测题。根据上下文可知,此处表示"考虑到无人驾驶技术的成本和责任与维护问 题,叫车服务(公司)将运用第一批可使用的商业无人驾驶汽车",fielded 这里引申为"使用, 应用"之意,故选 A。 4.B 观点态度题。根据第一段中的"Well, the future is apparently now",第二段中的"The arrival of driverless vehicles is a chance to make sure that those vehicles are environmentally friendly and more shared"以及最后一段中的"The coming technological advancement ...plan for it"可知,作者 对无人驾驶汽车的未来持积极正面的态度。故选 B。 【 长 难 句 解 读 】 But however long it takes, the technology has the potential to change our transportation systems and our cities, for better or for worse, depending on how the transformation is regulated. 分析:这是一个主从复合句。句中 however 引导让步状语从句,主句中 depending on how the transformation is regulated 为现在分词短语作状语,该结构中 how 引导宾语从句。 译文:但不管需要多长时间,这项技术有潜力改变我们的交通系统和城市向好或坏的方向发 展,这取决于转换是如何被监管的。 Passage 2 这是一篇说明文,介绍 3D 打印技术的新发展:打印食物,并详细介绍了与 3D 打印相关的 食品生产、可持续发展、营养,以及挑战。 1.B 推理判断题。根据题干中的 food production 可定位到第二段。这一段说,有了 3D 打印 机,厨师就能打印出复杂的巧克力雕塑等,做到这样的事需要多年的经验,而 3D 打印机很 容易做到这一点;一些餐馆使用这种技术能将厨师解放出来去完成其他工作。由此推断,3D 打印机能节省厨师的很多时间和努力,因此选 B。 【干扰项分析】A 项犯了理解错误,文章只说厨师能被解放出来完成其他工作(to complete other tasks),没有说 3D 打印机能帮助厨师创造新的菜肴;C 项属于空穴来风,文章没有提 到 3D 打印机能改善烹饪环境;D 项属于移花接木,文章说 3D 打印机能为婚礼蛋糕打印出 漂亮的饰品,而不是有助于装饰餐馆。 2.D 推理判断题。第三段谈到 3D 打印机打印食物的可持续性发展,说它能解决食物可持续 发展的问题,因为它可以使用可再生的藻类和草中的水解胶体来代替目前的烹饪原料。由此 可推知,3D 打印机使用可再生的原料作为食物来源,因此选 D。 【干扰项分析】本段提到 3D 食物打印或许有助于解决食物的可持续发展问题,因此 A 项不 对;本段没有提到 B 项"加快食品的运输"和 C 项"不需要储存食物的空间"。 3.B 细节理解题。第四段提到,3D 食物打印能让消费者按照自己的需求来打印含有定制的 (customized)营养成分的食物。因此 3D 打印的食物能满足个人的营养需求。 4.C 细节理解题。根据题干可知本题的答案应在文章最后一段。该段谈到 3D 食物打印面对 的挑战,其中最主要的问题是,打印机只能处理干燥的原料,因为肉类产品和奶类产品很容 易变质。由此可知选 C。 【干扰项分析】最后一段谈到 3D 打印机打印食物的过程很费时间,但这不是阻止 3D 打印 机打印食物的主要原因,因此 A 项错误;该段没有提及 3D 食物打印机价格昂贵,故 B 项 错误;有些专家对 3D 食物打印机持怀疑态度,认为它们更适合快餐业,与题干不符,因此 173 D 项错误。 5.A 标题判断题。本文介绍了 3D 食物打印机的相关信息,谈到其发展现状,它使用的原料 有利于食物的可持续发展,人们可以根据自己的营养需求来私人定制食物,但是其发展也受 到某些限制。因此本文的标题应该选 A 项"3D 食物打印:美味的新技术"。 【干扰项分析】本文没有提到 B 项"改善 3D 食物打印的新办法",C 项是本文的一部分内容,D 项的"3D 食物打印:从农场走向餐桌"不符合文章的内容。 【长难句解读】Currently, most ingredients must be changed to a paste(糊状物) before a printer can use them, and the printing process is quite timeconsuming, because ingredients interact with each other in very complex ways. 分析:本句是一个并列复合句。and 连接两个并列句,前一个并列分句含有 before 引导的时 间状语从句;后一并列分句含有 because 引导的原因状语从句。 译文:目前,大多数原料必须被处理成糊状物,然后 3D 食品打印机才能打印它们,而且打 印的过程很耗时,因为这些原料相互作用的方式很复杂。 2017 年 Passage 1 本文是一篇说明文。在难以获得饮用水的情况下,自制一个太阳能蒸馏器是最好的获取饮用 水的方法之一。本文主要介绍了自制太阳能蒸馏器的配件、大小及具体的使用方法等内容。 【段意梳理】 第一段:在难以获得饮用水的情况下,自制一个太阳能蒸馏器是最好的获取饮用水的 方法之一。美国农业部的两个博士发明了自制的太阳能蒸馏器。而组成该太阳能蒸馏器的部 件有塑料薄膜、塑料管和盛水的容器。 第二段&第三段:太阳能蒸馏器的具体制作方法。 第四段:太阳能蒸馏器的工作原理及人们饮用该太阳能蒸馏器中的水的方法。 1.D 考查细节理解。根据第一段的最后一句"These pieces can be folded into a neat little pack and fastened on your belt"可知,这种自制的太阳能蒸馏器便于携带。 2.B 考查推理判断。根据语境,尤其是第二段的第一句"To construct a working still, use a sharp stick or rock to dig a hole four feet across and three feet deep"可知,画线部分 the water catcher 指代前句中的 a working still,即自制的太阳能蒸馏器。 3.C 考查细节理解。根据第三段的内容,尤其是第一句"Next, cover the hole with the plastic sheet, securing the edges of the plastic with dirt and weighting the sheet’s center down with a rock"可知,做好太阳能蒸馏器装置后,还要在塑料薄膜的中心放一块石头以使塑料薄膜的 中心下垂,进而形成圆锥体的构造, 从而使该太阳能蒸馏器更好地发挥作用。 4.D 考查推理判断。根据第四段的第二句"Ground water evaporates(蒸发) and collects on the sheet until small drops of water form, run down the material, and fall off into the cup"可知,水 蒸气会附在塑料薄膜的表面,最后沿着塑料薄膜滴落到杯中。 Passage 2 本文是一篇说明文,介绍了植物在受到昆虫的攻击时是如何进行自我保护的,同时引出世界 比我们人类想象的要复杂得多这一观点。 【段意梳理】 第一段:一棵多叶的植物被攻击时,不是安静地忍受着,而是会释放出易挥发的有机化合物。 第二段:科学家发现所有的植物在受到攻击时都会释放出该化合物。 174 第三段:一些植物释放出带味的化学物质来驱赶昆虫。 第四段:经过多方的研究证明,这些释放出的化合物好像帮助了它们的邻居。 第五段:但科学家仍没有证实植物是否会互相交谈。 第六段:通过达尔文的想象说明世界比人类所感知的更为复杂。 1.D 考查细节理解。根据第一段的"young maple trees getting bitten by insects send out a particular smell that neighboring plants can get. These chemicals come from the injured parts of the plant"可知,当植物受到攻击时,它会产生一些化学物质。故选 D 项。 2.A 考查句意理解。根据第三段中的"The attacker who was lunching now becomes lunch"可知, 正吃午餐的袭击者变成了午餐,也就是袭击者被攻击了。故选 A 项。 【技巧点拨】对于句意理解题而言,应结合上下文内容充分理解。 3.B 考查推理判断。根据第三段中的"Some plants pump out smelly chemicals to keep insects away. But others do double duty"可知,植物能保护自己抵抗昆虫。故选 B 项。 4.C 考 查 推 理 判 断 。 根 据 最 后 一 段 中 的 "imagined a world far busier, noisier and more intimate(亲密的) than the world we can see and hear. Our senses are weak"可推知,世界比它显 现出来的更加复杂。故选 C 项。 【长难句解读】What the plants pump through the air is a mixture of chemicals known as volatile organic compounds, VOCs for short. 分析: 句中 What 引导主语从句,而且过去分词短语 known as volatile organic compounds 在 句中作后置定语,修饰 a mixture of chemicals。 译文:植物通过空气所输送出的是一种被称作挥发性有机化合物的混合化学物,简称 VOC。 Passage 3 本文是一篇科普说明文,介绍了智能交通团队建立移动实验室,研究老年人开车存在的问题 和难点,帮助老年人安全驾驶。 1.C 考查细节理解。根据第一段最后一句中的"in order to understand the challenges faced by older drivers and to discover where the key stress points are"可知,建立实验室的目的是要找出 老年人开车有困难的地方,故选 C。 2.A 考查细节理解。 根据第四段中的"For many older people, particularly those living alone or in the country, driving is important for preserving their independence, giving them the freedom to get out and about without having to rely on others"可知,对于老年人来说,开车对保持他们生活的独立性至关 重要,故选 A。 3.B 考查推理判断。根据第三段中的"the Newcastle team are developing in-vehicle technologies for older drivers which they hope could help them to continue driving into later life"可知,研究者 正在研发车载式的技术以帮助老年人晚年依然能开车,故选 B。 4.D 考查标题判断。本文的关键词是老年人驾车,重点讲述了为了让他们安全驾车所进行 的研究。 【干扰项分析】C 项比较容易被误选,Services 一词不恰当,概括太宽泛,偏离了文章的主 旨。 Passage 4 这是一篇议论文。随着无人驾驶车辆的逐步推广,无人驾驶车辆出现事故时的责任认定成为 一个难题。德国交通部长提出了第一套自主驾驶车辆的交通法规,并就此提出了自己的主张。 1.D 考查推理判断。根据下文"the grey area between...the driverless future"对"死亡谷"的解释, 可知这里的"死亡谷"指的是:半自动驾驶与完全无人驾驶的汽车之间的这片中间区域(界线 不明、难以界定的领域),故应指驾驶员的角色不明朗的情景,D 项描述与之吻合。 175 2.B 考查推理判断。根据第三段提到德国交通部长希望的三件事以及第四段说交通道路法 规中关于自主驾驶车辆的这种变化将会允许完全自主驾驶的车辆投入使用(permit fully automatic driving)可知,选 B。 3.D 考查推理判断。题干询问消费者(乘客)对于无人驾驶汽车的看法,在谈到谁应承担无人 驾驶车辆的责任时,第七段说,当你说无人驾驶车辆的时候,人们以为是没有司机,从这些 信息可以推断出,消费者在这种情况下会觉得没有为无人驾驶车辆的安全问题负责的人,故 选 D。 4.C 考查细节理解。在谈到无人驾驶车辆在各国的前景时,最后两段谈到在英国和新加坡, 由政府提供的无人驾驶车辆正在投入使用,但是在美国这种情况就会很糟糕(go down poorly in the US),在这里,认为政府会接管无人驾驶车辆,把它们作为对公众有益的事物,这是 完全行不通的(would get absolutely nowhere),由此推断,作者觉得这种无人驾驶车辆在美国 没有发展前景。 5.A 考查主旨大意。全文把德国交通部长的提议作为引入,讲述了无人驾驶车辆的法律责 任的认定,无人驾驶车辆在遇到车祸时首先应该保障的是车辆还是乘客的安全等问题,因此 选 A。 Passage 5 这是一篇科技说明文。文中主要介绍了人工智能(AI) 随着科技的发展可能超出人类的控 制以及人类应如何应对此类安全问题等相关信息。 【段意梳理】 第一段:人工智能有可能摆脱人类的控制。 第二段:智能机器有阻止自己被摧毁的本性。 第三段:防火墙防止机器人影响世界是不太可能的。 第四段:解决人工智能的安全问题困难重重,人们对此持有不同的观点。 【背景知识】[HT] [HTF]人工智能(Artificial Intelligence),英文缩写为 AI。它是研究、开 发用于模拟、延伸和扩展人的智能的理论、方法、技术及应用系统的一门新的技术科学。 人工智能是计算机科学的一个分支,它试图了解智能的实质,并生产出一种新的能以人类智 能相似的方式做出反应的智能机器,该领域的研究包括机器人、语言识别、图像识别、自然 语言处理和专家系统等。人工智能是对人的意识、思维的信息过程的模拟。人工智能不是人 的智能,但能像人那样思考,也可能超过人的智能。 1.A 考查推理判断。根据第一段中的"The real problem relates to the possibility that artificial intelligence(AI) may become extremely good at achieving something other than what we really want"可知,人工智能可能会变得非常擅长完成目标,而这些目标可能不是人们真正想要的, 所以人工智能有可能摆脱人的控制。 【干扰项分析】由文中的"artificial intelligence(AI) may become extremely good at achieving something other than what we really want"可知 B 项表述错误;由文中的"Hollywood’s theory that machines with evil(邪恶的) minds will drive armies of killer robots is just silly"可知,C 项表 示错误;D 项在原文没有被提及。 2.A 考查细节理解。根据第二段中的"A machine with a specific purpose has another quality, one that we usually associate with living things: a wish to preserve its own existence"可知,智能机器 可能会为保护自己的存在而阻止自己被摧毁。 3.D 考查细节理解。根据第三段中的"Some researchers argue that we can seal the machines inside a kind of firewall, using them to answer difficult questions but never allowing them to affect the real world"可知,有些研究者认为可用防火墙防止机器人影响世界。 4.C 考查推理判断。根据第四段中的"Solving the safety problem well enough to move forward 176 in AI seems to be possible but not easy"可知,人工智能的安全问题有可能得到解决,但又困难 重重。 【长难句解读】If we use, to achieve our purposes, a mechanical agency with whose operation we cannot effectively interfere ( 干 预 ), we had better be quite sure that the purpose put into the machine is the purpose which we really desire.  分析:这是一个主从复合句,句中 If 引导条件状语从句,to achieve our purposes 为不定式作 目的状语,从句中 with whose 引导定语从句,修饰先行词 agency;主句中 that 引导宾语从 句,作 sure 的宾语,which 引导定语从句,修饰先行词 purpose,put into the machine 为过去分 词短语作定语。 译文:为了达到我们的目的,如果我们使用不能有效地干预其运用的机械代理,我们最好要 很确定让机器做事的目的是我们真正希望(达到)的目的。[HT]〖KH-*1D〗〖FK)〗 2012—2016 年 Passage 1 本文讲述了科学家对大众传媒的新发现:好消息比坏消息传得更快、更远,越积极乐观 的信息越容易被大众分享。 1.A 细节理解题。根据第一段的内容尤其是"Those are the classic rules for the evening broadcasts and the morning papers"可知,新闻报道惯常报道坏消息来吸引人们的眼球。 2.C 推理判断题。根据第二段可知,大众传媒想要吸引你的眼球而不关心你的内心感受, 但是当你和朋友分享故事时,你对他们的反应要关心得多,你不希望他们把你看作 Debbie Downer。由此可以推断,Debbie Downer 和大众传媒是一样的,不会去关心他人的感受,故 选 C 项,inconsiderate 意为"不为别人着想的,不考虑他人的"。 3.B 细节理解题。根据第三段中的"One of his first findings was that articles in the science section were much more likely to make the list than non-science articles"可知,选 B 项。 4.D 标题判断题。本文第一段的最后一句概括了文章的主要内容:通过追踪人们的电子邮 件和网上的帖子,科学家已经发现好消息要比坏消息传得更快更远。下文是对这一发现的具 体介绍,故选 D 项。 【长难句解读】Researchers analyzing word-of-mouth communication — e-mails, Web posts and reviews, face-to-face conversations — found that it tended to be more positive than negative(消极 的), but that didn't necessarily mean people preferred positive news.分析:本句为连词 but 连接的 两个并列分句,第一个分句为含有一个宾语从句的复合句,主句的主语是 Researchers,其 后的现在分词短语 analyzing word-of-mouth communication 作定语,谓语是 found,后面是 that 引导的宾语从句;第二个分句也是一个含有宾语从句的复合句。译文:分析口述交际如电子 邮件、网帖、评论和面谈的研究者发现,它倾向于更积极而不是消极(的内容),但这并不 一定意味着人们更偏爱积极的消息。 177 Passage 2 一项研究表明,小时候劳动的男孩长大后更容易享受快乐的生活。 1.A 细节理解题。根据第一段中的"He had more job satisfaction, a better marriage and was healthier"可知,约翰享受工作和婚姻带来的快乐。 2.D 写作意图题。第二段的第二句话"The study showed that those...than those who had not"告 诉我们,小时候劳动的男孩要比不劳动的男孩成年后更能享受生活且更富有创造性,后面紧 接着引用 Vaillant 的话来进一步解释为什么有些男孩成年后会享受快乐的生活。 3.C 细节理解题。根据第三段的内容可知,Vaillant 的团队通过比较几组不同的得分情况得 出了他们的结论。 4.C 词义猜测题。根据画线词后面的"Those who had done the most boyhood activities...less likely to have been unemployed"可知,孩提时代的劳动和长大后的生活之间的联系是显而易 见的。 5.B 推理判断题。根据最后一段的内容可知,孩提时代的劳动能够培养一个人多方面的能 力,这些能力正是情感健康的基础,同时也能帮助他们过上更快乐的生活。由此可推断出情 感健康对享受精彩的成年生活十分重要。 【长难句解读】Boys who worked in the home or community gained competence(能力)and came to feel they were worthwhile members of society.分析:这是一个含有定语从句和宾语从句的复 合句。who worked in the home or community 为定语从句,先行词为 Boys;后面的 they were worthwhile members of society 为宾语从句,作 feel 的宾语。译文:在社区或家中劳动的男孩 能够获得能力并且感受到他们是有价值的社会成员。 Passage 3 本文探讨了科学家和六个月大的婴儿之间的关联。 1.D 推理判断题。从文章第一段的"but some developmental psychologists(心理学家) have argued that this ‘play’ is more like a scientific investigation than one might think"可知,一些发展 心理学家认为,婴儿的游戏在某种程度上类似于科学家的实验,所以选 D 项。 2.D 推理判断题。从第二段的"in the process, it brings out important evidence about...the baby's investigation and the scientist's experiment appear to share the same aim..."推断可知,婴儿收集 证据的方式类似于科学家的,故选 D 项。 178 3.C 细节理解题。从第三段的"they investigate human psychology and the rules of language using similar means...through repeated experiments..."可知,孩子们可能会通过不停地尝试他们 自己的"实验"来习得语言规则,故选 C 项。 4.B 段落大意题。从最后一段的内容尤其是"Viewing childhood development as a scientific investigation throws light on how children learn...look at science and scientists"可知,研究孩子们 的游戏可能有助于更好地理解科学,所以 B 项正确。 5.D 观点态度题。通读全文特别是最后一段的"Perhaps evolution(进化) provided..."可 知,作者的态度是很谨慎的,所以选 D 项。 Passage 4 这是一篇研究报告。文章指出在夜间取自奶牛的牛奶"night milk" 含有大量的褪黑激 素,能帮助人们入睡,减少焦虑。 1.B 细节理解题。根据第四段中的"Those given night milk, which contained 10 times the amount of melatonin, were less active and less anxious than those fed with the milk collected during daytime"可知,饮用在白天取自奶牛的牛奶的老鼠更焦虑。所以 B 项正确。 2.D 细节理解题。根据第六段中的"taking melatonin drugs has been suggested to those who are struggling to fall asleep at night"可知,褪黑激素被用在安眠药中。所以 D 项正确。 3.A 主旨要义题。根据第一段中的"But now a study has found it really does help people nod off — if it is milked from a cow at night"、第二段以及第五段内容可知,全文围绕"Night Milk and Sleep"展开,告诉读者饮用在晚上取自奶牛的牛奶有助于睡眠。所以 A 项正确。 4.D 推理判断题。通读全文可知,文中作者通过提供研究结果来支持文章的主题。所以 D 项正确。 【长难句解读】Those given night milk, which contained 10 times the amount of melatonin, were less active and less anxious than those fed with the milk collected during daytime, according to the study published in The Journal of Medicinal Food. 分析:这是一个主从复合句。given night milk 为过去分词短语作定语,修饰 Those; which contained 10 times the amount of melatonin 为 which 引导的非限制性定语从句,先行词为 night milk; fed with the milk collected during daytime 为过去分词短语作定语,该短语中 collected during daytime 为另一个过去分词短语作定语,修饰 milk。 译文:由发表在《药物强化性食品杂志》上的研究可知,饮用在夜间取自奶牛的牛奶的老鼠 比那些饮用在白天取自奶牛的牛奶的老鼠更不活跃、更不焦虑。在夜间取自奶牛的牛奶里所 179 含的褪黑激素是在白天取自奶牛的牛奶的十倍。 Passage 5 科学家经过实验发现与黑猩猩相比,合作是人类与生俱来的一种独有的本能。 1.A 细节理解题。根据第一段中的"they have little instinct (本能)to help one another"和下文 的实验可知黑猩猩很少关心别的黑猩猩,只顾自己。因此选项 A 正确。 2.A 细 节 理 解 题 。 根 据 第 三 段 首 句 "Human children, on the other hand, are naturally cooperative"和下文对于实验结果的描述可知帮助别人是儿童的本能,因此选择 A。 3.C 主旨大意题。文章前两段说明了黑猩猩的自私,第三段作者话锋一转,提出了本文要 说明的中心:Human children, on the other hand, are naturally cooperative。因此选项 C 正确。 Passage 6 本文介绍了目前在法国流行的主题咖啡馆产生的背景和对消费者起到的心理治疗的作用。 1.D 细节理解题。第一段中的"Here they are learning to get in touch with their true feelings."一 句表明,the cafe La Chope 鼓励人们表达自己的真实情感。 2.A 推理判断题。根据第二段可知,多年来,咖啡馆受到人们生活方式变化的影响。由文 中的"longer working hours, a fast-food boom and a younger generation's desire to spend more time at home"可推知人们(现在)不经常去咖啡馆了。 3.C 推理判断题。根据第二段第三句中的"It's trying to help the city's troubled neighborhood cafes"可知,主题咖啡馆的目的就是拯救日趋没落的咖啡行业。 4.B 推理判断题。根据最后一段可知,"心理咖啡馆"之所以变得受欢迎是因为在这里举办 讨论活动的心理学家能抚慰人们的心灵。文中的"People have few real friends. And they need to open up."暗示来这里的顾客需要精神支柱。 Passage 7 最新研究表明,房屋的窗帘、灯光设计、墙壁颜色、餐具大小等都会影响食欲和食量, 进而影响节食效果和身材。 1.B 推理判断题。依据第一段中的"Your house may have an effect on your figure...whether you pack on the pounds or keep them off.",再结合该段最后一句"Here are some ways to turn your home into part of your diet plan."可知,B 项正确。 2.B 细节理解题。依据第三段第三句"In one study, people who ate meals in a blue room consumed 33 percent less than those in a yellow or red room."可知,在蓝色环境中人们的食量会 180 减少 33%,因此身处蓝色的环境中有助于减少食量。 3.C 推理判断题。依据第四段的内容可知,吃饭吃得快的话会吃得多,因此为了帮助减缓 吃饭的速度,吃饭时可播放舒缓音乐,排除 A、B 项;D 项与第二段首句的内容相矛盾;依 据第五段的内容可知,使用小型的餐具有助于减少食物摄入量。故选 C。 4.A 标题判断题。本文主要讲的是房屋的窗帘、灯光设计、墙壁颜色、餐具大小等均会影 响食欲和食量,进而影响节食效果和身材,故 A 项"你的房屋在使你发胖吗?"作文章标题最 佳。 【长难句解读】Dark environments are more likely to encourage overeating, for people are often less self-conscious (难为情) when they're in poorly lit places — and so more likely to eat lots of food. 分析:这是一个主从复合句。for 为并列连词,表示原因,when 在句中引导时间状 语从句。 译文:昏暗的环境更有可能刺激人们暴饮暴食,因为在这种光线暗淡的情况下 人们往往会不那么难为情,所以就更有可能吃大量的食物。 Passage 8 作者介绍了几种吸引顾客的策略,包括商品的摆放、气味、背景音乐等。 1.C 细节理解题。依据第二段的第二、三句可知,商家知道那些购买新鲜蔬菜、水果的人 随后会乐于买更多的垃圾食品,因此商家这样摆放的目的是想销售更多的垃圾食品。 2.D 细节理解题。依据第三段可知,烘焙食物的气味能激发人们的购买欲望,因此商家在 早上把食物烘焙好,让气味在商场飘荡,以促使人们更多地消费。 3.D 推理判断题。依据尾段的第二句和尾句可知,因为有好的环境,顾客很快决定买房, 因此让顾客有一个良好的第一印象能增加销量。 4.A 写作目的题。文章介绍了商家采用的不同的吸引人们消费的策略,因此选 A 项。 Passage 9 文章对体型渺小,实则有着庞大团队和巨大力量的蚂蚁进行了分析,指出了它们在地球 上存活时间比恐龙还长的原因。 1.B 细节理解题。根据第一段中的"They also live nearly everywhere, except on frozen mountain tops and around the poles"可知,答案为 B 项。 2.B 细节理解题。根据第二段中的"Where we use sound and sight to communicate, ants depend primarily on pheromones (外激素)"可知,答案为 B 项。 3.D 词义猜测题。根据第三段中的"They are completely fearless and will readily take on a 181 creature much larger than themselves, attacking in large groups and overcoming their target"可知, 蚂蚁无所畏惧,敢与比自己大得多的动物较量。因而,答案为 D 项,fight against"对抗"。 4.A 推理判断题。根据文中最后一段中的"Behaving in this selfless and devoted manner, these little creatures have survived on Earth..."可知答案。 Passage 10 本文论述了生气的表现形式、表现种类,脑部区域的变化及由此产生的行为方式。 1.C 细节理解题。根据第二段的最后两句可知,表达愤怒方式之一——决斗,受到文化和 社会的双重影响,因此选择 C。 2.B 细节理解题。根据第三段中的"But when we are angry the EEG...aren't balanced"可知,当 我们生气时,左额叶前部和右额叶前部区域脑电图活动会不平衡,故 B 项正确。 3.A 细节理解题。根据最后一段中的"The angrier we are, the more likely we are to move towards the object of our anger" 和"This approach-and-confront behaviour is accompanied by a leftward prefrontal asymmetry(不对称) of EEG activity"可知选择 A。 4.D 段落大意题。根据最后一段中的"But anger is an exception to this pattern"和最后一段所 述内容可知,最后一段的重点内容是关于愤怒的行为模式,因此 D 项正确。 Passage 11 文章通过科学研究发现,美国人和法国人对于饮食的不同态度导致了美国人比法国人更 胖,所以美国人更热衷于减肥。 1.C 细节理解题。根据文章第二段中的"the French see eating as an important part of their lifestyle"可知,法国人把饮食看做他们生活方式中的一个重要的部分。故答案为 C。 2.B 主旨大意题。根据文章第二段可知,文章主要说明了美国人和法国人对于饮食的不同 态度,同时最后一段也说明了现在饮食的变化,这些都说明了生活方式与肥胖之间的关系。 故答案为 B。 3.A 文章结构题。文章第二段中用 the French 和 Americans 对饮食的态度进行对比,同时在 文章最后一段又对这两个国家的肥胖比率进行了对比,故可以判断出,作者主要使用了"对 比"的写作手法。故答案为 A。 4.C 文章出处题。本文主要讨论了由于美国人和法国人对于饮食的不同态度而导致了体重 上的差异,所以可以推断出,文章应该来自健康方面的话题。故答案为 C。 Passage 12 德国的一项研究表明,与悲观者相比,对自己的未来过于乐观的人在未来 10 年更容易 182 出现残疾或死亡。该研究还发现,年轻人对自己未来生活的满意度估计过高,中年人预测相 对较准确,而老年人往往低估自己未来生活的满意度。 1.B 细节理解题。根据第四段内容可知,18—39 岁的年轻人会高估自己未来生活的满意度, 65 岁及以上年龄的人往往会低估自己未来生活的满意度,而 40—64 岁的中年人则对自己未 来生活的满意度估计更准确一些。 2.C 推理判断题。根据第六段和第七段内容可知,对自己未来悲观的人比乐天派对自己的 行为更加小心,这样的人可能会及时采取措施应对潜在的危险,因此选 C。 3.A 推理判断题。根据第八段中的第一句话可知,与身体不好或收入不高的人相比,身体 好或者收入高的受访者往往预料自己身体垮得更厉害或收入明显减少。因此选 A。 4.D 推理判断题。文章开头提到不同年龄段的人对自己未来生活满意度的估计不同;从文 章接下来的内容可知,年轻人更乐观,中年人较实际,老年人更悲观,由此可推断出对未来 生活满意度估计的程度是随着年龄而递减的,因此选 D。 Passage 13 习惯了生活在温暖舒适的住所里的宠物如何安全地应对寒冷的天气呢?身为主人,你该 做些什么?本文给出了答案。 1.D 细节理解题。根据本段第二句的内容可知,宠物喜欢待在温暖的场所。 2.A 推理判断题。根据第二段中的"When you're cold enough to go inside, they probably are too."可推知,这样做是为了了解何时把它们带到温暖的场所。 3.A 推理判断题。根据最后一段最后一句的内容可知,它们可能会缺少清洁的水。 4.B 主旨大意题。第一段最后一句点明了文章的中心,再根据本文内容可知,本文意在提 供实用的意见。 专题三 生态环保类答案 2018 年 Passage 1 该文指出生活中常见的过时电子设备其实作用小、耗能高,并提出了解决方案。 1.A 推理判断题。根据对第一段的整体理解可知,与新的电子设备相比,过时电子设备不仅 耗能高,而且危害环境,所以可推断出,新的电子设备是环保的,故选 A 项。 【干扰项分析】根据文章第一段中的"as these outdated devices consume much more energy than the newer ones that do the same things"可知,新的电子设备耗能低,比旧设备好,故 B、 C 项错;本文未提及电子设备的过时速度快,故 D 项错。 183 2.D 细节理解题。根据文章第二段第一句中的"To figure out how much power these devices are using, Callie Babbitt and her colleagues at the Rochester Institute of Technology in New York tracked the environmental costs for each product throughout its life"可知,Babbitt 的团队做这 项研究的目的是计算电子设备的耗电情况,故选 D 项。 3.B 细节理解题。根据文章最后一段最后一句"They found that more on|demand entertainment viewing on tablets instead of TVs and desktop computers could cut energy consumption by 44%" 可知,用平板电脑替换电视机和台式电脑会节省 44%的能耗,根据第二段中的"LCD TVs entered ... showed up in 2007"可知,平板电脑应比液晶电视耗能低,故选 B 项。 4.A 推理判断题。根据文章最后一段第二句中的"the researchers also explored what would happen if consumers replaced old products with new electronics that serve more than one function"可推断出,文章建议用多功能的新设备来替换过时设备,即停止使用过时设备,故 选 A 项。take apart"拆卸";upgrade"使升级";recycle"回收利用"。 【长难句解读】The teams data only went up to 2007, but the researchers also explored what would happen if consumers replaced old products with new electronics that serve more than one function, such as a tablet for word processing and TV viewing. 分析:本句是一个并列句。but 是表转折的并列连词,连接两个分句。what 引导宾语从句, that 引导定语从句。 译文:虽然这个团队的数据只到 2007 年,但研究人员也探索了如果消费者用多功能的新电 子设备(比如可进行文字处理和观看电视的平板电脑)来替换过时设备将会如何。 Passage 2 本文介绍一种吃塑料的大蜡螟幼虫,它胃中的酶能够分解塑料,为解决塑料污染提供了新的 途径。 1.C 细节理解题。根据第一段中的"a new study suggests an answer may lie in the stomachs of some hungry worms"以及第二段中的"Researchers in Spain and England recently found that the worms of the greater wax moth can break down polyethylene, ...and the worms consumed and broke down about 92 milligrams, or almost 3% of it"可知大蜡螟幼虫能吃塑料,故选 C。 【干扰项分析】根据第三段中的"the worms ability to break down their everyday food — beeswax"可知,大蜡螟幼虫的日常食物是 beeswax,所以不能选 A 项;第三段中的"The wax worm evolved a method or system to break this bond"只谈及大蜡螟幼虫吃塑料是一种进化现 184 象,不是讲大蜡螟幼虫是一种进化后的新物种,不能选 B 项。 2.B 细节理解题。根据第四段中的"The next step, DeBruyn says, will be to identify the cause of the breakdown. Is it an enzyme produced by the worm itself or by its gut microbes(肠道微生物)?" 可知,DeBruyn 认为研究的下一步要找出酶的来源。故选 B。 3.D 推理判断题。根据第五段中的"But she expects using the chemical in some kind of industrial process — not simply ‘millions of worms thrown on top of the plastic.’" 可知,将来可能会在 工厂生产酶这种化学物。故选 D。 4.C 写作意图题。文章第一、二段提出有关大蜡螟幼虫分解塑料垃圾的研究发现,全文围绕 该话题展开,介绍一种分解塑料的方法,故选 C。 【长难句解读】Researchers in Spain and England recently found that the worms of the greater wax moth can break down polyethylene, which accounts for 40% of plastics. 分析:这是一个主从复合句。句中 that 引导宾语从句,从句中含有一个 which 引导的非限制 性定语从句。break down"分解", account for"(在数量、比例上)占"。 译文:西班牙和英国的研究人员最近发现,大蜡螟幼虫可以分解聚乙烯,而聚乙烯占塑料的 40%。 2017 年 Passage 1 这是一篇记叙文,讲述了作者作为一个野生动物救助和教育机构的志愿者,成功救助一只猫 头鹰雏鸟的经历。 【段意梳理】 第一段:作者描述担任野生动物救助和教育机构的志愿者的苦与乐。 第二段:作者接到一位女士的电话,让他去救助一只猫头鹰雏鸟。 第三段:这只猫头鹰雏鸟似乎没有受伤,因为作者还没找到它的巢,所以作者决定暂时给它 安个新家。 第四段:这家人提供了一个铁丝筐,作者放进去一些树枝并建好了鸟巢,这只猫头鹰 雏鸟很快在巢中安静下来。 第五段:作者想方设法让这只猫头鹰雏鸟的父母出现。 第六段:第二天,这只猫头鹰雏鸟的父母终于出现了。 1.A 考查推理判断。根据第一段的"Trying to help injured, displaced or sick creatures can be 185 heartbreaking; survival is never certain"可知,救助野生动物有时会让人感到心碎,因为即使努 力挽救它们,也不能保证它们都会活下来。因此答案为 A。 【干扰项分析】 文中没有提到在救助动物的过程中受伤,故 B 项错误;原文中说的是不确 定救助的动物能不能存活,故 C 项错误;文中没有提到动物被赶出家园,故 D 项错误。 2.C 考查细节理解。根据第二段的"I got a rescue call from a woman in Muttontown. She had found a young owl(猫头鹰) on the ground"可知,这位女士给作者打电话是为了让他来救助一 只猫头鹰雏鸟。 【干扰项分析】 短文中没有信息支撑选项A、B,故可排除;根据第三段的第一句"I examined the chick(雏鸟) and it seemed fine"可知这只猫头鹰雏鸟并未受伤,故可排除 D。  3.A 考查细节理解。根据第四段的"I put some pine branches into the basket to make this nest safe and comfortable... it quickly calmed down"可知,作者做的鸟巢非常舒服,所以把这只猫 头鹰雏鸟放进去后,它很快安静下来。 【干扰项分析】 文中没有提到给它喂食,可排除 B;根据第五段可知,作者后来让这家主 人播放饥饿的猫头鹰雏鸟发出的叫声的录音,是为了找到它的父母,因此可排除 C;当时作 者还没有找到它的父母,可排除 D。 4.B 考查推理判断。根据最后一段可知,通过播放猫头鹰雏鸟饥饿时发出的叫声的录音, 这只猫头鹰雏鸟最终与它的父母团聚,而且它的父母开始给它喂食,再结合第一段最后一句 "However, when it works, it is simply beautiful"可判断,作者付出的努力没有白费,因而结果 是美好的。 【长难句解读】 I gave the owner as much information as possible and headed home to see what news the night might bring. 分析:本句是一个复合句。and 连接两个并列的谓语部分;动词 see 后为 what 引导的宾语从 句。 译文:我给(这家)主人提供尽量多的信息,然后走向回家的路,看看这个晚上可能会有什 么消息。 Passage 2 本文是一篇说明文。在争议了几年之后黄石国家公园终于把灰狼重新引入公园,以保持公园 内生态的平衡。 【段意梳理】 186 第一段:在争议了几年之后黄石国家公园把 14 头在加拿大捕获的灰狼重新引入公园。 第二段:黄石公园里的灰狼在过去因受人类影响而逐渐消失。 第三段:灰狼的消失在黄石公园引发的一系列生态问题。 第四段:早在 1966 年,生物学家就提议在黄石公园重新引入灰狼。 第五段:灰狼重新引入这一工程的效果。 1.D 考查主旨大意。纵观全文可知,本文主要谈了将灰狼重新引入黄石国家公园的原因以 及这一举措的成效等,故选 D。 2.C 考查词义猜测。由第二段第一句话可知,曾经黄石公园里有灰狼,下文提到但后来由 于人类的发展灰狼数量逐渐减少,几乎消失了,因此可以推出,displaced 意为"被迫离开", 故选 C。 3.A 考查推理判断。第三段提到由于灰狼的消失,鹿和麋鹿的数量快速增长,消耗了大量的 植被,减少了植物种类同时丛林狼的数量增多,园内很多的红狐被猎杀,而河狸也完全消失 了,由此推断出灰狼的消失对公园内的生态环境产生了损害。故选 A。 4.B 考查观点态度。根据第五段最后一句中的"a valuable experiment"可知,作者认为这项 工程很有价值,因此他的态度应当是积极肯定的。故选 B。 2012—2016 年 Passage 1 美国加利福尼亚州的秃鹫因为各种原因濒临灭绝,但在科研人员的帮助下,秃鹫的种群 逐渐恢复生机。 1.D 推理判断题。文章没有直接说美国加利福尼亚州秃鹫引起了研究人员的兴趣,但是结 合第一段和下文的"So scientists have come up with a shocking idea"可推断,因为电线和铅中 毒,20 世纪 80 年代加利福尼亚州秃鹫濒临灭绝,因此它们引起了科研人员的兴趣。 2.B 细节理解题。从题干中的 electrical lines 可知,答案应定位在第三段。电线能使秃鹫丧 命,因为秃鹫晚上休息的时候看不见电线,而如果它们宽大的翅膀同时搭在两根电线上,它 们就会被电死,因此选 B。 3.D 细节理解题。第五段讲述了铅中毒对秃鹫的危害:影响秃鹫的神经系统(A 错),影响 秃鹫繁衍后代的能力(D 正确),导致秃鹫肾衰竭(B 错)。科研人员给秃鹫使用一种化学药物, 帮助它们排出了体内的铅(C 错)。因此选 D。 4.C 主旨大意题。本文介绍了美国加州秃鹫濒临灭绝的原因以及科研人员为挽救秃鹫而开 展的研究工作。另外,文章的标题也直接表明了本文的主题:保护秃鹫的工作正让秃鹫以令 187 人惊讶的速度恢复生机。因此选 C。 【长难句解读】So condors with high levels of lead are sent to Los Angeles Zoo, where they are treated with calcium EDTA, a chemical that removes lead from the blood over several days.分析: 本句的主语部分有 with 短语作定语,后面 where 引导的非限制性定语从句修饰先行词 Los Angeles Zoo;where 引导的非限制性定语从句中包含了一个 that 引导的定语从句,先行词为 a chemical。译文:于是,体内含有高剂量的铅的秃鹫被送往洛杉矶动物园。在那里,这些秃 鹫得到一种名为 EDTA 的钙物质的治疗。这是一种化学药物,能够在几天时间内清除血液 中的铅。 Passage 2 厄尔尼诺现象的发生给全球带来了巨大的影响,作者通过具体的数据呼吁人们做好防范 和准备工作。 1.D 细节理解题。根据第一段中的"El Nio sees warm water, collected over several years in the western Pacific, flow back eastwards when winds that normally blow westwards weaken, or sometimes the other way round"可知,厄尔尼诺现象发生时会有洋流的变化,因此 D 项正确。 2.C 细节理解题。根据文章第二段中的"Rich countries gain more from powerful Nios, on balance, than they lose"可知,富裕的国家的收益大于损失,因此选择 C。 3.A 推理判断题。根据文中的"however, just 12% of disaster-relief funding in the past two decades has gone on reducing risks in advance"与"This is despite evidence that a dollar spent on risk-reduction saves at least two on reconstruction"可知,这些数据说明了减少风险的投入太少, 政府应该加大这方面的投入,因此答案为 A。 4.D 写作意图题。根据最后一段中的"Since the poorest are least likely to make up for their losses from disasters linked to El Nio, reducing their losses needs to be the priority"可知,作者 通过此文呼吁人们为厄尔尼诺现象提前做准备,因此 D 项正确。 Passage 3 当今社会人们习惯了有光的生活,但是过多的人造光造成了光污染,这不仅给动物带来 了危害,也给人类制造了麻烦。因此,我们应该关注光污染问题。 1.B 细节理解题。由第一段第二句"Instead, we are diurnal creatures, with eyes adapted to living in the sun's light"可知,人类习惯生活在有光的环境中,是昼行生物,因此选 B。 2.A 代词指代题。联系上文中的"what we've done to the night"可知,it 指代的是 the night, 因此选 A。 188 3.B 推理判断题。联系第四、五段的内容可知,作者列举鸟类和青蛙的例子是为了说明光 污染给动物带来的危害,因此选 B。 4.D 推理判断题。由最后一段中的"... lose sight of our true place in the universe, to forget the scale of our being..."可知,光污染让人类忘记了自己在宇宙中的位置,因此人类应该反省自 己,审视自己的位置,所以选 D。 5.C 标题判断题。本文主要围绕光污染这个话题展开,本没有光的黑夜却被人造光充斥着, 这对动物和人类都造成了危害。所以 C 项作文章标题比较贴切。 Passage 4 本文介绍了英国达特穆尔草原的马驹保护项目。 1.C 细节理解题。根据第三段中的"By feeding the ponies, tourists increase the risk of them getting hit by a car, and make them harder to gather"可知,不让游客喂马驹是为了保护这里的 马驹免遭危险。故选 C。 2.B 细节理解题。第四段提到每年一度的马驹迁移的目的:把马驹聚集在一起,检查马驹 的健康状况;让小马驹不再吃母乳;跑出限制区域的马驹能够返回正确的区域;把有些马驹 卖掉,以按照 Natural England 的规定控制马驹的数量。四个选项中只有 B 项是马驹迁移的 目的之一。 3.D 细节理解题。根据第五段的内容可知,当作者看到这头马驹在地上打滚的时候,他以 为它只是为了嬉戏,后来才开始为它担心,并且打电话联系相关的官员,因此选 D。 4.B 推理判断题。根据最后一段中的"Many people are working hard...one of Dartmoor's most financially-troubled elements"可以判断,很多人都在努力保护这些马驹,帮忙想办法为达特 穆尔的这个饱受资金困扰的项目寻找可持续发展的未来。由此可推知,该保护项目缺少资金, 保护这里的马驹会花费很多钱,因此选 B。 Passage 5 本文介绍了环保意识在美国从无到有,逐渐增强,环保教育势在必行。 1.C 细节理解题。依据第一段第二句"We didn't know at that time that there even was an environment, let alone that there was a problem with it"可知,1970 年以前美国人对环境问题所 知甚少。 2.A 细节理解题。依据第二段第二句中的"especially millions of grass-roots Americans are taking part in the movement"可知,数以百万计的草根美国人才是支持环保的中流砥柱。 3.D 推理判断题。依据第二段后半部分推断,美国政府多管齐下,减少了污染。A 项错在 189 lowest,B 项错在 have settled, C 项错在 forty。 4.A 推理判断题。依据最后一段内容推断可知,环保意识教育至关重要。 Passage 6 本文论述了房屋使用者的行为对节能减排的重要性,号召人们规范行为,共同保护环境。 1.B 细节理解题。根据第一段第一句和最后一句可知,房屋使用者的行为对节能减排起着 至关重要的作用。 2.D 代词指代题。研究表明,人们在家中使用的能源和生活的方式在很大程度上已被提高 能源利用效率的现有努力所忽视,这些努力反而被集中于农业和技术的发展。画线词所在句 是一个非限制性定语从句,which 指代前文中的 existing efforts。 3.A 推理判断题。根据第三段中的"consumption patterns of building users can defeat the most careful design"可知,Katy Janda 旨在强调改变房屋使用者的消费模式的重要性。 4.A 细节理解题。根据第四段第四句中的"Feedback (反馈)facilities, like smart meters and energy monitors, could help bridge this information gap"可知,反馈设备,如智能仪表和能源监 测器,可以帮助填补这一信息缺口。 5.C 细 节 理 解 题 。 根 据 倒 数 第 二 段 中 的 "Social science research has added a further dimension( 方面), suggesting that individuals' behaviour in the home can be personal and cannot be predicted"可知,在家里的个人行为是无法预测的。 Passage 7 这是一篇关于人与环境的文章。人们热衷于低科技,将它应用于自己的生活与工作中, 减轻压力,舒缓心情,带来愉悦。 1.D 推理判断题。由文章第二段可推断出选项 D 正确。网络公司们发现了低科技的好处, 因此才会去加以利用"to take advantage of it"。故选 D。 2.A 细节理解题。由第二段第一句中的"a concept associated with the natural world"和最后一 句 "Other companies are using a broader interpretation(阐释) of low technology that focuses on nature"可知 A 项符合。 3.B 段落大意题。由第五段的第一句"This craft-based theory is rooted in history"可知,以手工 艺为基础的理论历史悠久、根深蒂固,因此选项 B"有着重视艺术和手工艺的传统"正确。 4.A 作者态度题。A 项 positive"积极的";B 项 defensive "防卫的"; C 项 cautious"谨慎的 ";D 项 doubtful"怀疑的"。通读全文特别是文章的最后一句可知,作者看好、支持低科技, 相信其能够造福人类,因此选 A。 190 5.C 标题判断题。A"辉煌的曾经,未来的愿望";B"虚拟的世界,真实的挑战";C"高科技 公司,低科技办公";D"多技艺,少创意"。通读全文可知,全文主要讲的是现在一些高科技 公司都在关注 low technology,因此选 C。 Passage 8 2011 年 4 月 22 日作者一家人决定开始践行绿色生活,让我们看看他们是如何做的吧。 1.A 标题概括题。文章第一段有两处出现了 going green,且下文也多次出现,本文主要讲 述的就是作者一家是如何开始践行绿色生活的,所以 A 项最适合作为文章标题。 2.D 细节理解题。根据第二段的第二句"This meant doing 365 different green things, and it also meant challenging ourselves to go green beyond the easy things."可知 D 项正确。 3.A 细节理解题。Our Green Year 这个关键信息可以在倒数第二段找到,该段说作者一家 完全改变了生活方式,如在有机商店购物,少吃肉,尽量少买不需要的东西,不用化学清洁 剂等,这些都是环保的习惯,与 A 项的概括相一致。故选 A。 4.B 推理判断题。文章最后一段中出现了 grateful,believe 和 do have the power 等关键词,这 些都暗示出作者夫妇在将来可能还会继续他们的环保生活。 Passage 9 文章阐述了人类活动对水资源的隐性影响。 1.C 细节理解题。根据第一段最后一句"The more roads and parking lots we pave, the less water can flow into the ground to become groundwater."可知选 C。 2.B 推理判断题。根据第二段首句"Human activity is not responsible for all water shortages (短 缺)."可排除 A 项。根据第三段最后一句"This shows how much we depend on water to live, but there's a lot we can do to lower the number."可知 D 项不正确。该句中的 the number 指的是用 水量,而不是家庭的人数,C 项不正确。根据全文的内容可推知,由于存在水浪费、水污染 现象,并且水需求不断增长,所以要解决水资源短缺矛盾需要付出大量的努力。故选 B。 3.D 写作意图题。本段首句"You can take steps to save water in your home."是主题句,接下 来从几个方面举例说明了节约水资源的方法。显然,本段主要是讨论我们在家里应该如何节 约用水。故选 D。 4.D 主旨大意题。通读全文可知,本文为议论文,讨论的是人类活动对水资源的隐性影响。 故选 D。 Passage 10 荒野是否应该开发?仁者见仁,智者见智,两种观点针锋相对,莫衷一是。作者客观分 191 析,冷静思考,期待着事态的发展。 1.B 推理判断题。第二段的最后一句提到 John Sauven 认为"生态系统服务"远比开发的利益 重要,再结合本段的叙述可推断他的观点是开发荒野有害。 2.C 段落大意题。本段第二、三句是对另外一种观点的概括,意思是荒野确实提供有用的 服务但这不能作为不开发荒野的理由。 3.A 作者态度题。作者在最后两段中并没有直接表明自己的观点,只是"客观"地进行叙述。 4.D 文章结构题。文章第一段提出中心论点;第二、三两段分别阐述了两种不同的观点; 第四段是作者自己的观点;最后一段是文章的结论。 专题四 社会生活类答案 2018 年 Passage 1 本文介绍了在英国的一档名为 Save Money: Good Food 的电视节目。主持人 Susanna Reid 和厨师 Matt Tebbutt 向观众介绍如何少花钱做出营养丰富的、可口的食物。 【熟词生义】storm 常用义:n.暴风雨,暴风雪 习语新义:cook up a storm 起劲地烹调 1.B 推理判断题。根据第一段中的"but she is cooking up a storm in her latest role — showing families how to prepare delicious and nutritious meals on a tight budget"可推知,这位电视节目 主持人正在主持一档新的节目,介绍如何少花钱做出好吃的饭菜。 【干扰项分析】A 项和 C 项的表述与第一段中的"Good Morning Britains Susanna Reid is used to grilling guests on the sofa every morning"意思不符,故可排除。文章中提及这个节目能 够帮助预算紧张的家庭做出好吃的饭菜,但并没有信息说明 Susanna Reid 的家庭预算紧张, 故可排除 D 项。 【长难句解读】In Save Money: Good Food, she visits a different home each week and with the help of chef Matt Tebbutt offers top tips on how to reduce food waste, while preparing recipes for under £5 per family a day. 分析:本句中 how to reduce food waste 是"疑问词+动词不定式"作宾语,while doing 结构为 状语从句的省略。 译文:在 Save Money: Good Food 节目中,她每周拜访不同的家庭,并在厨师 Matt Tebbutt 的协助下,针对如何减少食物浪费为他们提出建议,同时为他们准备每天花不到 5 英镑(就 可以做一顿好吃的饭菜的)食谱。 192 2.C 细节理解题。根据第二段的第一句"In Save Money: Good Food, she visits a different home each week and with the help of chef Matt Tebbutt offers top tips on how to reduce food waste, while preparing recipes for under £5 per family a day"可知,在节目中,厨师 Matt Tebbutt 帮 助 Susanna Reid 给家庭提出关于如何减少食物浪费的建议和准备食谱。 3.C 写作意图题。上文介绍了 Save Money: Good Food 这个电视节目的情况,本段介绍了与 此相关的节目 Save Money: Good Health 的情况,旨在提供一些背景信息。 【干扰项分析】本段不是对前三段内容的概括,可排除 A 项;本段没有给读者提出建议, 可排除 B 项;本段的目的不是介绍一个新的讨论话题,故可排除 D 项。 4.D 标题判断题。本文介绍了由 Susanna Reid 主持的一档电视节目,给观众介绍如何少花钱 做出好吃的又有营养的饭菜,因此本文的最佳标题应为 Cooking Well for Less。 Passage 2 本文主要介绍了全球语言多样性的发展变化以及现在所面临的危机。 1.B 推理判断题。根据第一段的最后一句"Some language experts believe that 10,000 years ago, when the world had just five to ten million people, they spoke perhaps 12,000 languages between them"可知,在狩猎时代,人口少,语言却很丰富,故选 B。 【干扰项分析】根据第一段第二句"When the world was still populated by hunter|gatherers, small, tightly knit( 联 系 ) groups developed their own patterns of speech independent of each other"可知,在狩猎时代,小的紧密联系的群体发展了各自独立的语言,故 C、D 项错误; 根据第一段的内容可知,文中未提及狩猎时代语言的发展速度,可排除 A 项。 2.C 词义猜测题。根据第二段最后一句"In recent centuries ... all have caused many languages to disappear, and dominant languages such as English, Spanish and Chinese are increasingly taking over"可知,近几个世纪以来,随着社会的发展,很多语言已经消失,英语、西班牙语、汉 语逐渐占主导地位。complex"复杂的";advanced"高级的,先进的";powerful"有影响力的"; modern"现代的"。故选 C。 3.B 数字计算题。根据第三段最后一句"The median number (中位数) of speakers is a mere 6,000, which means that half the worlds languages are spoken by fewer people than that"可知, 目前全球不同语言使用人数的中位数仅是 6 000,这就意味着世界上有一半的语言的使用人 数不到 6 000;结合第三段第一句"At present, the world has about 6,800 languages"可知,选 B。 4.C 主旨大意题。根据第一段第一句中的"but in recent times there has been less coming and a 193 lot more going"和第二段可知,随着人类社会的发展,尤其是工业化、全球化以来,很多语 言已经消失,即人类的发展使得语言的种类越来越少。故选 C。 【干扰项分析】本文并未提及新的语言将会被创造,可排除 A 项;本文并未提及人们的生 活方式反映在语言上,可排除 B 项;本文并未说明地理决定语言的进化发展,可排除 D 项。 【长难句解读】The median number (中位数) of speakers is a mere 6,000, which means that half the worlds languages are spoken by fewer people than that. 分析:本句是一个包含非限制性定语从句的复合句,which 代指逗号前的整个句子。 译文:全球不同语言使用人数的中位数仅是 6 000,这意味着世界上有一半的语言的使用人 数不到 6 000。 Passage 3 人们爱七月,因为七月是收获的季节,各种浆果和核果喜获丰收。那么如何加工食用这些果 实呢?这些果实各自又含有哪些营养成分呢? 1.C 细节理解题。根据第二段的倒数第二句可知应该选 C 项,樱桃很美味。have a pleasant taste 与 be so delicious 表义相同。 【干扰项分析】A 项是 raspberries 的特征;B 项是对 yellow and orange stone fruits 的特征的曲 解;D 项是 blueberries 的特征。 2.B 细节理解题。根据第三段倒数第二句"If you like, a squeeze of fresh lemon juice on the bananas will prevent them turning brown"可知,把新鲜的柠檬汁挤到香蕉上会防止它变色。 【干扰项分析】其他选项主观臆断,与本文所述内容不符。 3.D 词义猜测题。根据画线部分下文中的"they love feeding... into the top of the machine and watching the ice cream come out below"可推断出,juicer 应该是"榨汁机"的意思。 【干扰项分析】考生很容易误选 B 项,误认为画线部分是一种饮料,其实本段最后一句中 的"the machine"应该是猜测的关键,定冠词 the 表示上文提到过的事物,这样就可以直接推 出画线部分指代的是什么。 4.B 文章出处题。全文主要介绍了一些七月成熟的果实以及这些果实的营养成分等,而这自 然与健康饮食有关,介绍这类内容的文章应该是刊载在健康杂志上的。 【干扰项分析】考生很容易误选 C 项,这是受到了第二段对营养成分的介绍的干扰,因此 误认为本文摘自研究性论文。 194 Passage 4 研究表明,闲聊对于人际交往起着重要的作用。 1.C 细节理解题。文章第一段描述了在公共场所人们之间不互相交流的现象,本段中的 "uncomfortable silence"与选项 C 中的"Absence of communication"相呼应。 【干扰项分析】A、B 两项是对原文的曲解,文章提到 smartphones 和一些公共场所中人们 的行为只是为了说明人们不互相交流。 2.B 细节理解题。根据第三段中的"The key to successful small talk is learning how to connect with others, not just communicate with them"可知,Carducci 认为成功的闲聊的关键在于学会 与其他人沟通。 3.D 细节理解题。根据倒数第二段第三句"The results showed that those who chatted with their server reported significantly higher positive feelings and a better coffee shop experience"可知,闲 聊使得人们感觉良好。故选 D 项。 【干扰项分析】题干中的 coffee|shop study 是关键词,如果定位错误则会脱离文章,从而主观 臆断,误选 B 项和 C 项。 4.C 标题判断题。通读全文可知,文章主要讨论了闲聊在人际交往中的重要作用。故 C 项作 本文标题最佳。 【干扰项分析】B 项干扰性很强,但是 B 项是片面的,文章的主旨并不是介绍如何进行闲 聊。 【长难句解读】Weve all been there: in a lift, in line at the bank or on an airplane, surrounded by people who are, like us, deeply focused on their smartphones or, worse, struggling with the uncomfortable silence. 分析:这是一个主从复合句。冒号后面的内容用来解释 there 的具体内容;surrounded by people 是过去分词短语作状语;people 后接 who 引导的定语从句,从句中 like us 作插入语, 从句包含 or 连接的两个并列结构。 译文:我们都曾处于这样的场合——在电梯里,在银行(排队的)队列里或在飞机上,我们 周围的人,像我们一样,认真地关注着他们的智能手机,或者更糟的是,他们在与令人不安 的沉寂作斗争。 Passage 5 本文通过事例说明如何引导孩子主动捐献或变卖玩具,并从玩耍简单玩具中获得快乐的方法。 195 1.A 句意猜测题。B 项 "Enough is enough."意为:(认为不应再继续)够了,行了,适可而 止。C 项 "More money, more worries."意为:钱多烦恼多。D 项 "Earn more and spend more." 意为:挣多花多。 2.C 事实细节题。根据第二段中的 "She chose to sell a few larger objects that were less often used when we promised to put the money into her school fund(基金)" 可知,当得知卖玩具所 得费用将并入她的上学基金时,女儿就挑选了一些平时不经常玩的较大玩具卖了。A 项属张 冠李戴,第二段第一句话(I found the pre|holidays a good time to encourage young children to donate less|used things, and it worked)说的是节前是一个鼓励孩子捐赠物品的好时机,而非攒 钱度假。B 项属张冠李戴,是为自己的学习基金而不是为一个贫穷的女孩攒钱。D 项为曲解 文意。第二段中 "Georgia did decide to donate a large bag of toys to a little girl whose mother was unable to pay for her holiday due to illness "说的是由于患病,小女孩的妈妈不能给她支付假期 所需的费用。 3.A 事实细节题。根据第三段中的 "Yesterday, I sat with my son, Shepherd, determined to test my own theory on this"可知作者与儿子进行球类游戏的目的是验证自己的想法:如何让简约 的生活方式成为一种习惯?A 选项 To try out an idea.(测试想法。)与文中表述一致,实为 语义替换。B 项和 C 项均为无中生有。D 项 To help him start a hobby.(帮助他开始一个爱 好。),与文中说的 How do we make it a habit for them?(我们如何才能使其成为一种习惯?) 不一致。 4.C 标题归纳题。全文围绕 "Live More with Less" (简约生活过得更充实美好)提出论点, 逐层展开论述,最后得出论点的正确性。A 项 Take it or Leave it (要么接受,要么放弃), 文章并未讨论两种非此即彼的选择,故错误。B 项 A Lesson from Kids (源于孩子的教训) 属过度概括。D 项 The Pleasure of Giving (给予的乐趣),属曲解文意,语篇的重点并非是 聚焦给予。 【长难句解读】She chose to sell a few larger objects that were less often used when we promised to put the money into her school fund ( 基 金) (our kindergarten daughter is serious about becoming a doctor). 分析:本句为一个包含由 when 引出的时间状语从句的复合句,主句中的宾语为 to sell a few larger objects that were less often used,其中 objects 后为 that 引导的定语从句, that 在定语从 句中作主语。 译文:当我们承诺卖玩具所得费用将并入她的上学基金时,女儿就挑选了一些平时不经常玩 196 的较大玩具卖了,我们上学前班的女儿对于以后当医生的想法可是认真的。 2017 年 Passage 1 本文讲述的是拥有 75 年历史的广场剧院在放映完最后一部电影后即将关闭以及市民对它的 不舍之情。 【段意梳理】 第一段:广场剧院在放映完最后一部电影后,清洁工的善后工作标志着剧院的关闭。 第二段:剧院的老板亲自选定了最后一部放映的电影,观众们含泪向剧院告别。 第三段:介绍关闭剧院的多种原因。 第四段:介绍剧院所在地今后的用途。 第五段:观众向剧院告别。 1.B 考查推理判断。根据第一段的最后一句"As one group of workers carried out the rubbish, another group began removing seats and other theater equipment in preparation for the building’s end"可推断出,工人们清理垃圾和清除座位及其他剧院设备是在为剧院的关闭作准备。故选 B。 2.D 考查细节理解。根据第二段中的"Theater owner Ed Bradford said he chose the movie because it seemed appropriate"可知,剧院的老板亲自选定了The Last Picture Show这部电 影,因为他认为这部电影很适合。原文中的"appropriate"和 D 选项的"suitable"都表示"合适 的"。故选 D。 3.C 考查推理判断。第四段最后一句说剧院被卖给了当地的开发公司,该公司计划在剧院 所在地建综合购物大楼,因此可推断出剧院将被推倒拆除。故选 C。 4.B 考查推理判断。根据第二段第二句"Though the movie is 30 years old, most of the 250 seats were filled with teary-eyed audience wanting to say good-bye to the old building"可知,观众含泪 观看最后一部电影,想向剧院告别;再结合最后一段最后一句"The theater will be missed"可 以推断出观众对剧院恋恋不舍,与剧院告别让他们感到很悲伤。故选 B。 【长难句解读】Bradford sold the building and land to a local development firm, which plans to build a shopping complex on the land where the theater is located. 分析:本句含有两个定语从句。which 引导第一个定语从句,先行词是 firm;where 引导 第二个定语从句,先行词是 land。 197 译文:Bradford 把剧院和土地卖给了当地的开发公司,该公司计划在剧院所在地建一座综合 购物大楼。 Passage 2 这是一篇说明文。文章介绍了近年来曾经致命且几乎消失了的麻疹卷土重来。究其原因,是 少数人拒绝注射疫苗,导致了自己及其他人的健康受损。作者呼吁人人都应该注射麻疹疫苗。 【段意梳理】 第一段:麻疹卷土重来。 第二至三段: 不接种疫苗的危害。 第四段:美国的麻疹爆发。 第五至七段:人们对接种疫苗的抵制及一些州采取的措施。 第八段:接种疫苗的重要性。 【背景知识】麻疹是儿童最常见的急性呼吸道传染病之一,其传染性很强,在人口密集而未 普种疫苗的地区易发生流行,2~3 年一次大流行。麻疹病毒属副黏液病毒,通过呼吸道分 泌物飞沫传播,常并发呼吸道疾病如中耳炎、喉炎、气管炎、肺炎等。我国自 1965 年开始 普种麻疹减毒活疫苗,麻疹的发病显著下降。 1.A 考查推理判断。根据前两段的内容,尤其是第二段中的"The numbers might sound small, but they are the leading edge of a dangerous trend"可知,少量的麻疹病例却能引起危险的趋势。 2.C 考查细节理解。根据第三段中的"But herd immunity works only when nearly the whole herd joins in"可知,只有在大家都参加疫苗接种的情况下群体免疫力才有作用。 【干扰项分析】A 选项"颠倒是非",由第六段的"Making things worse are state laws that make it too easy to opt out(决定不参加)..."可知 A 项错误;B 项文章未提及;根据第七段 "Now, several...But no one does enough to limit exemptions"可知 D 项错误。 3.D 考查细节理解。根据全文内容,尤其是第一段和第六段中的"Making things worse are state laws that make it too easy to opt out(决定不参加) of what are supposed to be required vaccines for all children entering kindergarten"可知,麻疹死灰复燃的主要原因是一些人不参加疫苗接 种。 4.C 考查写作意图。根据最后一段中的"Everyone enjoys the life-saving benefits vaccines provide, but they’ll exist only as long as everyone shares in the risks"并结合文中所述的麻疹死 灰复燃的事实可知,作者的目的是强调麻疹疫苗接种的重要性。 198 【长难句解读】Making things worse are state laws that make it too easy to opt out(决定不参加) of what are supposed to be required vaccines for all children entering kindergarten. 分析:这是一个主从复合句,句中 that 引导定语从句,先行词为 laws;what 引导宾语从 句;entering kindergarten 为现在分词短语作后置定语。 译文:把事情弄得更糟的是各州法律,它们很容易让本应该被要求接种疫苗的上幼儿园的所 有儿童不参加疫苗接种。 Passage 3 这是一篇记叙文。作者在外出旅游的时候一个陌生人闯入她选好的取景点,这让作者意识到, 我们都有对美的追求,这些美丽的风景把陌生人联系到了一起。 1.B 考查细节理解。由第二段可知,作者找到一个取景的地方准备拍下全景的时候,一个 女士从她身后过来,然后就站在她前面,也在那里欣赏风景,因此选 B。 2.A 考查细节理解。由第三段中的"She seemed so content in her observation"可知,那位女士 似乎在怡然地观赏风景,因此选 A。 3.C 考查细节理解。根据第四段的最后两句可知,正是因为那位女士站在镜头里欣赏风景, 所以拍摄的照片有了生命力,因此选 C。 4.D 考查推理判断。第六段说,我们都生活在彼此的空间里,我们都欣赏美,都有分享快 乐的共同渴望,因此选 D。 5.A 考查推理判断。本文通过讲述作者一次旅游时拍照的经历告诉我们:我们都有对美的追 求。因此本文是作者对一次旅游拍照经历的遐思。 2012—2016 年 Passage 1 祖父母搬去子女身边照顾小孩是如今的一种趋势,但这是否应该成为每一个祖父母的选 择呢? 1.A 细节理解题。根据第一段的最后一句"Today all three generations regard the move as a success, giving them a closer relationship than they would have had in separate cities"可知,三代 人将这一搬家视为一种成功,因为它使一家人的关系更为密切。故选 A。 2.D 细节理解题。根据第二段的"Yet there is evidence suggesting that the trend is growing. Even President Obama's mother-in-law, Marian Robinson, has agreed to leave Chicago and move into the White House to help care for her granddaughters...the example of Obama's family"可知, 大多 数人认为 Marian Robinson 搬进白宫的决定是符合潮流的,故选 D。 199 3.C 推理判断题。根据第三段的"In the 1960s we were all a little wild and couldn't get away from home far enough..."可知,20 世纪 60 年代,人们想离开父母居住。故选 C。 4.A 细 节 理 解 题 。 根 据 最 后 一 段 "Almost every grandparent wants to be with his or her grandchildren and is willing to make sacrifices, but sometimes it is wiser to say no and visit frequently instead. Having your...may be harder"可知,作者建议祖父母做决定时考虑他们自己 的利益。故选 A。 【长难句解读】Having your grandchildren far away is hard, especially knowing your adult child is struggling, but giving up the life you know may be harder.分析:but 前面是第一分句,but 后 面是第二分句。动名词短语 Having your grandchildren far away 是第一分句的主语,现在分词 短语 especially knowing your adult child is struggling 是第一分句中的状语;动名词短语 giving up the life you know 是第二分句的主语,you know 是定语从句,修饰先行词 the life。译文: 与孩子们距离远是痛苦的,尤其是知道你的成年子女日子艰难,但是放弃你熟悉的生活可能 会更艰难。 Passage 2 本文介绍了 BookCrossing.com 网站帮助喜爱读书的人们互相交流和传递图书的内容。 1.B 推理判断题。根据第一段的"Now, the website BookCrossing.com turns the page on the traditional idea of a book group"可知,作者提到 book groups 是为了引出 BookCrossing。故选 B 项。 2.A 代词指代题。根据第二段中的"hoping that the book will have an adventure, traveling far and wide..."可知,此处的"it"指的是"the book"。故选 A 项。 【技巧点拨】代词指代题的答案出处一定是在上文,也就是代词指代上文出现的名词。it 指代单数名词,而 them 或 they 指代复数名词。 3.C 推理判断题。根据第二段中的"BookCrossing provides an identification number to stick inside the book. Then the person leaves it in a public place, hoping that the book will have an adventure, traveling far and wide with each new reader who finds it" 可 知 , 读 完 书 之 后 , BookCrossers 将会把它继续传递给其他的读者。故选 C 项。 4.D 主旨大意题。根据第一段的最后一句"Now, the website BookCrossing.com turns the page on the traditional idea of a book group" 以 及 其 余 的 内 容 可 知 , 本 篇 文 章 主 要 介 绍 了 BookCrossing.com 网站的功能及其创建的意义。故选 D 项。 【技巧点拨】主旨大意题的解答要注意文章的首段和尾段,同时结合文章中的要点进行总结。 200 【长难句解读】Then the person leaves it in a public place, hoping that the book will have an adventure, traveling far and wide with each new reader who finds it. 分析:句中现在分词短语 hoping that...在句中作状语,且其中的 that 引导宾语从句,且从句 中的现在分词短语 traveling...在从句中充当状语,而 who 引导定语从句,修饰 each new reader。 译文:然后,这个人把它放在一个公共场所,希望这本书会经历一次冒险,与发现它的每一 位新读者到处旅行。 Passage 3 大学生走出了家门,还没有进入社会,因此他们的大学生活是从受家庭庇护到成为自主 承担起成年人职责的人的一个过渡时期。但是随着手机、邮件和一些社交媒体的应用,大学 生依旧受到家庭的管控和庇护。作者就这个现象探讨校方应该如何处理这个问题。 1.B 推理判断题。题干的 continued parental guidance 让我们把答案定位在第一、二段。作 者首先说大学时光应该让学生有自主性的发展,但是如今很多大学生没有肩负起成年人的责 任;第二段分析原因,说这些都是因为家长在孩子读大学之后还通过手机等方式帮助子女, 结果,大学就成了与家庭一样的环境。从这些信息判断应选 B,作者对于这种现象是不赞成 的。 2.A 词义猜测题。从第二段画线词所在的语境看,此处指从家庭的庇护到有自主性和成年 人的责任之间的转变,因此该词的含义是 A,"转变"。 3.D 推理判断题。题干的"大学的角色"是本文后面部分讨论的话题。第四段说,如果学生 依靠管理者来规范他们的社交行为和思维模式,他们就不能面对寻找自己身份带来的挑战 (A 错,应该是鼓励他们 finding an identity);第五段说,大学监管和塑造学生的行为表现会 引起学生的反感(B 错);最后一段说,每个大学关于社会价值观、社会风气和行为的讨论应 该注重学生的自主性和自我规范以及学生既要寻求安全又想自我发现这二者之间的矛盾(C 错,D 正确)。 4.C 推理判断题。本题考查对文章篇章结构的推断。本文一共有七段,第一段指出本文要 讨论的现象:现在的大学生活与以往不同,很多大学生没有承担起成年人的责任。第二段分 析原因,后面探讨各方应该采取的做法。第三、四、五段是一个话题,说大学的管理者应该 做什么。第六段探讨在大学生活时代,我们应该如何承担起角色的转变,承担起成年人的责 任。最后一段总结全文,回归主题。因此本文的篇章结构应该是 C 项。 Passage 4 本文作者结合自身经历,向读者介绍了摆脱启动疲惫和执行疲惫从而获得成功、实现人 201 生价值的方法。 1.A 细节理解题。根据第二段中的"we keep putting off a task...the longer we delay it, the more tired we feel"可知应选 A,keep putting off a task 与 A 项的 delay tasks 照应。 2.D 细节理解题。根据第三段的第二句"The solution is...always handle the most difficult job first"可知,优先解决最困难的任务是避免启动疲劳的方法。 3.B 细节理解题。根据第五段的第三句"Its difficulties appear so great that, however hard we work, we fail again and again"可知,当所有的尝试都失败后,人就会感到疲惫。 4.D 推理判断题。倒数第三段的最后一句"Relieved, I sat back in an easy chair and fell asleep" 以及倒数第二段的"An hour later, I woke up suddenly with the solution clearly in mind"告诉我 们,当陷入困境时,不妨暂时放松休息,过一段时间之后,情况自会有所好转,在无意识的 状态下可能会找到正确的方法。 5.C 标题归纳题。文章首尾呼应,主题鲜明,主要介绍了摆脱启动疲惫和执行疲惫从而获 得成功的方法。 Passage 5 很多人都喜欢背后说人闲话。本文主要介绍了人们说闲话的原因以及说闲话给人们带来 的影响。 1.A 推理判断题。根据文章开头的对话内容和下文内容可知,作者使用对话的主要目的在 于引出本文的话题"人们为什么喜欢说闲话",故选 A 项。 2.D 推理判断题。从文章第三段中的"I have noticed three effects of gossip... in a group"和第 四段中的"An important negative effect of gossip is that...talked about"可推知,说闲话的一个 重要的负面影响是给被别人说闲话的人带来不愉快的体验,所以选 D 项。 3.A 细节理解题。从第三段中的"it can give gossipers a strange kind of satisfaction"和第五段 中的"gossip is satisfying because it gives people a sense of belonging..."可知,很多人喜欢说闲 话是因为他们可以从中获得某种满足感,所以选 A 项。 4.B 细节理解题。从全文倒数第二段中的"Professor David Wilson explains that gossip is important in policing behaviors in a group"可知,说闲话会帮助群体成员观察自身的行为习惯。 所以选 B 项。 5.D 推理判断题。从最后一段中的"The next time you feel the urge to spread the latest news, think about why you want to gossip..."可推断出,作者建议我们在说闲话时要慎重考虑,所以 选 D 项。 202 Passage 6 本文主要介绍的是在不同的文化中,食物在人们庆祝节日、家庭聚会等中都起到了非常 重要的作用,代表着团结、成功、兴旺等。 1.B 细节理解题。根据第一段中的"Sharing bread or other foods is a common human tradition that can promote unity and trust"可知答案为 B 项。 2.B 细节理解题。根据第三段中的"A coin is put into the cake, which signifies (预示) success in the New Year for the person who receives it"可知答案为 B 项。 3.A 推理判断题。作者是通过举例来说明食物在节日庆祝中的作用的,比如:在第三段中 分别以美国南部、希腊为例,而第四段中则以中国为例。 4.D 主旨大意题。第一段的中心句是"Food can also have a specific meaning,and play a significant role in a family or culture's celebrations or traditions";第三段的中心句是"Food also plays an important role in many New Year celebrations" ;第四段的中心句是"Many cultures have ceremonies to celebrate the birth of a child,and food can play a significant role"。综上所述, 可知答案为 D 项。 【长难句解读】A coin is put into the cake, which signifies (预示) success in the New Year for the person who receives it.分析:本句为主从复合句。主句是 A coin is put into the cake。定语 从句有两个:一个是 which signifies success;另外一个是 who receives it。译文:一枚硬币被 放进蛋糕里,这对拿到它的人来说预示着在新的一年里会取得成功。 Passage 7 英国是一个阶级分明的国家,但是近年来,很多作家都认为英国的阶级意识变弱了,甚 至说英国是一个无阶级的社会。真的是这样吗? 1.D 细节理解题。根据第三段第二句对调查结果的陈述可知,阶级意识在当代英国仍然很 普遍。故选 D。 2.B 词义猜测题。根据上文的"it remains an important part of British society"可知,英国人似 乎喜欢"(社会的)分层"。 3.D 细节理解题。 根据第四段中的"However,...placed some regional accents as the most attractive and BBC English as the least" 可知答案。 4.A 细节理解题。根据第四段中的"This suggests that British attitudes towards accent have deep roots and are based on class prejudice"可知,A 项切题。 5.C 主旨大意题。本文主要讲的是阶级区分在英国社会中的普遍性。第三段中的"yet it 203 remains an important part of British society"也是提示。 Passage 8 根据文化相对主义,艺术作品的价值反映了当地的社会经济条件。好的艺术作品会永久 受人们欢迎。 1.C 细节理解题。依据文章第一句话可知,根据文化相对主义,艺术作品的价值是当地社 会经济条件的反映。 2.A 推理判断题。第二段谈到了莎士比亚的作品被世人用多种语言表演,莫扎特的音乐在 日本的音乐大厅演奏等, 这说明伟大的艺术作品是没有国界的。 3.D 细节理解题。依据第三段可知,David Hume 认为人们的欣赏原则是不变的,有些艺术 作品的价值是会永远存在的,这些作品会永久被人们喜欢。 4.A 标题判断题。文章一开始就告诉读者艺术的价值是什么, 因此 A 项作为本文标题最 合适。 Passage 9 本文介绍了耶鲁大学的科学家 Mark Turin 为保护濒临消失的语言而做出的不懈努力。 1.B 细节理解题。第一段说到下个世纪,很多语言将濒临消失。根据第二段中的"In an effort to prevent language loss"可知,很多学者成立了一些机构,致力于记录即将消失的语言和它 们所体现的文化。 2.A 词义猜测题。根据第二段的内容可知,有很多学者一直致力于语言和文化的保护工作。 再结合第三段的第一句话可知,耶鲁大学的 Mark Turin 也在做这件事,即记录语言,防止 语言的消失。 3.D 细节理解题。根据第三段的最后一句可知,Mark Turin 的书是以自己在尼泊尔的一个 村子里的个人经历为基础的。 4.C 推理判断题。根据文章内容可知,Mark Turin 正在从事的工作是保护即将消失的语言。 因此他要搜集语言、保护语言、然后将其重新连接起来。文中出现了 documenting, record,其 实就是指他搜集(collect)语言;文章第五段中的"in need of care and protection"与选项中的 protect 相照应;最后一句出现了 reconnected。因此最佳答案为 C。 Passage 10 本文介绍了中国互惠生(Chinese au pairs)在美国越来越受到青睐。 1.D 词义猜测题。结合语境可知 au pair 通常指的是住在外国家庭中,帮忙做些家事以换取 膳宿并学习语言的女留学生,又称互惠生。 204 2.D 细节理解题。依据第三段第二句中的"Because I am Chinese, my husband and I wanted the children to keep exposed to (接触) the language and culture"得知她们这样做的目的是让孩子接 触到中国文化。 3.A 推理判断题。依据最后一段尤其是最后一句"It is expected that American demand for au pairs will continue to rise in the next few years"可推断在美国学习汉语热潮会继续高涨。 Passage 11 1.D 细节理解题。根据第一段中的"Englishmen had been referring to insects as bugs...in the nineteenth and twentieth centuries"可知,美国人和英国人在十八世纪都用过 bug 这个词。 2.D 词义猜测题。根据画线词后面的"In 1878 he explained bugs as ‘little problems and difficulties’"可知 flaw 与 fault 同义。 3.B 主旨大意题。通读全文可知,整篇文章就是在向我们介绍 bug 一词的演变,故 B 项正 确。 Passage 12 文章就不同文化中对于颜色含义的理解以及颜色在商业、生活中的作用展开了说明。 1.A 细节理解题。根据第二段的主题句"The answer depends largely on cultural values as well as personal experiences."并结合该段内容可知,此段讲述的是在不同文化中人们对颜色的理 解,故选 A。 2.C 推理判断题。根据第四段最后一句可知,商业网站选用红色作为"现在就买"的按钮的 颜色是因为红色容易吸引顾客的眼球,可以鼓励顾客购买商品。 3.C 细节理解题。根据第五段中的"...blue can cause people to lose appetite. So if you want to eat less, some suggest that eating from blue plates can help."可知,蓝色可以减少人的食欲,有助于 减肥。 4.D 标题概括题。本文就不同文化中对于颜色含义的理解以及颜色在商业、生活中的作用 展开了说明,故 D 项最适合作本文标题。 专题五 时事新闻类答案 2018 年 Passage  本文报道了中国建筑设计师王澍在其作品中弘扬中国传统文化,最终荣获素有"诺贝尔建筑 奖"之称的普利兹克国际奖。 1.B 推理判断题。根据第二段 "Wang Shu, a 49|year|old Chinese architect, won the 2012 205 Pritzker Architecture Prize — which is often referred to as the Nobel Prize in architecture — on February 28" 可知,中国 49 岁的建筑师王澍获得了 2012 年普利兹克国际奖,即他的作品赢 得了国际认同和肯定,故选 B。A 项曲解文意。王澍获奖并不意味着中国的建筑设计师正紧 跟世界最新潮流,而是表明中国建筑设计师在作品中创新使用中国传统的建筑艺术。C 项和 D 项均属无中生有,与文中信息不符。 2.C 事实细节题。根据第四段"The style of the campus is quite different from that of most Chinese universities"和 "The curves(曲线)of the buildings perfectly match the rise and fall of hills, forming a unique view"可知,杭州香山学院的建筑风格与中国其他大学的校园建筑有很 大差异,建筑曲线与山峦起伏完美融合,形成了一道独特的风景。故选 C。A 项 "Its hilly environment."(丘陵环境)曲解文意。B 项、D 项属无中生有。 3.D 事实细节题。根据第五段 "This creation attracted a lot of attention thanks to its mixture of modern and traditional Chinese elements"可知,王澍的作品之所以引起了广泛的关注,是因为 在作品中他将中国建筑的传统和现代元素进行了有机融合,构成了传统与现代的和谐,故选 D。 A 项曲解文意,是传统与现代元素的融合而非不同形状的混合。B 项指平衡中、西建筑文 化。C 项为时尚技术的运用。 4.D 意图态度题。根据最后一段信息 "The study of traditions should be combined with practice. Otherwise, the recreation of traditions would be artificial and empty, he said"可知,中国的传统应 该赋予新的时代内涵,与时俱进,否则就会是空洞的。A 项 "Spread them (Chinese traditions) to the world"(向世界传播中国传统),属无中生有。B 项将其保存在博物馆,属颠倒是非。C 项在大学里教授中国传统,属张冠李戴,文章以大学校园为例说明建筑艺术的规划,而非教 授传统建筑艺术。 2017 年 Passage  Terrafugia 公司研制出飞行汽车并成功进行了第一次飞行,使飞行汽车成为现实更进一步。 1.A 考查段落大意。第一段的第二句"The vehicle...like a car"介绍了飞行汽车 Transition 的构 造,第一段的后半部分介绍了它的空中飞行速度、路地行驶速度和油耗情况,这些都是飞行 汽车的基本数据。故选 A 项。 2.C 考查细节理解。根据第二段中的"But don’t expect it to show up in too many driveways. It’ s expected to cost $ 279,000"可知,该飞行汽车是非常贵的,所以不太可能大量出现在车 206 道上。故选 C 项。 3.B 考查推理判断。根据第三段中的"The government has already permitted the company to use special materials to make it easier for the vehicle to fly"可推知,政府对该项目是支持的。故选 B 项。 4.D 考查主旨大意。根据第一段中的"Terrafugia Inc. said Monday that its new flying car has completed its first flight, bringing the company closer to its goal of selling the flying car within the next year"可知,Terrafugia 公司的飞行汽车成功进行了第一次飞行,从而使飞行汽车更加 接近现实。所以选 D 项。 【干扰项分析】 主旨大意题的解答要注意文章的首段和尾段,同时要结合文章中大部分内 容所讲述的观点。文章中只提及 Transition 会出现在 New York Auto Show,但这不是文章主 要内容,故排除 A 项;B 项只是飞行汽车的第一次飞行,过于片面;C 项中的"Pilots’ Dream" 未在文中提及。 【长难句解读】 〖HTF〗The Transition, which flew at 1,400 feet for eight minutes last month, can reach around 70 miles per hour on the road and 115 in the air.  分析:句中 which 引导非限制性定语从句,先行词为 The Transition。 译文:飞行汽车 Transition 上个月在 1,400 英尺的高度飞行了 8 分钟,它在公路上的行驶 速度能达到每小时 70 英里,在空中能达到每小时 115 英里。〖KH-*1D〗〖FK)〗 2012—2016 年 Passage1 本文的主人公——14 岁的女孩娜塔莉——主动参与重建遭受飓风重创的家乡。她建立 了一个网站,为外界的捐赠者和家乡需要帮助的人结对,同时帮助很多孩子实现了自己的梦 想。她应邀到白宫,并被授予荣誉称号。 1.B 细节理解题。由题干的"returned...after the hurricane"可知,答案应该在第三段。娜塔莉 一家在飓风过后回到家乡,发现街区已变为废墟(neighborhood in ruins),很多朋友失去了家 园,并且住到了很远的地方。人们都在遭受苦难,特别是老年人。从这些信息可知选 B。 2.A 细节理解题。由第四段第一句中的"the men and women helping Rockaway recover inspired Natalie"可知选 A。 3.D 细节理解题。由第七段可知,娜塔莉建立了一个网站,让捐助者与需要帮助的人通过 这个网站建立联系,以此来帮助他们,因此选 D。 4.A 主旨大意题。本题给出的四句谚语的含义依次为:"小人物也有大作为";"患难之友才 207 是真正的朋友";"金窝银窝不如自己的狗窝";"技术就是力量"。本文介绍的是 14 岁的小女 孩娜塔莉如何帮助家乡的重建工作,因此选 A。 【长难句解读】Natalie posted information about a boy named Patrick, who lost his baseball card collection when his house burned down.分析:本句中过去分词 named Patrick 作定语修饰名词 a boy,who 引导非限制性定语从句修饰 a boy。从句中的 when 引导时间状语从句,动词短语 burn down 表示"烧毁"。译文:娜塔莉在网站发布了一个叫帕特里克的男孩的信息。他家的 房子(在飓风中)被烧毁了,因此他失去了自己收集的棒球卡。 Passage 2 这是一篇社会生活类的新闻报道。越来越多的高中毕业生没有直接去上大学而是选择了 间隔年,这样他们可以打工攒钱上大学,以便将来能更好地适应大学生活。 1.C 推理判断题。根据第一段中的"More students than ever before are taking a gap year (间隔 年) before going to university."和第三段中的"That is a record 14.7% increase in the number of students taking a gap year."可推断,间隔年这种现象越来越普遍。 2.A 推理判断题。根据第三段中的"Students who take a well-planned year out are more likely to be satisfied with, and complete, their chosen course. Students who take a gap year are often more mature and responsible"可推知,选择间隔年的学生将来能够更好地适应大学生活。 3.B 推理判断题。根据第四段的首句"But not everyone is happy."和下文内容可推断,Owain James 认为间隔年现象从一个侧面反映了大学生上学压力大、费用高的现状,因此他感到担 忧。 4.C 细节理解题。根据第四段中的"It is not surprising that more and more students are taking a gap year to earn money to support their study for the degree."和最后一句可知,大多数学生会充 分利用间隔年打工挣钱攒学费。 Passage 3 数年后,不管在家里还是在工作场所,社会机器人将非常普遍。社会机器人将用更人性 化的方式将技术带到日常生活中去。 1.B 推理判断题。根据第二段第一句可知,现在家务机器人做正常的家务,而社会机器人 不仅仅是工具,更像是伙伴。由此可知社会机器人更像人类,故选 B。 2.D 细节理解题。第三段中提到:你可以问 Jibo 机器人问题,也能要求它做不同的任务。 它不仅仅给出一般的答案,它能根据对家庭成员的了解做出不同的回应。它能够提醒老人吃 药,也能拍摄家庭照片。由此可知 Jibo 机器人能够服从你的命令,也能提醒你吃药,故选 208 D。 3.C 细节理解题。根据倒数第二段可知,Oshbot 能够在商店中帮助顾客,故选 C 项。 4.B 细节理解题。最后一段提到社会机器人不是用来取代工人的,而是和其他员工一起工 作的,由此可知社会机器人将成为我们的同事,故 B 项正确。 5.D 主旨大意题。通读全文可知,本文介绍了社会机器人的一些情况,故选 D 项。 Passage 4 本文为新闻报道。文中报道了 9 岁女童勇敢救父的故事。Izzy 的父亲因过敏反应心脏停 止工作,Izzy 采取急救措施挽救了父亲的生命。 1.D 细节理解题。由第一段中的"A schoolgirl saved her father's life by kicking him in the chest" 以及第二段中的"Izzy, nine, restarted father Colm's heart by stamping(踩)on his chest"可知, Izzy 踩她父亲的胸的目的是让心脏复苏。 2.C 细节理解题。由上下文可知,Izzy 先前了解了一些 CPR(心肺复苏术),后来她父亲因 过敏反应心脏停止了工作,Izzy 的母亲 Debbie 拨了 999 急救电话,然后 Izzy 采取了急救措 施。 3.A 段落大意题。第八段主要讲 Colm 所患的疾病,即奇怪的过敏反应。所以 A 项正确。 4.C 写作目的题。文章为新闻报道,第一段中的"A schoolgirl saved her father's life by kicking him in the chest"为导语,所以作者的写作目的是报道 9 岁女童勇敢救父的故事。 Passage 5 一具 320 万年的猿人骨骼化石即将首次在国外巡回展览,相关事宜正在紧锣密鼓地策划 之中。 1.D 写作目的题。本文是一则新闻报道,第一段是导语,根据本段的内容可知,作者写本 文的主要目的是报道一件即将发生的事情。 2.C 词义猜测题。根据画线部分后面的"while the real remains"可推知,画线部分指的是骨 骼复制品。 3.B 细节理解题。结合第二段后半部分的内容以及第四段的内容可知,包括 Houston, Washington, New York, Denver 和 Chicago 五个城市。 4.B 细节理解题。根据最后一段前半部分的内容可知,这具骨骼化石是根据甲壳虫乐队的 一首歌曲命名的。 专题六 人物故事类答案 2018 年 209 Passage 1 文章讲述了没有运动天赋的作者不畏艰辛,挑战自我,最终成功完成了自己第一次马拉松比 赛的故事。 【长难句解读】When I started running in my 30s, I realized running was a battle against myself, not about competition or whether or not I was athletic. 分析:句中 running was a battle against myself, not about competition or whether or not I was athletic 为宾语从句,从句中 whether or not I was athletic 也为宾语从 句,作介词 about 的宾语。 译文:当我在三十多岁开始跑步时,我意识到跑步是一场与自己较量的战斗,而不是比赛, 也与我是否是运动型的人无关。 1.C 细节理解题。根据第一段中的"Yet, I was determined to go ahead" 可知在马拉松比 赛前一个月,虽然作者的脚踝受伤,但他依然下定决心参加比赛。故选 C。 【干扰项分析】根据该段中的"this meant not running for two weeks, leaving me only two weeks to train"可知,作者没有充足的时间参加训练,所以不能误选 A 项。 2.C 推理判断题。根据第二段中的"I didnt do either well. He later informed me that I was ‘not athletic’" 可知,作者谈及体育课的目的是向读者表明"他没有运动天赋"。 故选 C。 【干扰项分析】文章主要讲述作者虽然没有运动天赋,但通过坚持不懈的努力,最终成功完 成了马拉松比赛,所以第二段的目的不是分享珍贵的记忆,不能误选 D 项。 3.A 细节理解题。根据倒数第二段中的 "I was one of the final runners to finish. But I finished!" 可知,作者完成了自己的第一次马拉松比赛。make it 意为 "达到预定目标, 做到,获得成功"。故选 A。 【干扰项分析】根据第七段中的"Despite the pain, I stayed the course walking a bit and then running again"可知,作者并非是走到终点线的,所以不能误选 D 项。 4.B 主旨大意题。根据第一段中的"Yet, I was determined to go ahead"以及最后一段可知, 作者主要表达的内容是"成功者是一个有坚强意志的人"。故选 B。 【干扰项分析】文中虽然谈及了作者妻子的鼓励,但不是文章的中心,不能误选 A 项。 Passage 2 本文是一篇记叙文,讲述了英国伟大的作家狄更斯的创作历程和他伟大的原因。 210 1.C 细节理解题。根据第一段中的"Novels, for the most part, were looked upon as silly, immoral or just plain bad"可知,在 18 世纪的时候,小说被人们看作是愚蠢的,不道德的 和非常糟糕的,所以选 C 项。 2.D 推理判断题。根据第二段,尤其是 "Today Dickens greatness is unchallenged. Removing him from...the Louvre selling off the Mona Lisa"可推知,狄更斯的伟大是不容 置疑和无可辩驳的,如果说要把他从英国文学史上拿掉的话,就犹如卢浮宫把蒙娜丽莎卖掉 一样。由此我们推断出此处是在强调狄更斯在英国文学史上的重要性。 【干扰项分析】根据常识可知,狄更斯不是法国人,文中也未提到他在法国的名声,故排除 A 项;B 项在文中无细节信息支撑,故排除 B 项;根据第二段可知,狄更斯虽然同时也是个 出版商,但这并不是他的成就所在,故排除 C 项。 3.A 写作意图题。根据文章最后一段中的"No one will ever know…such a distinguished writer. But as the 200th anniversary of his birth approaches…lasting one"可推 知,作者写作本文主要是为了纪念狄更斯 200 周年诞辰。 2017 年 Passage 1 本文是一篇记叙文。作者讲述了与 Paul 的相识及与其建立友谊的原因,虽不常见面但是却 因共同的信念而保持着友谊。 【段意梳理】 第一段:讲述作者与 Paul 相识的背景; 第二段:作者与 Paul 发展友谊的原因; 第三段:作者与 Paul 是如何保持他们之间的友谊的; 第四段:讲述作者与 Paul 之间的友谊之深。 1.C 考查细节理解。根据第一段中的"When the studio didn’t want me for the film — it wanted somebody as well known as Paul — he stood up for me"可知,摄影棚起初不愿意给作者角色的 原因是想找一个与 Paul 一样著名的人物。故可知作者当时不够有名。该题选 C 项。 2.D 考查推理判断。根据文章第二段中的"We were respectful of craft(技艺) and focused on digging into the characters we were going to play. Both of us had the qualities and virtues that are typical of American actors"可知,作者与 Paul 之所以拥有持久的友谊是因为他们有相似的性 格特征。故 D 项符合题意。 211 【干扰项分析】 该题文中只是给出了一些琐碎的信息,需要考生根据这些信息总结出两人 拥有持久友谊的原因,这就要求考生具有一定的归纳推理能力。文中明确提到了作者与 Paul 并不同龄,所以排除 A 项;B 项文中并未提及;C 项不是两人拥有持久友谊的原因。 3.A 考查代词指代。根据第三段中的"We shared the belief that if you’re fortunate enough to have success, you should put something back"可知,下文的内容是对 the belief 的解释,that 引 导同位语从句,解释说明 the belief 的具体内容。下文提到了我们并不经常见面,但是分享 这个信念把我们带到了一起。故选 A 项。 【干扰项分析】 该题要求考生具有一定的句法分析能力,能从一个长难句中找出关键词, 并根据上下文信息判断出代词的指代内容。而 B、C 项只是 the belief 后的同位语从句中的 部分内容,故排除;根据画线词后一句可排除 D 项。 4.B 考查写作目的。根据全文内容,尤其是第一段中的"I first met Paul Newman in 1968"以及 最后一段中的"I last saw him a few months ago"可知,该篇文章的写作目的是回忆一位朋友, 所以选 B 项。 【干扰项分析】 写作目的题是考查作者写这篇文章是为了什么,达到什么目的。对于记叙 文而言,往往是开篇引出主题,并以时间顺序叙述全文,最后进行总结。主旨大意题是考查 文章讲什么,中心思想是什么。所以两者要有所区分。A、C 和 D 项都过于片面,故排除。 【长难句解读】The friendship that grew out of the experience of making that film and The Sting four years later had its root in the fact that although there was an age difference, we both came from a tradition of theater and live TV. 分析:该句中第一个 that 引导定语从句,修饰先行词 The friendship;第二个 that 为限定词, 修饰 film;第三个 that 引导同位语从句,说明 the fact 的内容,而该同位语从句中 although 引 导让步状语从句。 译文:从拍摄那部电影以及四年后的《骗中骗》的经历中成长的友谊源于尽管有年龄的差异, 但是我们都来自于戏剧和现场直播电视的传统这个事实。 Passage 2 这是一篇记叙文。文中讲述了垒球队中的女队员 Paris 在打球过程中突发心力衰竭,队友 Taylor 对她进行了紧急心肺复苏,Paris 最后恢复了心跳的故事。Taylor 勇敢、冷静,用所 学的急救知识救助他人。 【段意梳理】 第一至三段:Paris 突发心力衰竭。 212 第四段:对 CPR(心肺复苏法)的介绍。 第五段:Taylor 用心肺复苏法实施救助。 第六段:队友参与救助。 第七段:Paris 对队友的感谢。 第八至九段:心肺复苏法的重要性及其对 Taylor 的影响。 1.B 考查细节理解。根据第三段中的"It certainly was. Paris had suffered a sudden heart failure" 可知,Paris 突发心力衰竭。 【干扰项分析】A 项"无中生有",不能因为文中出现天气寒冷等方面的信息就说 Paris 患了 感冒;C 项"鱼目混珠",第二段中的"Suddenly, Paris fell to the ground"不能表明 Paris 倒地的原因是被球击中;选项 D"张冠李戴",根据文中的"Taylor Bisbee shivered(发抖) a little as she watched her teammate Paris White play"可知,Taylor Bisbee 在看她的队友 Paris White 打球时冷得发抖。 2.C 考查细节理解。根据第五段中的"But when no one else came forward, Taylor ran to Paris and began doing CPR"以及第六段在队友和学校护士的帮助下 Paris 的心跳恢复了(Paris’s heartbeat returned) 以及倒数第二段中的"For a sudden heart failure, the single best chance for survival is having someone nearby step in and do CPR quickly"可知,Paris 得到了及时的心肺复 苏(CPR)救助。 【干扰项分析】A 项"似是而非、答非所问",文章的话题不是交朋友;B 项"偷梁换柱", Paris 是突发心力衰竭,而绝非"shock(休克) ";文中最后一段提到了 Paris 回到球队里,但这不 是 Paris 说她很幸运的原因。 3.B 考查推理判断。根据第五段中的"But when no one else came forward, Taylor ran to Paris and began doing CPR. ‘It was scary. I knew it was the difference between life and death,’ says Taylor"和第六段中的" Taylor’s swift action helped her teammates calm down"可知, Taylor 是一位勇敢、冷静的女孩。面对紧急情况,她不慌不乱,用所学的急救知识迅速帮助 他人,并用自己的举动感染队友,让她们镇静下来参与救助。 【长难句解读】Two more ran to get the school nurse, who brought a defibrillator, an electronic device(器械) that can shock the heart back into work. 分析:这是一个主从复合句,句中 who 引导定语从句,an electronic device 为同位语,that can shock the heart back into work 为定语从句。 译文:还有两个队员跑去找学校护士,护士带来了除颤器——一种能震击心脏让其重新工作 213 的电子器械。 Passage 3 本文主要介绍了美国绘画之父本杰明·韦斯特小时候的艺术启蒙。 【段意梳理】 第一段:6 岁时本杰明就展现出艺术天赋,用猫毛制作画笔。 第二段:用猫毛制作画笔的事被父亲发现。 第三段:本杰明的天赋引起了堂兄 Pennington 的注意,并得到了帮助。 第四段:9 岁时本杰明被堂兄 Pennington 带到费城。 第五段:在费城,本杰明开始叩响艺术殿堂的大门,并立志成为艺术家。 【背景知识】 本杰明·韦斯特,美国画家,生于宾夕法尼亚州斯普林菲尔德,6 岁学画,被誉为神童,20 岁时在费城画肖像。1760 年被费城商人送往欧洲,到罗马时引起轰动,被贵族视为高贵的 未开化的人。1771 年《沃尔夫将军之死》震惊画坛,他得到了乔治三世的保护和友谊。1772 年他被委任为国王历史画家,为王家学院的创建者之一, 一生作有 400 多幅油画,主要以历史神话和宗教为题材,是第一位在欧洲获得承认的美国艺 术家,对美国 19 世纪初美术发展产生重大影响。 1.C 考查主旨大意。根据文章第一段首句"Benjamin West... when he was only six years of age" 以及第四段首句"In 1747, when Benjamin was [JP3]nine years old"和最后一段,并结合全文可 知,全文主要讲述了本杰明·韦斯特少年时代的艺术启蒙,故选 C 项。 2.D 考查句意理解。该句需要结合上下文来理解,第二段提到本杰明剪猫的毛来制作画笔, 导致猫惨遭"毁毛破相",而下文又提到本杰明的堂兄 Pennington 给他送来了颜料和画笔。由 此可以推知,画线句说明"猫的命运得到了改善",隐含着"本杰明将得到真正的画笔", 故选 D 项。 【干扰项分析】 A 项"猫会被密切关注",文章中并未提及;B 项"猫会得到一些药物治疗",虽然第二段中提 到本杰明父亲认为猫病了,但并没有说明它需要治疗;C 项"本杰明将很快离开家",根据下 文可知,本杰明只是 9 岁的时候离开过家,故与语境不符。 3.B 考查细节理解。根据文章第五段首句"In the city, Mr. Pennington gave Benjamin materials for creating oil paintings"可知,为了更好地发展本杰明的绘画天赋,堂兄 Pennington 给了他 很多帮助,提供了创作油画的材料,故选 B 项。 214 4.D 考 查 推 理 判 断 。 根 据 文 章 最 后 一 段 本 杰 明 说 的 话 以 及 "While it is likely that he understood ...The nine-year-old boy decided then that he would be an artist"可推知,Williams 给 本杰明的两本关于绘画的著作帮助本杰明打开了艺术之门,使他立志成为艺术家,故选 D 项。 【 长 难 句 解 读 】 While it is likely that he understood very little of the books, they were his introduction to classical paintings. 分析:这是一个复合句。主句是 they were his introduction to classical paintings,While it is likely that he understood very little of the books 是让步状语从句; that he understood very little of the books 是主语从句。 译文:虽然很有可能本杰明对这些书理解甚少,但它们却是他了解古典绘画的入门指导。 2012—2016 年 Passage 1 本文介绍了一个名叫 Peter Hodes 的人从事的有关干细胞传递的志愿者工作,并主要讲 述了他的一次难忘的经历。在传递干细胞的过程中,他深刻地意识到时间就是生命。 1.B 词义猜测题。根据第一段中的"Since March 2012, I've done 89 trips — of those, 51 have been abroad. I have 42 hours to carry stem cells(干细胞) in my little box"可知,作者是干细胞传 递员,故选 B。 2.D 细节理解题。根据第一段中的"I have 42 hours to carry stem cells(干细胞) in my little box because I've got two ice packs and that's how long they last"可知,冰维持的最长时间是 42 小时, 故选 D。 3.B 细节理解题。根据第二段的最后一句"She arranged for a flight on a small plane to be held for me, re-routed( 改 道 ) me through Newark and got me back to the UK even earlier than originally scheduled"可知,这位女士安排作者乘坐一架小型飞机改道经纽瓦克回到了英国, 故选 B。 【 长 难 句 解 读 】 For this courier job, you're consciously aware that in that box you've got something that is potentially going to save somebody's life.分析:For this courier job 是介词短语, 在句中作状语;that in that box you've got something that is potentially going to save somebody's life 是宾语从句,in that box 是宾语从句中的地点状语,you've got something 是宾语从句中的 主谓宾;that is potentially going to save somebody's life 是定语从句,修饰前面的 something。 译文:对于这份传递员的工作而言,你会自觉地意识到你手中的那个箱子装着可能挽救某人 215 生命的东西。 Passage 2 本文介绍了美国著名女作家 Eudora Welty 在纽约的一次经历,并以此作为背景说明她 的创作源于生活。 1.A 细节理解题。第三段中的 "the woman"指的是第二段中的那个陌生人,根据第三段中 的"When her dinner partner showed up, she also pulled up a chair"可知,陌生女子的晚餐伙伴也 加入了她们的行列中,故有两个陌生人加入到了 Welty 和朋友的晚餐中。 2.D 代词指代题。画线词所在句"These are the people that make me write them"的主语 These 指的就是像那两位陌生人那样的人,换言之,这些新朋友就成了 Welty 笔下的写作素材,这 些人就是让她写小说的人,故 them 指的是"小说"。 3.C 推理判断题。根据最后两段的内容可知,Welty 小说中的人物源于真实生活。 【长难句解读】1.Heading back downtown toward her hotel, her big-city friends were amazed at the turn of events that had changed their Big Apple dinner into a Mississippi state reunion(团聚). 分析:本句为复合句,逗号前为现在分词短语作时间状语,定语从句 "that had changed their Big Apple dinner into a Mississippi state reunion(团聚)"在句中修饰先行词 the turn of events.译 文:在返回市区宾馆的途中,韦尔蒂的大都市的朋友们惊讶于他们活动的转变,他们的聚会 早已由纽约式的宴会变成了密西西比人的地方性团聚。 2.Beauticians, bartenders, piano players and people with purple hats, Welty's people come from afternoons spent visiting with old friends, from walks through the streets of her native Jackson, Miss., from conversations overheard on a bus. 分析:句中的 "Beauticians, bartenders, piano players and people with purple hats"与"Welty's people"构成同位关系,三个 from 介词短语与 come 连用,表明小说中人物角色来源的多面性。译文:Welty 小说中的角色——美容师、 酒吧招待、钢琴演奏者及那些戴着紫色帽子的人——源自于她与老朋友共度的下午,源自于 她在土生土长的密西西比州杰克逊城的大街小巷散步的过程中,还源自于她在公交车上无意 间听到的闲谈。 Passage 3 这是一封书信,作者有注意力缺陷,因此没能实现大学梦,但是一家培训中心的网站给 了作者在线学习的机会,让她的生活有了目标,并且找到了一份好工作。 1.B 细节理解题。题干的"不去读大学"告诉我们,本题的答案应该定位在第四段。该段说, 作者的病情变得更糟了,因此不能去读大学,而这个疾病是第二段说到的 ADHD,具体表 216 现是注意力不能集中较长时间(unable to keep focused for more than an hour at a time),这与 B 项叙述吻合。 【特别提醒】高考阅读理解细节题的特点:题干通过同义转述引导我们去思考并定位答案所 在段落,而不是直接问"根据第几段我们知道……";正确项也不是原文复现,而是注重同义 转换,比如本题 B 项中的 had more difficulty keeping focused 与文章第二段的 unable to keep focused...和第四段的 got worse 就是同义转换。这对我们的理解能力提出了更高的要求。 2.A 细节理解题。根据题干的 working environment,我们找到第六段的 looking for a job, 该段说到作者希望的工作环境:I could work alone, but still have a team to talk to。A 项的 working by herself 与该句的 work alone 表达的意思是一致的,因此选 A。 3.C 写作目的题。作者在文章开头直接说明写信的目的"告诉你你的帮助对我的人生是多么 重要";然后描述自己的病情,未了的大学梦,以及收信人开办的网站给作者带来的影响, 因此本文的写作目的是 C:表明作者对于这个培训中心的感激。 【长难句解读】Growing up, I had people telling me I was too slow, though, with an IQ of 150+ at 17, I'm anything but stupid.分析:本句使用现在分词短语 Growing up 作时间状语,现在分 词短语 telling...作 people 的后置定语,以及 though 引导的让步状语从句;短语 anything but 表示"一点也不"。译文:在我成长的过程中,总有人告诉我说,我反应太迟钝,尽管我 17 岁时智商就超过了 150,一点也不笨。 Passage 4 本文介绍了牙买加短跑运动员 Shelly-Ann Fraser-Pryce 的情况。 1.B 细节理解题。根据第一段的"Her times were not exactly impressive, but even so, he sensed there was something trying to get out, something the other coaches had overlooked when they had assessed her and found her lacking"可知 Stephen 教练看到了她的潜能,因此本题答案为 B。 2.C 细节理解题。根据第二段中的"before concluding that she must be one of those one-hit wonders that spring up from time to time, only to disappear again without signs"可知短跑界认为 她的成就将是昙花一现,不会持续多久,因此 C 正确。 3.C 细 节 理 解 题 。 根 据 第 三 段 中 的 "Maxime's early entry into the adult world with its responsibilities gave her the determination to ensure that her kids would not end up in Waterhouse's roundabout of poverty..."可知她想让孩子们摆脱贫穷的愿望使她决定把女儿送 上跑道,因此 C 项正确。 4.B 推理判断题。本段中画线部分后面的内容是对这句话的诠释,因此这句话的含义是她 217 想为祖国做更多的事,因此 B 项正确。 5.D 推理判断题。根据 Shelly-Ann Fraser-Pryce 的成长经历和 Muhammad Ali 的话可知冠军 不是诞生在体育馆,而是由他们内在的东西铸就成的。因此答案为 D。 6.A 标题判断题。本文主要讲的是一位短跑名将是如何铸就的,因此 A 作为文章标题最为 合适。 Passage 5 小时候,作者的家教十分严格,母亲不轻易给子女买东西。但一件事情深深地震撼了作 者,也教给了她值得铭记一生的道理——用勤劳的双手去创造属于自己的财富。 1.C 推理判断题。从第二段中母亲对我们种种严格的要求可推知,作者的母亲与其他父母 养育小孩的方式不同,她对我们很严格,故选 C 项。 2.A 细节理解题。从文章第三段的"...the basket winked at me and I knew — I knew — I had to have it"以及下文作者特别期盼得到它的行为可以推断出,作者第一眼看见这个车篮就爱上 了它,故选 A 项。 3.B 推理判断题。从第三段至第五段的内容可知,作者在文中多次提到"please" 这个单词是 为了表达自己迫切想要得到这个车篮的心情,故选 B 项。 4.B 推理判断题。根据上文作者为了获得车篮而作出的种种努力可知,此处表示的是对作 者来说,没有车篮的车子看起来就像是"赤身裸体的",这从侧面反映了车篮对作者的重要性, 故 B 项正确。 5.D 细节理解题。从文章倒数第二段"Days later the unthinkable happened...This horrible turn of events"可知答案。 6.C 推理判断题。通读全文可知,作者通过自己小时候想要车篮的经历告诉我们,要用勤 劳的双手去创造属于自己的财富,C 项最能表达文章的主旨。 Passage 6 本文介绍了著名的西班牙艺术家萨尔瓦多·达利的艺术作品即将在法国巴黎 The Pompidou Centre 展出的情况。 1.B 推理判断题。根据第一段可知,艺术家萨尔瓦多·达利的 200 多幅油画、雕塑和绘画作 品等将在巴黎的 The Pompidou Centre 展出。由此可判断这个艺术家的创作颇丰。optimistic" 乐观的"; productive"多产的";generous"慷慨的";traditional"传统的"。故选 B。 2.A 细节理解题。根据第一段中的"Among the works and masterworks on exhibition the visitor will find the best pieces, most importantly The Persistence of Memory."可知,萨尔瓦多·达利的 218 作品 The Persistence of Memory 被认为是他的一幅杰作。 3.D 细节理解题。根据第二段中的"The exhibition follows a path of time and subject with the visitor exiting through the brain."可知,达利的世界是按照时间和主题组织作品展览的。 4.A 词义猜测题。根据最后一段可知,多家博物馆通力合作,共同精选展出作品。文中提 到,像西班牙马德里的博物馆和圣彼德斯堡的博物馆这样的机构也提供萨尔瓦多·达利的作 品参加展览。由此推测,contributions 指"作品",故选 A。 Passage 7 本文介绍了 Brooks 的一本新书。在 Brooks 的新书中,他从科学的角度,用故事的形式 讲述了人从出生到死亡的整个生命进程中所蕴含的科学理论。 1.D 推理判断题。文章第一段说科学有很多用途,因此人们总想把科学运用到不需要科学 的地方,然后介绍了 David Brooks 的新书。从第一段可判断,列举科学的用途是为了说明 很多作家都喜欢在自己的作品中运用科学的原因,因此选 D。 2.C 推理判断题。第三段讲述 David Brooks 的这本书的具体内容,然后在第四段说到"his writing is mostly clear",因此选 C。文章最后一段说这本书的角色"not strong or memorable"(D 错),第三段说作者在写作的时候"tries to make his points"(A、B 错)。 3.D 作者态度题。最后一段作者在评价这本书的时候说到"mostly clear; some chapters stand out above the rest; not strong or memorable characters; the more serious problems; attempt to translate his tale into science",从这些信息看,作者对于这本书的态度是批判性的(critical)。 4.A 推理判断题。文章最后一句说到这本书的一些问题,那么后文很可能会具体说明这本 书的问题所在。因此选 A。 Passage 8 文章介绍了 Bill Drayton 创办的名叫"阿育王"的扶贫济困机构。 1.A 标题判断题。本文主要介绍的是 Ashoka 这个扶贫济困机构,该机构旨在让每一位公民 都成为 changemaker。第一段两次出现 changemaker,第二段首句"Drayton believes that anyone can become an agent for change."为主题句,an agent for change 与 changemaker 同义,最后一 段回归到结果:bringing about social change by changing the way。显然贯穿文章始终的是 changemakers。故选 A。 2.D 代词指代题。第三段讲的是在 1995 年 Masqsood and Iftekhar 负责教城里的穷人们把垃 圾制成堆肥,因为他们认为这些化肥会有市场,但起初遭到拒绝,当他们说服这些芬人这样 做有钱可赚时,这一项目得以实施。故画线的 them 指代 the poor people in the city。故选 D。 219 3.C 推理判断题。根据第二段第三句"If you see a problem that you care about, you can help solve it." 以 及 第 五 句 "In fact, it is many young people's ambition to set up programmes or businesses that improves social conditions."综合可知选 C。 4.D 作者态度题。文章对 Ashoka's program 自始至终充满着赞赏之词,显然作者对此是持 积极态度的。故选 D。 Passage 9 本文讲述了作者大学毕业后,经过犹豫后,选择参加 Lighthouse Project,去 Nigeria 支教 的难忘经历。 1.A 细节理解题。根据第一段第二句中的"My degree, with honors, in English literature had not really prepared me for anything practical"可知,作者所接受的英国文学未能让他学到实用的知 识,他的大学更关注的是理论性知识。故选 A。 2.A 推理判断题。根据第二段尾句"Neither did my family"可推知作者与家人讨论过自己的 决定。 3.D 推理判断题。根据第三段第二句中的"countless interviews and presentations"可知,作者在 申请过程中经历了无数次的面试和陈述,最终才可以"stand out among the candidates and survive the test alone"。这足以说明竞争很激烈。故选 D。 4.C 细节理解题。 根据第四段第二句"Though the local villagers were poor, they offered their homes, hearts, and food as if I were their own family"可知,当地居民把作者视为家人一般热情 款待。 5.A 推理判断题。根据最后一段中的"I realized that all those things that had seemed so strange or unusual to me no longer did"可推知,他最初是很难适应当地文化和习俗的。 Passage 10 乔治·格什温是一位举世闻名的作曲家,他 18 岁开始作曲,在短暂的一生中为世人留下 了 500 多首曲子。 1.D 细节理解题。根据第二段的最后一句"Over the years they have been sung and played in every possible way — from jazz to country."可知,人们以各种形式演唱或演奏格什温创作的曲 子。故选 D。 2.B 细节理解题。根据第三段最后一句中的"showed that jazz music could be both serious and popular"可知,B 项正确。 3.A 细节理解题。根据第四段中的"While there, Gershwin wrote An American in Paris."和"It 220 still remains one of his most famous works."可知,A 项正确。 4.B 推理判断题。根据最后一段的第三句"Newspapers all over the world reported his death on their front pages."可知,世界各地的报纸都对他的去世进行了报道。故选 B。 5.A 推理判断题。通读全文,尤其是第一段的内容可知,格什温 18 岁开始作曲,因此他很 有"天赋";他一生创作了 500 多首曲子,因此他是位"多产的"作曲家。 Passage 11 一对母女合写了一本书,书中描述了这对母女生活的快乐和回忆。 1.D 细节理解题。根据第二段第一句可知,Perri 发现自己的生活折射了妈妈的生活:她们 都有自己的事业,都出版了书籍、文章和小说,都生了三个孩子,都喜欢看书和旅游。由此 可知选 D。 2.C 词义猜测题。根据第三段可知,Sheila 生长在 Brooklyn,她的父母都觉得让女孩儿上 学是 luxury;对 Sheila 来说,浪费时间或者金钱是犯罪,luxury 是难以想象的,而女儿 Perri 则可以偶尔享受一下小小的 luxury。从这些信息来看,luxury 是指"昂贵但不必要的东西"。 3.A 段落大意题。根据第四段可知,这对母女在书中描述了把她们的生活连在一起的那些 快乐与痛苦,爱与恨,讲到了母女两人交流抚养孩子的看法,一起享受珍贵的回忆等。这是 这本书的内容,而不是该书的目的、影响或写作风格。 4.D 推理判断题。根据最后一段可知,这本书是用两个人的声音在讲述,好比二重奏;根 据第四段可知,她们用自己的不会被混淆的声音在轮流讲述。从这些信息看,这本书的写作 风格是母女两人从不同的角度探索女性生活,而不是"用音乐形式(误解了二重奏的含义)、 通过野外研究(Perri 父亲的工作)或独特的写作技巧(本文没有谈到她们的写作技巧)"。 Passage 12 墨西哥女画家弗里达·卡罗,身残志不残,创作了无数优秀作品。虽然在有生之年其作 品未能受到应有的关注,但她最终于上世纪 70 年代赢得了国际声誉。 1.D 短语理解题。她去世后,能力(A 项)、天赋(B 项)、身体强壮程度(C 项)不会发 生变化,改变的只是人们对她的评价和了解而已,故选 D 项。第一段的第二句是答案提示。 2.C 细节理解题。根据第二段的第三句和第四句可知,卡罗背部的剧烈疼痛是由那次校车 事故造成的。而手术是为了治好背部的伤,而不是造成她背部疼痛的原因。 3.B 细节理解题。根据第三段第二句的后半句"but her later works from the 1940s, known today as her best works, show less influence from her husband"可知选 B。 4.B 观点态度题。通读全文,尤其是第一段中的 Sadly 和最后一段中的 Unfortunately 可以 221 看出,作者对卡罗的遭遇以及其作品只是在去世后才引起关注充满了同情,故选 B。 专题七 逸闻趣事类答案 2018 年 Passage  这是一篇记叙文。作者 17 岁的时候和好友去博物馆,结果误闯民宅,30 年后,这家的一位 成员认出了作者并感谢作者让她意识到自己家的美。 1.A 细节理解题。第一段说,McNay 是水彩画家(watercolorist),她要求社区在她死后把她 曾经居住的地方改成博物馆,由此判断她是个画家。 2.D 推理判断题。根据题干信息,我们把答案定位在第四段。作者和朋友来到这个博物馆, 然后有人询问是否需要帮助的时候,作者说导游让她紧张,从这些信息判断,作者误以为这 个人是导游,因此选 D。 【干扰项分析】文章提到 A 项,但这不是让作者拒绝帮助的原因;文章没有提到作者与陌生 人交流的时候觉得紧张,故 B 项不正确;C 项属主观臆断,文章没有提到该信息。 3.A 推理判断题。根据题干的"被大厅里的人盯着看"判断,本题答案在第四段。该段说,当 作者发现大厅里的人似乎都喜欢窥探,都好奇地盯着自己的时候,心想,"他们这是怎么了 "(What was their problem)。从这些信息推断选 A,作者对此十分困惑。 4.D 推理判断题。第六段说作者和朋友来到真正的 McNay 博物馆,但是她们一直觉得紧张, 害怕又出现先前那种不同寻常的事。由此推断,她们一直想着先前误闯他人的家的事,因此 选 D。 【干扰项分析】A 项不对,该段说这里有梵高、毕加索等名家的画作;B 项不对,该段没有 说作者在这里停留的时间很短,只说自己在那里的时候一直很紧张;C 项不对,文章没有说 这个地方让作者很失望。 5.C 推理判断题。最后一段这个女士告诉作者,30 年前自己还是一个十几岁的青少年,在作 者误闯她家之前,自己从来不知道自己的家有多美丽,由此推断选 C,"人们往往对自己身 边的美丽视而不见"。 【干扰项分析】A 项很有干扰性。文中的女士说当年的自己从来没有觉得自己家有多美,并 非对生活没有品味,因此 A 不对;B 项"人们应该多与家人在一起"也不是这个故事要传达的 信息;D 项无中生有。 2017 年 222 Passage  研究发现胎教并非人类独有,一些鸟儿在这方面做得甚至更出色。 1.B 考查词义猜测。根据句中的转折词 But 与下文提到的鸟类在胎教方面做得很出色可知 选择 B。 2.A 考查细节理解。根据第二、三、四段的内容,尤其是第二段中的"the baby birds made the similar chirp to their mothers",第三段的最后一句及第四段的第二句可知,Kleindorfer 的发现是 基于母鸟与幼鸟发出的叫声的相似性的,因此本题答案为 A。 3.C 考查细节理解。根据文章倒数第二段的最后一句和最后一段中的"An evolutionary inference can then be drawn"可知,与母鸟的叫声最相似的幼鸟可以得到最多的食物,这也能说 明它符合进化论的理论:适者生存,故本题答案为 C。 2012—2016 年 Passage 1 本文讲述了作者五年前在西雅图教学生们美术时在教学中所发生的事情,以及培养学生 们的想象力的方法。 1.A 细节理解题。根据文章第一段的"to find out something about my students"可知,作者使 用 Tinkertoys 是为了弄清楚有关学生们的一些事情。故选 A 项。 2.D 推理判断题。根据第三段中的"Here was an exceptionally creative mind at work"可知,这 个男孩儿非常具有创造性,故选 D 项。 3.B 词义猜测题。根据下文的"I ran the risk of losing those students who had a different style of thinking"可知,作者冒着失去那些有不同思维风格的学生的风险。故可知该词的意思为"不 足,缺点",所以选 B 项。 【技巧点拨】对于词义猜测题的考查要结合画线词前后文的内容来判断,该题就是根据画线 词后的具体内容推断出来的。 4.A 推理判断题。根据第四段中的"Without fail one would declare, ‘But I'm just not creative.’" 及下文内容可推知,作者问学生们他们是否做梦了是为了让他们看到自己的创造力。故选 A 项。 【长难句解读】His presence meant that I had an unexpected teaching assistant in class whose creativity would infect(感染) other students. 分析:句中 that 引导宾语从句,且 whose 引导定语从句,修饰 an unexpected teaching assistant。 223 译文:他的存在意味着我在班级有一位意想不到的教学助手,并且他的创造力会感染其他的 学生。 Passage 2 一本新的影集把一次不成功的南极洲的旅行带回到现实生活中,由此引出了历史上去南 极洲探险的事迹。 1.D 细节理解题。根据第二段中的"In fact, they were shot from 1914 through 1916, most of them after a disastrous shipwreck(海难)"可知,这些照片记录了一次灾难性的冒险,故选 D 项。 2.C 细节理解题。根据第三段中的"From that point Shackleton wanted to force a passage by dog sled( 雪橇) across the continent. The journey was intended to achieve more than what Captain Robert Falcon Scott had done...in 1912"可知 Captain Robert Falcon Scott 是第一个到达南极的 人。故选 C 项。 3.C 细节理解题。根据最后一段中的"adventuring was even then a thoroughly commercial effort" 和 "started a business before his 1914 voyage to make money from movie and still photography"可知,Alexander 认为 1914 年旅行的目的是赚钱。故选 C 项。 Passage 3 天生我材必有用。就读于技校的儿子虽然没能实现父亲的愿望,但凭借娴熟的技艺让身 为报社记者的父亲刮目相看,从而改变了作者的观点并登上了作者心中的光荣榜。 1.D 细节理解题。根据第一段第二、三句话"As he grows you also age...could probably accomplish what you hoped for"可知,作者希望自己的儿子实现自己没有实现的目标。 2.A 细节理解题。根据第三段中的"My daughter is a university graduate...he became a ‘vo-tech’ student(技校学生)"可知,作者的女儿在学校里的表现要比儿子好。 3.C 细节理解题。根据第六段的最后一句可知,作者同意儿子修车是因为他认为这样做不 会造成更大损失。 4.B 细节理解题。根据倒数第三段中的"These kids are happiest when doing repairs" "And their minds are bright"可知,在作者眼里这些修理工既快乐又聪明。 5.A 推理判断题。根据倒数第二段中的"Most important, I have learned that fathers don't need clones in footsteps or anywhere else"可推知,作者最终意识到希望孩子追随自己的脚步并实现 自己实现不了的目标这种想法是不明智的。 【长难句解读】We who labor in clean shirts in offices don't have the abilities that motorheads have.分析:这是一个含有两个定语从句的复合句。主句为 We don't have the abilities。第一个 224 定语从句为 who labor in clean shirts in offices ,先行词为 We ;第二个定语从句为 that motorheads have,其先行词为 the abilities。译文:我们这些穿着干净的衬衫在办公室工作的 人并没有修理工们所具备的能力。 Passage 4 作者克服了注意力障碍,从而发现了一个新的自己。 1.C 细节理解题。第一段作者就谈到自己有注意力障碍,第二段的尾句说作者认为自己的 身体状况使他不能完成老师布置的任务。 2.B 细节理解题。依据第五段的"he invented a reading system of raised dots(点)"可知,Louis Braille 为盲人发明了一种非常有用的阅读体系。 3.D 推理判断题。文章尾段作者的老师 Mrs. Smith 写道"See what you can do when you keep trying?"说明了这位老师对作者的肯定,她鼓励作者继续努力。 4.C 主旨大意题。作者在文中讲述了自己经过努力,最终完成任务的经历。由这件事可以 看出,只要努力克服困难就能够成功。 Passage 5 文章主要讲述了女主人 Helene An 在五个女儿幼时,对她们进行家庭团结观的培养,为 她们日后的成功打下了坚实的基础。本文揭示了团结就是力量的道理。 1.A 细节理解题。由文章第一段中的"she tied several chopsticks together, representing a family. She showed the girls it was hard to break the tied chopsticks"可知,绑在一起的筷子显示了家庭 团结的力量。 2.B 细节理解题。根据文章第二段中的"when Helene and her husband Danny left their home in Vietnam in 1975, they didn't have much money"可知,答案是 B 项。 3.C 推理判断题。A 项与文中事实"the girls all graduated from college"不符;B 项"孩子们不 能忍受在家族企业中工作"与她们都在为家族企业工作的事实不符。D 项"她们为家庭成员间 的分歧所困扰",而文中事实是"Even though family members sometimes disagreed with each other, they worked together to make the business successful"。C 项由文中的"Our mother taught us that to succeed we must have unity"推理可知。 4.D 标题判断题。由第一段中绑在一起的筷子预示着家庭团结的力量开始一直到最后一段, 文章都在讲述一个主题——家庭团结造就了成功。 Passage 6 作者用诙谐、幽默的语气讲述了自己高价购买的一台电视机给自己带来的诸多烦恼。 225 1.C 细节理解题。依据第一段中的"I let a salesclerk fool me into buying a discontinued model. I realized this a day later, when I saw newspaper advertisements for the set at seventy-five dollars less than I had paid."可知答案。 2.A 词义猜测题。结合画线部分所在的句子"The set worked so beautifully when I first got it home that I would keep it on until stations signed off for the night."可知,此处指作者一开始把它 带回家时,一切如此顺利以至于作者一直看到电视台"停止播放节目"。故选 A。 3.D 细节理解题。依据最后一段的"My trip to the repair shop cost me $62, and the set is working well now"可知,最终作者到维修店修理才使电视机恢复正常。 4.D 推理判断题。由文章第一段的"My color television has given me nothing but a headache.", 第二段中的"I actually began to build up my arm muscles (肌肉) shaking my set.",以及最后一段 中的"but I keep expecting more trouble"可知,作者用幽默的语气描述了这台电视机给自己带 来的烦恼。 【长难句解读】I was able to buy it a little over a year ago because I had my relatives give me money for my birthday instead of a lot of clothes that wouldn't fit. 分析:这是一个主从复合 句。连词 because 引导原因状语从句,同时该从句又包含了一个由 that 引导的定语从句,修 饰先行词 clothes。 译文:我能够在一年多前买下它(电视机)是因为我让我的亲戚们在 我生日时给我钱而不是买大量不合身的衣服。 Passage 7 本文主要讲述了"我"从父亲那里学到的钓鱼智慧。 1.A 细节理解题。通过第一段中的 "But the only thing that wasn't very fun about it was that...I couldn't catch anything"可知答案。 2.C 推理判断题。通过第二段的内容可推知,父亲说那句话的真正含义是让作者了解鱼的 习性,故答案为 C。 3.D 细节理解题。通过第二段中的"That is why fish prefer shallow water to deep water"与"water is usually warmer in direct sunlight...the sun hurts their eyes"可知答案。 4.B 推理判断题。通过最后一段描述的作者回忆起父亲教导他的话可推知,进入商界后, 作者发现父亲的钓鱼建议很有启发。 5.B 文章出处题。根据最后一段的内容尤其是最后一句"I will show you how in the following chapters"可知,本文节选自一本关于销售的书。 226 Passage 8 一个人干净利落,做事很有条理,而另一个人却正好相反,这样的两个人同住一室不 打闹才怪呢。是什么使两位室友吵闹后又和好如初呢? 1.D 细节理解题。根据第二段的"Why under my bed"可知,凯特进屋之后大喊是因为作者把 自己的鞋子放在凯特的床下面了。 2.C 细节理解题。作者打扫房间是在得知凯特的奶奶病重的消息之后,再根据第三段最后 一句的提示可知,作者之所以这样做主要还是出于同情和关怀。 3.B 篇章结构题。第一段用连词 while,but,and 巧妙地将作者和凯特的不同进行了对比, 由此可知应该选择 B 项。 4.D 主旨大意题。全文通过对两个生活习惯截然不同的室友由闹矛盾到和好如初的叙述生 动地展示了"学做室友"这一主题。 Passage 9 一对陌生的老年夫妇,一束美丽的鲜花,一颗纯真的童心,一幅温馨的画面。 1.D 主旨大意题。举手之劳的一枝花竟然让对方无比感激,作者显然意在告诉我们小小的 善意也能带来巨大的快乐。前三项分别表示"人越小心越好""事实胜于雄辩""爱始于微笑"。 2.B 细节理解题。根据第三段中的"I was amazed that he'd come up with this sweet idea"可知, 他提出的买花的主意赢得了作者的支持。 3.C 词义猜测题。根据本句后半部分中的内容可推测作者的儿子看到血和医疗设备后非常" 害怕"。另外,下一句中的内容也是他害怕的表现。 4.A 标题判断题。文章最后凸显的是花的力量,所以此项最能概括文章大意。其余选项过 于笼统,不具有针对性。 Passage 10 文中讲述了作者的童年趣事,作者回顾了小时候随父母从 Coeur d'Alene 搬迁到 Fairbanks 后的一些冒险经历。 1.C 细节理解题。根据第二段中的"Working in the garden at midnight tended to throw her timing off, so she didn't care much about my bedtime" 以及第三段中的"Dad was a Railway Express agent and Mom was his clerk.That left me in a mess"可推知,作者的父母很忙。 2.D 细节理解题。根据第三段中的"Once I had a little fire going in the dirt basement of a hotel. I had tried to light a barrel (桶) of paint but couldn't really get a good fire going"以及第四段中的"I hadn't turned 5 yet"可知,作者四岁时差点儿引发火灾。 227 3.A 细节理解题。根据倒数第二段中的"As for Leonhard Seppala, famous as a dog sledder(驾 雪橇者 )"可知,Leonhard 擅长驾驶雪橇。 4.A 作者意图题。文章回顾了作者小时候随父母从 Coeur d'Alene 搬迁到 Fairbanks 后的一 些冒险经历。 Passage 11 文章叙述了作者 13 岁时妈妈送给她的生日礼物——一个图章戒指失而复得的故事。 1.A 词义猜测题。结合第一段第二句可知,好像除了作者以外每个女孩都有图章戒指,由 此可知在当时戴图章戒指很"流行"。fashionable"流行的,时兴的";available"可得到的"; practical"适用的";renewable"可更新的"。故选 A 项。 2.D 数字计算题。第一段提到作者在 13 岁时得到图章戒指;第三段提到图章戒指丢失两年 后房屋被卖掉;第六段提到新住户在买完房子 13 年后装修房屋时意外发现图章戒指。综合 上述内容可知,此时作者大概是 28 岁(13+2+13=28)。 3.A 细节理解题。根据第三段可知,作者在 15 岁时搬了一次家,又依据第四段首句可知数 年后,作者又搬了两次家,故 A 项正确;B 项与第三段倒数第二句不符;依据倒数第二段 最后一句可知,清洗戒指的是 a jeweler 而不是房屋买家,故 C 项错误;D 项与第三段第二 句不符。 4.B 标题判断题。通读全文可知,本文主要讲述图章戒指"失而复得"的故事,因此"Lost and Found"作为标题最适合。 专题八 史地人文类答案 2018 年 Passage 1 文章主要介绍美国纽约及加拿大道森市的历史发展与现状,倡导和谐自然,关注人类命运共 同体。 【段意梳理】第一段:决定城市选址的因素。 第二段:人们蜂拥至加拿大道森市的原因。 第三段:道森的衰落和阿拉斯加的兴起。 【背景知识】发展理念决定了区域协调发展的必要性,不同国家、不同地区的崛起离不开建 立有效的区域协调发展机制。以城市群为主体构建大中小城市发展的格局可加快农业转移人 口市民化。但在发展过程中应坚持"高起点规划、高标准建设"的原则进行顶层设计,从而保 障城市化进程的有序推进和长远发展,避免盲目建设方可彰显城市发展的魅力。美国纽约、 228 阿拉斯加,英国伦敦,法国巴黎,加拿大道森等城市的在交流、贸易及工业方面的发展就是 最好例证。 1.C 推理判断题。根据第一段中的"New York City, for example, is near a large harbour at the mouth of the Hudson River. Over 300 years its population grew gradually from 800 people to 8 million"可推知,由于纽约市地处哈德逊河入海口,其优越的地理位置吸引了早期的定居者。C 选项"Its geographical position"正确。A 选项"它的贸易文化",B 选项"它的少量人口",D 选 项"它的有利气候"在文中均无信息支撑。 2.B 细节理解题。根据第二段中的"Of the first 20,000 people who dug for gold, 4,000 got rich"可 知,在最先的两万淘金者中,有 4 000 人变得富有了,换言之,即有五分之一的人(One out of five people)变得富有了,故答案为 B。A 项属无中生有,文中并未提及最先到达道森的 20, 000 人中有三分之二的人留在了那里;C 项说几乎大家都放弃了,这与下文描述的五分之一 的人变得富有矛盾;文中第二段提到在前往道森的途中,雪崩造成了 63 人死亡,并非是 10,000 人死亡,故 D 项错误。 3.B 细 节 理 解 题 。 根 据 第 三 段 中 的 "But soon...and when they heard there were new gold discoveries in Alaska, they left Dawson City as quickly as they had come"可知,当道森的黄金都 被发掘后,人们听说在阿拉斯加发现了新的金矿,他们便很快离开道森了。由此可推知,他 们是想去阿拉斯加碰碰运气。A 项属偷换概念,文中说的是 "The city was crowded with disappointed people with no interest in settling down." (这个城市到处都是感到失望且无心定 居下来的人们)。C 项属无中生有,文中并未说他们不能忍受这里的冬季。D 项属曲解文意, 文章提到 "Necessities like food and wood were very expensive" (诸如食品和木材这样的生活 必需品非常昂贵),并非是说他们缺少食品。 4.A 主旨大意题。本文主要以道森为例,讲述了城市的兴衰,故 A 项正确。B 项"加拿大淘 金热",以偏概全;C 项"荒野之旅",曲解文意;D 项"道森的旅游业",不符合文意。 Passage 2 本文是一篇说明文,讲述了美国的汽车经济带动了其他经济的发展。但与此同时,也带来了 严重的环境问题。 1.B 推理判断题。根据第二段中的"The car shaped some of the most lasting aspects of American culture ...even the hamburger"可知,汽车业的快速发展带动了大量体现美国文化的其他行业 的发展。由此可知,本段提到汉堡的主要目的在于说明汽车业的发展对美国文化的影响。 229 2.B 细节理解题。根据第四段中的"The cars that drove the American Dream have helped to create a global ecological disaster" 和第五段中的"The problems of excessive( 过度的) energy consumption"可知,汽车业的发展带来的大问题是环境问题,选 B 项。 3.C 观点态度题。根据第五段中的"but hopes for the best"和全文最后一段可知,Friedman 对 美国的前景还是很看好的,认为在绿色经济的带动下,美国的未来会变得更好,选 C 项。 【干扰项分析】A 项表示"模棱两可的",B 项表示"感到怀疑的",D 项表示"容忍的,忍受的", 都不符合文意。 2017 年 Passage 1 本文介绍了举办第一届国际爵士乐日的宗旨、当前爵士乐面临的挑战以及如何让爵士乐重新 流行的设想。 【段意梳理】 第一段:联合国教科文组织设定 4 月 30 日是国际爵士乐日以及设立这个节日的宗旨。 第二段:在美国,爵士乐不像以前那样受欢迎。 第三段:肯尼迪中心的爵士乐顾问杰森·莫兰希望爵士乐能受到更多听众的喜爱,并 保护它的历史和文化。 第四段:杰森·莫兰想做的事是让他们那一代人以及年轻一代能重新认识爵士乐的价值。 第五段:杰森·莫兰认为如今爵士乐的娱乐功能已经丧失,而时代在改变,因此爵士乐也 应与时俱进。 第六段:杰森·莫兰希望人们借助音乐可以表达思想和感情,并希望通过他自己的努力, 让爵士乐长盛不衰。 1.D 考查细节理解。根据第一段的第二句"UNESCO(United Nations Educational, Scientific and Cultural Organization) recently set April 30 as a day to raise awareness of jazz music, its significance, and its potential as a unifying(联合) voice across cultures"可知,联合国教科文组织 近来把 4 月 30 日设立为国际爵士乐日,旨在增强人们对爵士乐价值的认识,故答案为 D。 2.C 考查代词指代。根据语境可知,画线词 that 指代上 段"Despite the celebrations, though, in the U.S. the jazz audience continues to shrink and grow older, and the music has failed to connect with younger generations"的内容,即爵士乐听众的数 量减少并且趋于老龄化,不受年轻人喜欢的现状。 230 【干扰项分析】 A 项是杰森·莫兰希望做到的事情,可排除;原文没有信息支撑 B 项,可 排除;D 项与原文的意思相反,可排除。 3.C 考查推理判断。根据第五段的内容尤其是"It has to continue to move, because the way the world works is not the same" 可知,由于时代在改变,人们的娱乐方式已经发生改变,所以 爵士乐应该随着时代改变,才能吸引更多的听众。 【干扰项分析】 根据文章内容可知,A 项和 B 项错误,应排除;根据第五段中的"The music can’t be presented today the way it was in 1908 or 1958"可知,当前的爵士乐不能按照 1908 年 或 1958 年的方式向观众呈现,因此排除 D 项。 4.A 考查主旨要义。本文介绍了为保护爵士乐,联合国教科文组织设立国际爵士乐日。纵 观全文内容尤其是第三至第六段中提到的杰森·莫兰对爵士乐的看法及希望可知,A 项"探 索爵士乐的未来"作本文标题最佳。 【干扰项分析】 虽然文中提到了爵士乐的衰落,但 B 项不能概括全文的内容,应排除;本文并不是讲述爵 士乐音乐家的故事,可排除 C;本文并没有提到在国际爵士乐日当天有哪些庆祝活动,可排 除 D。 【长难句解读】Despite the celebrations, though, in the U.S. the jazz audience continues to shrink and grow older, and the music has failed to connect with younger generations. 分析:本句是一个由 and 连接的并列句。前句中的 Despite"尽管"后加名词,相当于一个让 步状语从句。though 表示句意的转折。  译文:尽管有各种庆祝活动,美国爵士乐听众的数量在继续缩减并且趋于老龄化,并且音乐 不能引起年轻一代的共鸣。 Passage 2 一种新的商品引发了高利润的、快速增长的产业,迫使反垄断管理者介入控制那些处于主导 地位的公司。 1.A 考查细节理解。根据第二段的首句"Such situations have led to calls for the tech giants to be broken up"可知,这种形势要求科技巨头解体。这种形势是指上文提及的那几家大公司垄断 了数据市场,故本题答案为 A。 2.C 考查细节理解。从第三段可知,以谷歌公司为例,这些科技巨头发现了数据可以转变 为新的服务,例如翻译和视觉识别,这些都可以卖给其他公司。因特网公司对于数据的控制 231 赋予了他们巨大的权力,从而可以增强他们的控制地位。 3.B 考查推理判断。根据第五段可知,反垄断机构在考虑兼并时,通常根据规模来决定何 时进行干预,现在当评估这些交易的影响时,他们需要考虑到这些公司的数据资产的范围, 再根据"The purchase price could...a new-born threat"可知,通过重视公司的数据资产,反垄断管 理者可以避免规模陷阱。 4.D 考查细节理解。根据倒数第二段可知,放松网络服务商对于数据的控制,把更多的份 额给数据提供者,从而让小公司也有机会发展。 【长难句解读】A new commodity brings about a highly profitable, fast-growing industry, urging antitrust (反垄断)regulators to step in to check those who control its flow. 分析:urging...是非谓语动词短语,在句中充当结果状语;who 引导的从句是定语从句,修饰 句中的 those。 译文:一种新的商品引发了高利润的、快速增长的产业,迫使反垄断管理者介入控制那些处 于主导地位的公司。 Passage 3 本文是一篇议论文,主要探讨了意大利刚出台的对日益增长的移民所采取的语言测试项目及 人们对该项目的不同看法。 【段意梳理】 第一段:Cojochru 来意大利已经七年了,但为了继续待下去,她需要参加语言测试。 第二段:意大利政府对移民的新政。 第三段:不同的人对语言测试的看法。 第四段:其他欧洲国家的类似要求。 第五段:意大利的移民现状——移民数量大量增加。 第六段:Cojochru 的愿望和对新政的态度。 第七段:Cojochru 眼里的"歧视"。 1.A 考查细节理解。根据文章第一段的"but in order to stay she’s had to prove her language skills"可知,Cojochru 为了能够继续在意大利待下去,将不得不参加语言测试,所以选 A 项。 2.B 考查推理判断。根据文章第三段的"Some immigrant advocates worry that as hard financial times make ...for intolerance than integration(融合)[HT]"可知,许多移民支持者认为这种语言 要求将会带来更多的不宽容,而不是更多的融合,从而也可能导致更多的冲突,所以选 B 232 项。 3.C 考查推理判断。根据文章最后一段中的"she’s stayed in the country for years and can speak the local language fluently, she said"可知,Cojochru 已经在意大利长居多年,能流利地讲当地 的语言,这足以使她应对她在当地的工作,故选 C 项。 【干扰项分析】A 项"她与姐姐现在生活在意大利",这与倒数第二段提到的"她的两个孩子 与她姐姐在摩尔多瓦"不符,故排除;B 项"她喜欢学习意大利语",与文章开头提到的 "feels hurt"和全文提及的她的焦虑不一致,故排除;D 项"她希望能够回国"与倒数第二段提 到的"她希望获得意大利的永久居住权"不一致,故排除。 【长难句解读】The Moldovan has lived here seven years as a caregiver to Italian kids and the elderly, but in order to stay she’s had to prove her language skills by taking a test which requires her to write a postcard to an imaginary friend and answer a fictional job ad. 分析:本句是并列句。根据 but 可知,本句是表示转折关系的并列句,但 but 前后又都包 含较复杂的成分。前一部分的主要成分是 The Moldovan has lived here seven years;后一部分 主要成分是 she’s had to prove her language skills。同时第二部分中又包含定语从句 which requires her to write a postcard to an imaginary friend and answer a fictional job ad,修饰先行词 a test。 译文:这个摩尔多瓦妇女已经在意大利 7 年了,她的工作是照顾意大利的小孩和老人,但 为了能够继续留在意大利,她将不得不参加一个语言测试,以此来证明自己的语言能力,这 个测试要求她给一个虚构的朋友写一张明信片,同时回复一则虚构的招聘广告。 2012—2016 年 Passage 1 我们每个人都有一种"超能力"。我们应该尽量超越自己,利用这种"超能力"去做一些非 凡的事情,帮助他人,并改善世界。 1.B 词义猜测题。根据第三、四、五段的内容,尤其是"to improve the world" "to make a difference in the lives of others"及"devote some of your time...involved"可知,画线部分指的应 是帮助他人、改善世界的人生价值。故 B 项正确。 2.C 细节理解题。根据第三段中的"we are inspired...doing extraordinary things to improve the world"和第四段中的"They have a different kind of superpower that all of us possess: the power to make a difference in the lives of others"可知,作者讲他们每天都受鼓励的原因是他们周围的 一些人在做非凡的事情以让世界变得更美好。故 C 项正确。 233 3.B 推理判断题。根据第五段中的"But we do think that you can live a more powerful life when you devote some of your time and energy to something much larger than yourself... Volunteer or, if you can, contribute a little money to a cause..."可知,作者强调的内容是"我们要超越自我, 采取行动,帮助他人"。所以 B 项正确。 4.A 推理判断题。根据最后一段中的"...the most rewarding parts of our lives. It has changed who we are and continues to fuel our optimism about how much the lives of the poorest people will improve in the years ahead"可知,作者认为最穷困的人的生活会变得更加美好。所以 A 项正确。 【长难句解读】It has changed who we are and continues to fuel our optimism about how much the lives of the poorest people will improve in the years ahead. 分析:这是一个主从复合句。句中 who 和 how 均引导宾语从句,分别作 changed 和 about 的 宾语。 译文:它改变了我们,继续激发我们对未来几年里最贫困的人们的生活质量会有多大的提高 保持乐观。 Passage 2 文章简单介绍了沉默在不同文化里所代表的含义以及沉默的治疗价值等内容。 1.C 推理判断题。根据第一段第三句"A silence in a conversation may also show stubbornness, uneasiness, or worry"(某一谈话中的沉默可能也表示倔强、不安或者担忧)可知,silence in conversations 具有文化特异性。故选 C。 2.A 细节理解题。根据第二段中的"Many Native Americans value silence and feel it is a basic part of communicating among people, just as some traditional Chinese and Thai persons do. Therefore, when a person from one of these cultures is speaking and suddenly stops, what may be implied( 暗 示 ) is that the person wants the listener to consider what has been said before continuing"可知,中国人可能认为一个人在谈话时突然停顿是希望听话人去仔细思考。故选 A。 3.D 细节理解题。根据最后一段,尤其是最后一句"A nurse who understands the healing(治愈) value of silence can use this understanding to assist in the care of patients from their own and from other cultures"可知,护士应该充分利用沉默的治愈价值。故选 D。 4.B 主旨大意题。通读全文可知,文章介绍了沉默在不同文化里所代表的含义,故选 B。 文章未谈及 Sound,所以 A 项不对;文章虽谈到了 Silence to Native Americans,但这不是文 章的中心内容,故排除 C 项;文章未涉及 Speech Is Silver 的内容,所以 D 项也不对。 234 【长难句解读】Therefore, when a person from one of these cultures is speaking and suddenly stops, what may be implied( 暗示) is that the person wants the listener to consider what has been said before continuing.分析:when a person from one of these cultures is speaking and suddenly stops 是时间状语从句,what may be implied 是主语从句,what 在主语从句中作主语,that the person wants the listener to consider what has been said before continuing 是表语从句,表语从句 中含有宾语从句 what has been said before continuing,what 在宾语从句中作主语。译文:所 以当一个来自这些文化之一的人在讲话时突然停住了,可能那个人是在继续(讲话)之前暗 示听众去思考他之前说了什么。 Passage 3 这是一篇社会类文章。本文介绍了有关法属圭亚那土著人的生活方面的内容。 1.A 细节理解题。根据第一段中的"Surprisingly, these people live largely by their own laws and their own social customs" "In theory, they should live by the French law"可知,作者感到惊讶的 是法属圭亚那的土著人有自己的法律和社会风俗,他们很少遵守法国的法律。所以 A 项正 确。 2.D 细节理解题。根据倒数第二段中的"I wanted to show the audience a photographic record touching upon the uncivilized life"及最后一段中的"His black-and-white pictures present a world almost lost in time. These pictures show..."可知,Gin 是以一名摄影师的身份介绍圭亚那的。所 以 D 项正确。 3.D 观点态度题。由第三段中的"I have a special love for the French Guianese people" "I don't see it as a lawless land. But rather I see it as an area of freedom"可知,Gin 很欣赏圭亚那土著人 的生活方式。所以 D 项正确。 4.A 代词指代题。根据最后一段中的"These local citizens now have to balance their traditional self-supporting hunting lifestyle with the lifestyle offered by the modern French Republic, which brings with it not only necessary state welfare, but also alcoholism, betrayal and even suicide"可 知,it 指代的是上文的"the lifestyle offered by the modern French Republic"。所以 A 项正确。 【长难句解读】However, their remote locations mean that the French law is often ignored or unknown, thus making them into an interesting area of "lawlessness" in the world. 分析:这是一个主从复合句。句中 that 引导宾语从句,从句中的 making them into an interesting area of "lawlessness" in the world 为现在分词短语作结果状语。 译文:然而,他们偏远的位置意味着法国的法律经常被忽略或是不为人知的,因此这使它们 235 变成一个世界上有趣的不受法律约束的地区。 Passage 4 本文描述了作者在严冬季节去佛罗里达州的萨拉索塔市(Sarasota)过周末的感受。在温 暖如春的萨拉索塔市,各种蔬菜水果生机勃勃,而在寒冷的美国东北部,表面诱人的西红柿 却食之无味。 1.B 推理判断题。根据文章第一句中的"The freezing Northeast hasn't been a terribly fun place to spend time this winter"可判断,作者认为纽约的冬天很无趣。 2.D 细节理解题。根据第一段最后一句中的"the best part...was a 7 a.m. adventure to the Sarasota farmers' market that proved to be more than worth the early wake-up call"及第二段内容 可知,作者早上七点起床去集贸市场,看到各种新鲜的蔬菜水果,所以她认为早起很值得。 3.B 推理判断题。根据第三段可知,多年来,作者在冬季都不买西红柿。由第二句"No matter how attractive they look in the store, once I get them home they're unfailingly dry, hard, and tasteless"可判断,在纽约市的冬季,西红柿在商店看起来挺好的,但是拿到家里吃起来却没 有味道。因此答案为 B。 4.C 细节理解题。根据文章最后一段中的"I was planning to have dinner that very night"可知 答案为 C。 【长难句解读】These were the real deal — and at that moment, I realized that the best part of Sarasota in winter was going to be eating things that back home in New York I wouldn't be experiencing again for months. 分析:破折号后面的句子对前句进行解释。破折号后为主从 复合句。主句 I realized 后跟 that 引导的宾语从句,在宾语从句中含有 that 引导的定语从句, 该从句修饰先行词 things。 译文:这些都是货真价实的瓜果蔬菜——就在那一刻,我意识 到,萨拉索塔冬天的最美妙之处在于可以吃到新鲜的瓜果蔬菜,而我一旦回到纽约的家中, 连续几个月都无法品尝到这些瓜果蔬菜。 Passage 5 芝加哥早期城市规划不合理,一遇洪灾路面就会十分泥泞,后来在工程师 Ellis Chesbrough 和 George Pullman 的努力下,成功将城市路面以及建筑物提高,避免路面受洪灾影响,但这 又导致了另外一个问题的出现。 1.B 细节理解题。从文章第一段的内容可知,作者讲这个笑话是为了证明芝加哥的路面十 分泥泞。 2.D 细节理解题。从文章第三段第一句"An engineer named Ellis Chesbrough convinced the city 236 that it had no choice but to build the pipes above ground and then cover them with dirt"可得出答 案。 3.B 词义猜测题。根据文章第四段画线词后面的"Small wood-frame buildings could be lifted fairly easily"可知,hoist 与 lift 同义。 4.A 推理判断题。根据第五段最后一句"Astonishingly, the Tremont Hotel stayed open during the entire operation, and many of its guests didn't even notice anything was happening"可推断出 Tremont Hotel 的移动工程进行得很顺利。 5.C 主旨大意题。根据整篇文章可知,本文主要讲的是芝加哥早期(关于城市规划)的工 程问题以及解决方案。 Passage 6 文章主要说明了在阳光充足的环境中生长的咖啡豆对周围环境的影响。 1.B 细节理解题。根据第四段中的"...leads to air pollution and global warming"可知,在阳光充 足的环境中生长的咖啡豆会导致空气污染和全球变暖。 2.D 推理判断题。根据文章最后一段中的"But consumers do have a choice. They can purchase shade-grown coffee whenever possible... The future health of the planet and mankind is surely worth more than an inexpensive cup of coffee"可知,消费者是有选择的,要尽可能购买树荫下生 长出来的咖啡豆,因为这对于地球的未来来说是很值得的。由此可以推断出,作者写作本文的 目的是"劝说"。故答案为 D。 3.A 推理判断题。文章主要说明的是在阳光充足的环境中生长的咖啡豆对周围环境的影响, 文章主题是有关农业的,由此可判断出本篇文章来自一本农业杂志。故答案为 A。 4.A 文章结构题。首先文章第一段提出在阳光充足的环境中生长的咖啡豆会带来一些危害; 第二、三、四段通过具体的事例来说明第一段的观点;第五段中,作者总结上文内容并给出 了自己的观点。故答案为 A。 【长难句解读】It is obvious that the way much coffee is grown affects many aspects of life, from the local environment to the global ecology.分析:根据句子结构来分析,此句中 It 是形式主语, 真正的主语是 that 引导的从句;much coffee is grown 是一个定语从句,修饰先行词 the way。 译文:很显然,许多咖啡的种植方式影响着生活的许多方面,从当地环境到全球生态。 Passage 7 本文介绍了在非洲骑着马近距离观赏野生动物的奇妙之处。 1.C 代词指代题。上两句中谈到了"Several safari camps"给游客提供的服务以及它们的特点, 237 本句又说明它们因为给游客提供美妙的骑马的经历而闻名,因此 They 指的是上文提到的 "Several safari camps"。 2.D 细节理解题。依据第三段的第四、五、六句可知,作者认为坐在马背上近距离接触那 些野生动物最让人激动。 3.B 词义猜测题。依据上一句中的"a more relaxed and unhurried pace"可知,傍晚的旅程是 缓慢的、平静的, 故选 B 项。 4.B 写作手法题。作者在第三段中先说了早上的旅程,第四段涉及晚上的旅程,尾段谈到 回到营地,由此可知作者是按时间顺序写的。 Passage8 保护古建筑与城市的发展是对矛盾,如何解决这个矛盾?作者就此谈了自己的看法。 1.A 细节理解题。根据第一段第三句"Not all historical buildings are attractive"可知并非所有 的历史建筑都有吸引力,也即部分没有吸引力。故选 A。 2.C 细节理解题。根据第三段首句"It is true that there are examples of new buildings which have spoilt (破坏) the area they are in, but the same can be said of some old buildings too"可知,作者 认可有些古建筑破坏了其所在的地区。故选 C。 3.C 短语猜测题。根据画线部分后的"If we always reproduced what was there before, we would all still be living in caves. Thus, I would argue against copying previous architectural styles and choose something fresh and different, even though that might be the more risky choice"可知选 C。 4.D 写作意图题。本文为议论文,作者就热门话题——古建筑保护与新建筑的建设是否矛 盾的问题发表了自己的看法。作者在第二段提出自己的观点:In my view, new architectural styles can exist perfectly well alongside an older style. 接下来就此进行了论证。故选 D。 Passage 9 虽然很多动物比如蜥蜴可以在水上行走,但是在水上行走一直是人类难以实现的梦想, 本文介绍一种可行的原理,不过目前仍然处于理论研究层面。 1.B 细节理解题。根据第一段第二句可知答案,人类不能在水上行走主要是因为受到其生 理上的局限。 2.C 推理判断题。根据第二段第四句中的"20 steps per second"可知,一种蜥蜴能够在水上行 走,速度是非常关键的。B 项干扰性很强,但是蜥蜴并不是待在水面上,需要用脚快速击打 水面。 3.A 推理判断题。根据第三段第二句可知,玉米粉的作用是使水变稠。"thickening"与 A 项 238 中的"thick"一致。 4.D 推理判断题。根据最后一段的陈述可知,该设想理论上可行但是目前尚不能实现。 Passage 10 本文主要介绍了探戈舞的起源和发展过程等。 1.D 细节理解题。文章第一、二段介绍的是探戈舞的起源,belly dancing 是当时在欧洲流 行的舞蹈之一,与探戈舞的起源无关,因此 A 项可排除;一战归来的美国士兵将探戈舞带 到了美国,这属于探戈舞的发展史,而不是起源,B 项也可排除;C 项和 D 项为两个地名, 通读第一段可知,探戈舞起源于阿根廷的首都 Buenos Aires,故选 D。 2.C 细节理解题。A 项的内容文中没有提及,故排除 A 项;根据第二段的首句"At the beginning the tango was a dance of the lower classes."和最后一句"Gradually, the dance spread into the upper classes of Argentinean society and became more respectable."可排除 B 项;D 项与 最 后 一 段 最 后 一 句 中 的 "and in 2003 the Argentinean embassy in Seoul hired a local tango dancer to act as a kind of dance ambassador, and promote tango dancing throughout South Korea" 不符;根据第二段的第三句"At that time there were many fewer women than men, so if a man didn't want to be left out, his only choice was to dance with another man so that he could attract the attention of the few available women."可知 C 项正确。 3.C 细节理解题。通读全文后可知,探戈舞传到美国是在第一次世界大战之后,传到日本 是在 1926 年,在韩国流传开来是在 2003 年,显然 A、B、D 三项均可排除。根据第三段的 内容可知,探戈舞在其诞生后不久就传到了法国,其诞生大约是在 1870 年,因此传到法国 应该是在一战前。故选 C。 4.B 标题归纳题。本文介绍的是探戈舞的起源及其发展史,故选 B。 【长难句解读】At that time there were many fewer women than men, so if a man didn't want to be left out, his only choice was to dance with another man so that he could attract the attention of the few available women. 分析:这是个 so 连接的前后为因果关系的并列句。At that time there were many fewer women than men 为第一分句,so 后的句子为第二分句,第二分句中又包含一个条件状语从句:if a man didn't want to be left out,his only choice was to dance with another man 为这个条件状语从 句的主句,so that he could attract the attention of the few available women 为目的状语从句。 译文:当时,(跳探戈舞的)女性比男性少得多,所以,如果某个男子不想被冷落的话,他 239 唯一的选择就是和另一个男子跳舞,以便能够吸引为数不多的女性的注意。 Passage 11 本文讲述了西红柿被西方人逐步接受的过程。 1.D 推理判断题。 根据文章第一段描述的"那些基督教徒认为苹果是一种邪恶的水果"以及 第一段的最后一句可推知,西红柿被拒绝的原因与宗教有关,故选 D 项。 2.C 推理判断题。根据第三段的第一句中的"Cautious Europeans long ignored the tomato"可 知,谨慎的西方人长期忽视西红柿。故选 C 项。 3.B 推理判断题。根据最后一段中 Robert Johnson 说的话"I'll show you ... things are good to eat!"可知,他想推广西红柿,让人们接受西红柿。故选 B 项。 4.C 写作意图题。本文讲述了西红柿被西方人逐步接受的过程。故选 C 项。
查看更多

相关文章

您可能关注的文档